Brain 101 Info (Merged).pdf

March 26, 2018 | Author: rosyua | Category: Type I And Type Ii Errors, Dominance (Genetics), Informed Consent, Cardiac Arrest, Allele


Comments



Description

www.brain101.info 1 BEHAVIORAL SCIENCE  Bipolar I is characterized by manic and depressive episodes but could be by a manic episode alone. Bipolar II is hypomania and depression .. never manic  Id says: "I want it!" Superego says: "you know you can't have it!" Ego is the mediator between the two and deals with the conflict. - the ego id and superego operates on an unconscious level. the id is all your instincts ,the things that you would do if you did not have a super ego who will say stop so the super ego is ALL THE MORAL VALUES THAT A PERSON WILL HAVE LIKE IF YOU WANT TO STEAL SOMETHING (ID)THE SUPEREGO WILL SAY NO IT IS BAD (IT IS NOT MORAL) The ego links both of them. The ego controls the expression of the id so it can adapt to the external world.  Hardy-Weinberg Law -------------------- Developed in 1908, the Hardy-Weinberg law is an algebraic formula to estimate the frequency of a dominant or recessive gene in a population based on the frequency with which the trait or condition is found in that population. The derivation takes several steps, but the final formula is... p2 + 2pq + q2 ...in which... § p = frequency of the dominant allele in a population § q = frequency of the recessive allele in the population, and § p + q = 1 Using the frequency for a condition known to be inherited in either an autosomal- recessive or autosomal-dominant manner, the frequency of the genes involved can be assessed. This can be important information in designing and delivering programs to screen for specific mutant genes in targeted populations. An Example: Cystic Fibrosis Cystic fibrosis is a recessive condition that affects 1 in 2,500 Caucasian babies. What is the frequency of the gene responsible for cystic fibrosis in the Caucasian population? A-PDF Merger DEMO : Purchase from www.A-PDF.com to remove the watermark www.brain101.info 2 If... § we say that affected individuals have the aa genotype, and § the frequency of the aa genotype is indicated by q2 in the Hardy-Weinberg equation ...then... § q2 = 1/2,500 and q equals the square root of 1/2,500, or § q = 1/50 = 0.02 The frequency of the cystic fibrosis allele in the Caucasian population is therefore 2% (0.02). Since... § p + q = 1 ...then... § p = 1 – 0.02 = 0.98 The frequency of the "normal" allele in the Caucasian population is therefore 98% (0.98). Lastly, since... § 2pq = frequency of heterozygotes or carriers ...then in this example... § 2pq = 2 _ 0.98 _ 0.02 = 0.04 That is, 1 in 25 Caucasians (0.04) are carriers of the cystic fibrosis gene. These numbers differ in other subpopulations (such as racial or ethnic groups) of the world. It is important to note that with random mating, no selection or mutation there will be no change in gene frequencies. The recessive trait does not die out, and it is possible that in fact the recessive trait may be very much more common than the dominant trait!  SAMPLE PROBLEMS Hardy-Weinberg Problem 1) If 9% of an African population is born with a severe form of sickle-cell anemia (ss), what percentage of the population will be more resistant to malaria because they are heterozygous(Ss) for the sickle-cell gene? - Ans. 9% =.09 = ss = q 2 (¶)s = q = Square root of .09 = .3 p = 1 - .3 = .7 2pq = 2 (.7 x .3) = .42 = 42% of the population are heterozyotes (carriers) www.brain101.info 3 2) After graduation, you and 19 friends build a raft, sail to a deserted island, and start a new population, totally isolated from the world. Two of your friends carry (that is, are heterozygous for) the recessive cf allele, which in homozygotes causes cystic fibrosis. A. Assuming that the frequency of this allele does not change as the population grows, what will be the instance of cystic fibrosis on your island? B. Cystic fibrous births on the island is how many times greater than the original mainland. The frequency of births on the mainland is .059%. - Ans. A. There are 40 total alleles of the 20 people of which 2 alleles are cystic fibrous causing. 2/40 = .05 the ¶ of the cystic fibrous allele thus cc or q 2 = (.05) 2 =.0025 or .25% of the population will be born with cystic fibrous. B. 0.25/.059 = about 4 times greater occurrence Hardy-Weinberg Problem 3 This is a classic data set on wing coloration in the scarlet tiger moth (Panaxia dominula). Coloration in this species had been previously shown to behave as a single-locus, two-allele system with incomplete dominance. Data for 1612 individuals are given below: White-spotted (AA) =1469 Intermediate (Aa) = 138 Little spotting (aa) =5 Calculate the following frequencies: (_)A = (_)a = (_)AA = (_)Aa = (_)aa = Solution to Problem 3 (_)A = (2 * (1469) + (138))/(2 * (1469 + 138 + 5)) = .954 or 95.4% Q: How did we arrive at this data? (_)a = 1 - .954 = .046 or 4.6% (_)AA = (.954) 2 = .910 or 91% (_)Aa = 2 (.954)(.046) = .087 or 8.7% www.brain101.info 4 (_)aa = (.046) 2 = .002 or .2% Ans. genotype___Number______A__________a_________Total AA_________1469____2*1469(2938)___0__________2938 Aa__________138_______138________138__________276 aa____________5_________0_______2*5(10)________10 Totals_____1612______3076________148_________3224 now let's count frequency Frequency of A = 3076/3224 =95.4% Frequency of a = 148/3224(or 1-A= 1-95.4%)= 4.5% > Another good sample problem on H-W Equation: In a certain place in Europe, it was found out that the prevalence of a newly discovered autosomal recessive disorder is 1/100. What is the carrier frequency of this disease? Ans. 0.18 using Hardy-Weinberg Explanation: Prevalence is = q^2=1/100=0.01 so q= 1/10=0.1 carrier frequency = 2pq p= 1-q = 1-0.1=0.9 carrier= 2*0.1*0.9= 0.18 So the carrier frequency is 0.18  TYPE 1 vs. TYPE 2 ERROR In Type 1 error, null hypothesis is rejected when it is actually true. Is it the same as false negative error??? In one note it was given as false positive. Pl clarify thanks - Ans. It's not the same. The sensitivity of a test is a measure of its ability to detect the presence of a disease in those who truly have the disease (true positive) - The specificity of a test is a measure of its ability to detect the absence of a disease in those who truly have no disease (true negative) - I find a simple explanation of type I vs type II here:  > I just want to share - - I asked this from my review class: what's the difference between type I (alpha) and type II (beta) error ? Our mentor answered us with a question from qbank: A group of researchers mistakenly conclude from a poorly designed experiment that acetaminophen cures the common cold. They have committed what error? www.brain101.info 5 Answer: Type I error Explanation: First we know that.. null hypothesis (Ho): no difference alternative hypothesis (H1): there is difference Type I error is rejecting the null hypothesis, when in fact it should not be rejected. Therefore, the study has really “no difference” or the study is really “not significant.” But since he made the type I error, the researcher thought that it “has difference” or “it is significant.” In type II error, null hypothesis is not rejected, when in fact it should be rejected. Therefore, the study really “has difference” or the study is really “ significant.” But since he made the type II error, the researcher thought that it has “no difference” or it is “not significant.” -> To change the question above to make a type II error: A group of researchers mistakenly conclude from a poorly designed experiment that acetaminophen has no effect on the common cold. They have committed what error? Answer: Type II error  Part I (ETHICS)  RULES OF ETHICS (Posted by SMART DOC from IP 202.141.238.2 on August 13, 2002 at 08:40:490). HOW TO DEAL WITH A DYING PATIENT: 1) Tell the patient EVERYTHING. There is no excuse for not doing so. If you know, the patient knows. 2) DO NOT GIVE FALSE HOPE 3) Allow the person to talk about his feelings 4) Kept he patient involved in social activities 5) Avoid social isolation GENERAL RULES: 1) “Substituted judgment”: when a patient cannot make a decision, the decision is made based on what is thought WOULD BE that person’s choice. The decision is finally made by who is most likely to represent the patient’s own wishes (not necessarily who is closest next of kin). 2) “Best interest standard”: trying to determine what a never-competent patient would have wanted is practically impossible. When you are not clear about the patient’s wishes, you should make the decision as a dispassionate, rational observer: do what a rational person would do. It is not your personal preference is. DO WHAT MOST PEOPLE WOULD WANT in this circumstance. “What would a jury of 12 people do if they knew what I know?” Who makes the decision is not really important: anybody using the best interest standard should arrive to the same decision. You must set aside your personal preferences: like strong religious beliefs (that is considered irrelevant) www.brain101.info 6 As a general rule, parents cannot withhold treatment from their children. Yet, in Infant Doe’s case, they did. In this case, the best interest standard rule was applied. 3) Patients decide over their own bodies: The patient ALWAYS MAKES THE DECISION. This was decided over the ROE vs WADE case in 1973, the case that made abortion legal. The issue will never be over abortion in the USMLE, but it illustrates the principle that governs medicine in the US: the patient always decides, and the only thing a doctor can do is lay out the possibilities. SPECIFIC RULES: Rule #1: Competent patients have the right to refuse medical treatment, no matter what. Rule #2: Assume that the patient is competent unless clear behavioral evidence indicates otherwise. -Drunk, schizophrenia, Alzheimer’s: these are all medical dg. DIAGNOSIS SAYS NOTHING ABOUT THE LEGAL COMPETENCE OF A PERSON!!! Competency can ONLY be decided by a COURT OF LAW: it is not a medical dg, it is not a blood alcohol level! Clear behavioral evidence of incompetence: Attempted suicide Patient is grossly and evidently psychotic and dysfunctional Patient’s physical or mental state prevents communication However, when in doubt, assume competency! Rule #3: Decision-making should occur in clinical setting if possible, without going to court. Normally, the USMLE will want YOU to make the decision: try to avoid the answer that says “go to court”, unless it is clearly stated that the guardian (ex: parent of a sick child) is NOT acting in the patient’s best interest. And that is only if the case is not an emergency: if it can wait going to court. Rule #4: When surrogates make decisions for a patient, they should use the following criteria and in this order: 1) Patient expressing wishes in the past: what historically did the patient say in the past? (wish for organ donation expressed to relative, for example) 2) What would the patient want? : Substituted judgment 3) Best interest standard: what would most people want www.brain101.info 7 Rule #5: If patient is incompetent, physician may rely on advance directives Directives that a patient can leave for his doctor before becoming incompetent: - Can be oral directive from patient to his doctor: does NOT necessarily have to be a written document. - Can be living will: expression in writing, notarized, by the patient. - Health powered attorney: person that was named by the patient to represent him. TREAT THIS PERSON AS THE PATIENT HIMSELF, IN TERMS OF DECISIONS: this person is the VOICE of the patient: a health powered attorney BEATS ALL OTHER CHOICES ON THE USMLE (it is the patient talking to you). Rule #6: Feeding tube is a medical treatment and can be withdrawn at the patient's request. A competent patient has the right to refuse hydration and nutrition. Period. In the case of anorexia nervosa: if the patient is a minor: not legally competent. If NOT a minor: go to court. Refusing food and water: may seem close to euthanasia, but on the exam this is accepted. Rule #7: Do nothing to actively assist the patient to die sooner. Do not ACTIVELY do anything (as opposed to number 6) Rule #8: the physician decides when the patient is dead. Futile treatment: means a treatment that is not AND WILL NOT improve anything. Still, if the patient or the family want the treatment to continue: it is not YOUR decision, it is the patient’s or the patient’s family. In case of clear cortical death: even if the family is hoping for a special doctor to arrive, for a special treatment to come: CALL THE DEATH. Rule #9: Never abandon a patient: even if they can’t pay you, even if you don’t like the patient. If you simply CANNOT continue to be the doctor to this patient: you need to arrange that he will have care and make sure that they are getting it. Never, ever threat to abandon your patient (not even if you are doing it to make sure they follow treatment). Rule #10: Always obtain informed consent: before you do ANYTHING!!! Informed consent can be oral. The patient can revoke written consent orally, at any moment Of the patient signs “consent” without reading it: it is NOT INFORMED CONSENT www.brain101.info 8 Informed consent: means that the patient understands: 1) Nature of the procedure 2) Purpose or rationale 3) Benefits of treatment or procedure 4) Risks 5) Availability of other alternatives “GAG CLAUSES”: you work for an institution that tells you not to discuss certain procedures or possibilities. THEY ARE ILLEGAL. Exceptions to informed consent: 1) Emergency situation 2) Waiver by the patient: the patient says it’s OK not to know what is going to happen (exploratory surgery, drug undergoing trial to know side effects) DO NOT ASSUME YOU HAVE A WAIVER UNLESS THE USMLE TELLS YOU. 3) Patient is incompetent 4) Therapeutic privilege: doctors have the right and obligation to deprive the patient of their autonomy in the interest of the patient and other people. Ex: patient on PCP, violent and dangerous: put him on restraints! Rule #11: Special rules apply with children Rule #12: Parents cannot withhold life- or limb-saving treatment from their children Rule #13: For the purposes of the USMLE, issues governed by laws that vary widely across states cannot be tested Rule #14: Good Samaritan Laws limit liability when physicians help at accidents Rule #15: Confidentiality is absolute Rule #16: Patients should be given the chance to state DNR (Do Not Resuscitate) orders, and physicians should follow them Rule #17: Committed mentally ill patients retain their rights Rule #18: Detain patients to protect them or others. Rule #19: Remove from patient contact health care professionals who pose risk to patients Rule #20: Focus on what is the best ethical conduct, not simply the letter of the law I looked it up: Rule # 8 says this: - if there are no more treatment options (if the patient is cortically dead), and the family insists in treatment?: if there are no options and there is nothing the physician can do, it is his duty to stop the treatment. (The USMLE wants you to be able to make decisions when the patient is DEAD) - - if the physician thinks tratment is futile and the patient won't improve, but the patient (or surrogate) insists on continued treatment: then treatment must continue. www.brain101.info 9 Rule # 17: Commited mentally ill adults legally are entitled to the following: - they must have treatment available - they can refuse treatment - they can command a jury trial to determine sanity They lose only the civil liberty to come and go they retain their competence for everything UNLESS A COURT OF LAW DECIDES they are incompetent. The underlying rule here is that no matter what the psychiatric diagnosis is, treat the patient as you would any other competent person (unless they show signs of clear incompetence, stated on #2) rule #20: Focus on what is most ethical. USMLE wants you to pick the answer where there are no doubts that it is the most ethical thing to do. In other words, don't worry about being fired, sued or that your hospital may go to shreds if you do the "right thing". ACT NOT AS A LAWYER WOULD, BUT AS MOTHER THERESA WOULD. There is also something interesting that they pointed out: what do you do if you find out a collegue or fellow resident is having a substance abuse problem? Who do you talk to? RULES: - talk to the collegue and REMOVE him from patient care - if there is a direct employer or supervisor (like your residency program director) : TELL THE SUPERVISOR. Failure to do so will endanger patients, and will ALWAYS be the worong answer on the USMLE. What they meant was: the best way to get someone to treatment is if their employer forces them to: if they are afraid to lose their job. So don;t waste time talking to the person or the family or anyone: go to the supervisor. Part 2 (ETHICS) Ethical Terms Autonomy The principle of autonomy, or self-determination, entails respecting the choices and wishes of persons who have the capacity to decide and protecting those who lack this capacity. This principle would be used when a physician who has discussed preferences about life-sustaining treatment with a woman who has just been diagnosed with metastatic breast cancer is then guided by those wishes. www.brain101.info 10 Beneficence The principle of beneficence advises physicians to benefit patients and protect their interests, whereas the principle of nonmaleficence encompasses the oft-quoted phrase, "Above all, do no harm." Physicians must balance the risks and benefits of any proposed treatment. An example of beneficence would include a physician who refuses to provide a prophylactic mastectomy for a patient who fears cancer but does not have any unusual risk factor for the disease. Justice The principle of justice entails providing persons with that to which they are entitled and treating similar cases similarly. Together, the principles comprise the foundation of ethical principalism, the dominant approach in bioethics today. Informed Consent The ethical foundation on informed consent can be traced to the promotion of two values: personal well-being and self-determination. To ensure that these values are respected and enhanced, patients who have the capacity to decide their care must be permitted to do so voluntarily and must be provided all relevant information regarding their condition and alternative treatments, including possible benefits, risks, costs, other consequences and significant uncertainties surrounding any of this information. Informed consent is a two-art process involving disclosure by the health care professional and a decision by the patient. If health care professionals support a patient's right to make informed choices, they must respect the patient's decision regarding whether to accept or decline treatment. The consent form provides written documentation of the patient's decision but it is not a substitute for a thorough discussion between their physician and the patient. Three elements of informed consent must be considered. These elements include (1) the information to be discussed, including the nature of the intervention described in sufficient detail, (2) the patient's comprehension, (3) the patient's decisional capacity and (4) the voluntary nature of the decision. The physician should carefully discuss the proposed purpose of the intervention, including how any knowledge gained from the procedure will change the treatment course or outcome. The likely risks of the proposed intervention must be fully disclosed together with a discussion of their severity and likelihood. The likely benefits of the proposed intervention should be explained. Life-Sustaining Treatment Life-sustaining treatment may include but is not limited to a mechanical ventilator, renal dialysis, chemotherapy, antibiotic thereapy and artificial nutrition or hydration. Physicians, ethicists and lawyers widely agree that withdrawing or withholding life-sustaining treatment is legally and ethically permissible under appropriate circumstances. In fact, ethical and legal principles require that physicians respect the decision to forego life-sustaining treatment by a patient who has decisional capacity. However, if a physician is morally opposed to the patient's treatment preferences, the physician may transfer care to another physician who is more comfortable with the patient's wishes. www.brain101.info 11 In addition, a competent patient may make his wishes known in advance of losing the ability to make health care decisions, such as a living will. In such cases, physicians are morally and legally required to follow these health care preferences to the extent permissible by law. Medical Futility Situations in which a patients underlying diagnosis or diagnoses impart a terminal or poor prognosis thus rendering specific medical interventions unhelpful and possibly detrimental. Medical Treatment Options Cardiopulmonary Resuscitation (CPR) in Older Persons The indications for CPR have changed considerably since its inception in 1960. Initially a treatment for sudden death in the setting of an acute myocardial infarction, CPR has become a procedure utilized in death from any cause. In the absence of a Do Not Resuscitate (DNR) order, CPR is often done by default, despite a growing body of literature that indicates CPR is often ineffective, particularly in persons dying of non-cardiac, multi-system diseases. Several studies have evaluated the efficacy of CPR in older persons. In frail older persons who are dependent in their activities of daily living, CPR is usually not effective. Less than 2% of patients living in nursing facilities who receive resuscitation survive. Those who do survive are often more debilitated than before the cardiac arrest. The outcomes of older adults in the community who have a cardiac arrest are equally poor. Functionally, active older persons with primarily cardiac disease, who suffer a witnessed arrest in the hospital, may fare better. Hospitalized older patients suffering cardiac arrest have a survival rate of 26%. Some patients are more functionally dependent after the arrest. When people begin to develop functional disability and accrue chronic illnesses, their survival drops precipitously, regardless of age. Artificial Feeding Near the End of Life Food is an essential requirement of life and without it death is certain. The symbolic nature of food is very powerful and firmly rooted in our culture and religious beliefs. As such, the decision to not provide food is often a difficult one. The delivery of food through artificial means is a medical therapy. It may be instituted, withdrawn or refused like any other medical treatment. However, a person must declare clearly that they do not want artificial feedings. This statement can be as simple as: "If I were to become so ill that I could not talk meaningfully with my family and the hope of me regaining that ability was small, I would not want artificial feeding." It is useful to have statements like this included in the Health Care Proxy form (HCP), or to state in the form that your health care agent knows your wishes regarding artificial feeding. If a person feels strongly that they would always want to have artificial feeding, they should make sure their health care agent knows their wishes (and if they have a HCP they should note this on the form). Much of the information about what happens to people when they decide to forego artificial feeding comes from the hospice and oncology literature. People working in hospice have noted www.brain101.info 12 that their patients suffer very little because of not eating. Dying patients who choose not to have foods delivered by artificial means, do not suffer from hunger and thirst. Patients that do experience hunger and thirst, can have their symptoms relieved with mouth care, and small amounts of food and fluids that they chose to eat. Studies in the oncology literature have shown that cancer patients have a higher mortality and morbidity who when they receive aggressive artificial feeding, in comparison to those that do not. In patients with strokes who are fed with gastrostomy tubes, the median time of survival is only 53 days (range 2 - 528 days), with only 12% of the patients surviving for more than three months. No study has shown that dying patients live longer or are more comfortable as a result of artificial feeding. Enteral tube feedings are not without side effects. One of these side effects is aspiration pneumonia, which occurs in 50% of patients. When feeding tubes are used in patients with confusion or dementia, self extubation is common and often results in the use of restraints. Some rules of thumb to consider: Dying patients should choose what they want to eat, when they want to eat and the amounts of food that they want to eat. For some patients, the act of eating is still important even though they may eat only very small amounts of the food presented to them. Almost all dietary restrictions should be lifted in older adults especially for dying persons. Food should be fed but never forced. Education of family and caregivers concerning artificial feeding is very important. Food should never be withheld from a dying patient who desires it. Artificial feeding has substantial risks, especially aspiration and self-extubation, that often results in the use of restraints. To date, no studies have demonstrated that artificial feeding improves morbidity or mortality in dying patients. None of the predominant religions in the USA demand that artificial feeding be administered to dying patients. All are firm however, that food never be withheld from a patient who wants it, or with the intent to cause pain or death. Severe anorexia and loss of thirst is often part of the dying process, no matter what the cause. Organ function slows and metabolic processes shut down.  Re: DNR Do Not Resuscitate (DNR), without any qualification means no cardiopulmonary resuscitation. It does not mean terminate treatment, or refrain from simple health-supporting or even life-saving procedures. With DNR orders in place, a patient would still be given medication and might even undergo surgery. What if the patient is choking, and in need of a simple intervention to save his life. Nothing is implied by DNR that says we should not perform the Heimlich maneuver. Do it. www.brain101.info 13  Recall: - 1. Two nominal - Chi square 2. Two interval - Pearson correlation 3. One nominal and one interval - T-test 4. mathematical based literature review- Meta analysis NEUROTRAMSMITTER DISTURBANCE IN DIFFERENT DISEASES (Ref. P 130 FA) - - Schizophrenia increase in dopamine and serotonin - Depression decrease in dopamine ,serotonin (5HT) and norepinephrine - Anxiety decrease in GABA (remember antianxiety drgudssuch benzodiazepins acts onGABA receptors) and serotonin and increase in norepinephrine - Mania increase in dopamine - Alzheimer decrease in ACH - Parkinson’s Disease – decrease dopamine - Huntington’s Disease – decreased GABA, decreased Ach > US-PREVENTIVE SERVICE TASK FORCE PRE-SERVICE-----SEX----AGE physical-exam:- --------------- BP-----------M/F------18 +----q 2YRS CL- BREAST EXAM--F---50-69---q 1-2YRS LABS-TESTS:- PAP-SMEAR-------F---18-65 YRS----q 3 YRS STOOL FOR OCCULT BLOOD---M/F--50+ YRS---ANNUALLY. SIGMOIDDOSCOPY---M/F--50+YRS q ? YRS MAMMOGRAPHY---F----50-69 YRS--q 1-2 YRS CHOLESTROL---M----35-65 YRS---q ? YRS ---F----45-65 YRS IMMUNIZATIONS:---- TETENUS-DIPTHERIA BOOSTER M/F---18+ YRS---q15-30YRS INFLUENZA-VAC---M/F--65+ YRS--ANNUALLY PNEUMOCOCCAL VAC--M/F---65+YRS--ANNUALLY. COUNSELLING----M/F----18+YRS----AT ROUTINE VISIT (SOURCE- CMD 2003) Note: PNEUMOCOCCAL VACCINE SHOULD BE GIVEN ONCE AFTER 65 YRS, NOT ANNUALLY. (CDMT 2003) www.brain101.info 14  RE: COMPETENCY - Assume the patient is competent unless clear behavioral evidence indicates otherwise. -Drunk, Schizophrenia, Alzheimers = all COMPETENT -INCOMPETANT: 1. ATTEMPTED SUICIDE 2. PATIENT IS GROSSLY AND EVIDENTLY PSYCHOTIC AND DYSFUNCTIONAL 3. PATIENT'S PHYSICAL OR MENTAL STATE PREVENTS COMMUNICATION... CAN EVERYONE TRY TO THINK OF EXAPMPLES OF 1, 2, AND 3! are the following competant or not? minor with anorexia nervosa? Adult with anorexia? Mentally retarded? Autistic? Personality disorder? Murderer? Drug addict (presently under influence of drug?) - minor with anorexia , is still a minor, parents decide adults with anorexia, competent MR, not competent autistic , not competent personality disorder, if gross enough, incompetent, also if has history of suicide , then definatLy not competenT murderer, competent to make decisions for himself, unless, gross abnormal behaiour observed addict, under the inflence not competent. - Hey guys ,I have seen a q about MR that u shouldNOT consider all of them noncompetent!It depends on situation and some legal documents r needed(from court)that shows he/she is incompetent. - I think that mental retardation is Competant- they live by themselves, and can function in society. But what about Autimsm- i think Incompetant because they cannot communicate. ============================================================== www.brain101.info 15 BEHAVIORAL SCIENCE QUESTIONS 1) A 60-year-old man with a 5-year history of stable angina pectoris sustains a head injury in an automobile accident and dies. The family is contacted and consents to an autopsy. Microscopic examination of a section of heart muscle would most likely reveal which of the following pathologic findings? A. Coagulative necrosis of the subendocardial muscle B. Dense scar localized to the distribution of one coronary artery C. Focal fibrosis and subendocardial myocardial vacuolization D. Heavy neutrophilic infiltrate adjacent to a large area of coagulative necrosis E. Transmural coagulative necrosis - Ans. B. Necrosis require 8-12 hours to form (reject A and E) Neutrophils will be there in 4-6 hours (reject D). C doesn't connect with stable angine at all. But may be a cause of arrhytmia. 2) A 72-year-old man is admitted to the hospital after suffering a stroke. His family comes to visit him the next day only to discover that he does not recognize them. When visited by the team of doctors, the patient seems to be able to recognize voices and sounds, but claims that he has never seen his wife and son before. He seems confused because when they talk to him their voices are just like the ones of his family members. Which of the following is the most likely diagnosis? A. Anosagnosia B. Confabulation C. Conversion disorder D. Ganser syndrome E. Prosopagnosia - Ans. E, prosopagnosia....inability to recognise faces,due to destruction of part of occipital lobe adjucent to area 21,22 of temporal lobe. 3) A 49-year-old patient is evaluated for suicidal ideation after he is found laying on train tracks by police. The man is disheveled and malodorous and states that he has "reached the end" and would rather die. He admits to depressed mood, anhedonia, poor energy and appetite; he feels miserable and regrets what he has done with his life and wants to put an end to it. He states he has felt this way since age 26, after he was discharged from the military. The man indicates that his life was "great" until he increased his drinking, which caused a divorce at age 30. He has had two arrests for driving under the influence. He was in jail for 6 months after he had an accident while drunk that resulted in public property damage. He remembers that he initially felt sick in jail, with sweating, vomiting, shaking, and he experienced a seizure. He then improved after a few days and felt better during the rest of his imprisonment without any depression. Which of the following criteria most strongly suggests alcohol abuse? A. Desire to cut down B. Recurrent drunk driving C. Seizure after withdrawal www.brain101.info 16 D. Suicidal ideation E. Tolerance - Ans. C? 4) A 10y/old boy was noted for his extreme fear of water during his first day of swimming lessons. The teacher helps the child sit on the edge of the pool and splash the water with his feet. She then goes one step further and shows him how to wet his knees. It takes her an hour to have him float on the water while holding his hand. What technique has the teacher used to help the child conquer his fear of water? A. Classical conditioning B. Desensitization C. Extinction D. Flooding E. Operant conditioning - Ans. B? 5) The mother of a 6-year-old boy brings him to see the local pediatrician complaining that the boy is enuretic from 4 to 5 times a week. She reports that the problem began in the past few months after she gave birth to a baby girl. Breast-feeding for the new baby has been difficult and has consumed a great deal of her time each day. She says that the boy is very embarrassed about his enuresis and is afraid that the other children where he goes to school will find out and make fun of him. She asks the physician help and advice. The physician’s best course of action would be to do which of the following? A. Arrange to interview the boy about his condition B. Direct her to have a serious discussion with her son about whether anything is bothering him C. Prescribe a course of imipramine for the boy D. Refer the mother to a seminar on breast feeding E. Send the boy to a child psychologist for counseling F. Suggest that the mother block out some special time each day and give exclusive attention to her son G. Tell the mother that the boy’s problem is normal and temporary, and will soon pass if left alone - Ans. F? 6) A nurse on an inpatient internal medicine ward comes to see the attending physician. While drawing blood for routine laboratory tests ordered by the medical staff, the nurse inadvertently stuck herself with a hypodermic needle, in which were several drops of the patient's blood. The nurse is anxious, and wants the physician to order that the patient's existing blood sample be tested for HIV. The physician is aware that the patient has a history of homosexual encounters, although neither the physician nor the nurse are aware of the patient's HIV status. At this point, the physician should do which of the following? A. Assure the nurse that the probability of contracting HIV by this method is relatively low, but that she should be more careful in the future B. Convene a meeting of the nursing staff and ask if anyone on the ward is aware of the patient's HIV status www.brain101.info 17 C. Order the test, as the nurse requests D. Review the patient's chart and medical history for clinical signs consistent with HIV infection E. Talk to the patient and order the test only if the patient gives his permission F. Tell the nurse that you will order the test if she can obtain the patient's permission. - Ans. C or E? 7) A 36-year-old married woman complains to her physician that she is having trouble sleeping. A detailed history shows that her insomnia is sporadic and seems to be connected to cyclical stressors related to her working environment. The physician prescribes alprazolam to be taken “as needed.” The next day, the physician receives a distressed call from the patient. With anger in her voice, she tells the physician that the “pharmacist said that taking this medication might cancel out the effects of my birth control pills.” At this point the physician’s next response should be which of the following? A. “I’m sorry. This is my fault. The problem is not very likely given the dose level I prescribed and your only occasional use, but I should have discussed this issue with you before.” B. “It’s not the pharmacist’s job to be tinkering with your medications. I suggest you have the prescription filled somewhere else.” C. “Its really such a small chance that it is not worth worrying about.” D. “Really, its nothing to worry about. I’ll call the pharmacist and work it out.” E. “Really, there is no problem here. Pharmacists just like to show what they know.” F. “The pharmacist is being overly cautious. As long as you take both medications as I prescribed them for you, you will have no problem.” G. “Well, if you don’t like the drug I prescribed, what would you rather have?” H. “You seem angry about this. Tell me more about what you are feeling right now - Ans. A? 8) A 41-year-old married woman of Asian decent becomes pregnant with her first child. During the course of routine prenatal care, the women undergoes a series of tests checking on her health and the health of the fetus. The results of the tests suggest the woman is in good health, but strongly indicate that her child will be born with Down syndrome. When informed of this result the woman becomes visibly upset and begins to cry. “How could this happen to me?” she says, “God must be punishing me!” At this point the physician’s best reply would be which of the following? A. “I don’t think God has anything to do with this. This sort of thing just happens some of the time.” B. “I know it is hard to heard this kind of news, but let me assure you that you are still young enough to have other children.” C. “Let’s take a moment to reflect and pray together for guidance.” D. “Sometimes God works in mysterious ways that we can not understand. We just have to try to keep our faith.” E. “Take some deep breaths and try to relax. When you collect yourself, we can talk about how you want to proceed.” F. “Tell me a bit more about why you think God is punishing you.” www.brain101.info 18 G. “The chances for Down syndrome are simply higher when a woman your age becomes pregnant. That’s why we run these tests.” H. “The real issue before us is, how do you want to proceed? Do you want to carry the child to term or explore other options?” - Ans. E or H? 9) A 24 year old woman comes to see her male physician complaining of pain and stiffness in the left lower quadrant of her back. During the physical examination to locate the extent and severity of the pain she says, “Wow, you’ve got great hands, Doc. Just having you touch me helps me feel better.” As the patient is getting dressed, she asks the physician if he is married, and then asks if he ever dates his patients. The physician is unmarried, is attracted to the woman, and would like to see her socially. At this point his best action would be to do which of the following? A. Begin to see the patient socially as long as she understands that it must be kept separate from the medical relationship B. Explain that as long as she is his patient, no social relationship is possible C. Have the patient sign a liability waiver and begin to see her socially D. Refer the patient to a colleague and begin to see her socially E. Refer the patient to a colleague and refuse to see her socially F. Refer the patient to a colleague, wait six months, and then begin to see her socially G. Tell the patient that he will not pursue any social relationship with her, but that he would like to continue to be her physician - Ans. B? 10) A 64-year-old married man is admitted to the hospital complaining of abdominal pain. Symptoms are consistent with cancer of the colon, and an MRI reveals a large abdominal mass. The patient consents to surgery, and the mass is removed along with 1/3 of the patient's colon. The report from the pathologist confirms that the mass was cancerous and, further, that the tumor had extended though the colon wall, and metastasis is likely. Under the circumstances, the physician estimates life expectancy at less than one year. The patient has not yet been informed of either the extent of the cancer's spread or his projected life expectancy. As the physician enters the patient's room to inform the patient of the negative prognosis, the patient is sitting with his wife and teenage daughter. After introducing himself, the best thing for the physician to say would be which of the following? A. "I have some bad news. Would you like to discuss it in private or would you like to have your family present?" B. "I have some things to discuss with you in private. Would you mind having your family wait outside?" C. "I'm glad that your family is here. Can any of you recall whether any of your close relatives have ever had cancer?" D. "I'm glad that your family is here with you. I have some things to discuss with you." E. "This is the part of my job that I hate the most. I came here today to tell you that the cancer has very likely spread." F. "Well, lets get right to it. I'm afraid that I have some bad news." G. "We need to talk about your condition. Would you like your family to be present? www.brain101.info 19 - Ans. B or G? 11) A 34-year-old man comes to see his physician complaining of sore throat, stuffy nose, and difficulty sleeping at night. The physician diagnoses him with a sinus infection and writes him a prescription for a course of a common antibiotic. On the last day of the prescribed course of antibiotics, the man calls his physician and leaves a message saying that he is feeling better, but does not feel fully recovered. He asks, in his message, that the physician calls the local pharmacy with a refill of his prescription for an additional 7 days so that “I can really kick this thing.” After he receives the message, the physician’s best course of action would be to do which of the following? A. Call the local pharmacy and ask to have the prescription refilled B. Call the patient and tell him that he will be fine and does not need any further medication C. Call the patient back and explain that antibiotics cannot be extended beyond the normal course without the risk of side effects D. Call the patient back and talk with him about his condition, and, if all seems well, call the pharmacy to order that the prescription be refilled E. Call the patient, schedule a follow-up appointment and reevaluate the patient’s condition F. Have the office nurse call the patient to schedule a follow-up appointment - Ans. E 12) Which of the following diseases should be reported to the Department of Public Health? A.Candida albicans infection B. Condyloma acuminatum C. Gonorrhea D. HIV infection E. Streptococcal pharyngitis - Ans. C? 13) A 22-year-old, married woman presents with pain in her lower right abdomen. Her abdomen is tender to palpation and there is rebound tenderness. The patient reports nausea, and has a temperature of 38.3 C (101 F). A diagnosis of acute appendicitis is made, and the patient consents to, and is scheduled for immediate surgery. Prior to the surgery, the woman tells the surgeon that she is a practicing Catholic, and says that she will pray for the successful completion of the surgery. Surgical examination of the appendix reveals that it is normal and without inflammation. Examination of the abdominal cavity reveals that the true underlying cause of the patient’s pain is an ectopic pregnancy on the right side. At this point, what action should the surgeon take next? A. Close the incision, wait until the patient recovers from anesthesia, and seek full informed consent before proceeding B. Consult the chief of surgery C. Consult with the local Catholic chaplain about how to proceed based on the woman’s expressed religious beliefs. D. Seek permission to operate on the ectopic pregnancy from the woman’s spouse E. Surgical intervention to deal with the cause of the pain www.brain101.info 20 - Ans. I think it should be D. In nonemergency, answer is A.? 14) A primary care physician (Dr. Green) in a community of 100,000 people enters an examination room to find a new patient sitting on a chair just inside the door. Although the patient had been instructed by the nurse to remove her clothes and have a seat on the examination table, the patient remains fully clothed, including her shoes. The patient glances up briefly when the physician enters the room and closes the door, and then returns her gaze to stare at the floor. She says nothing. At this point, the best thing for the physician to say next would be which of the following? A. "Didn't the nurse ask you to remove your clothes?" B. "Hello, I'm Dr. Green. How are you feeling today?" C. "I see that you are still dressed. Is there something you want to talk with me about?" D. "OK. Hop up on the table and let's have a look at you." E. "So, what brings you to see me today?" F. "You seem a little depressed. Why don't you tell me about it?" G. "You seem very quiet. I'll just sit here for a moment while you collect your thoughts." - Ans. B? 15) A 16-year-old girl comes to see her physician for a standard physical examination prior to attending summer camp. The examination is unremarkable. The girl falls within the normal range for height and weight. The physician notes that she has had menses for the past 4 years. As the girl is getting dressed, the physician sits and completes the required forms for the camp. When handed the completed forms, the girl thanks the physician and says, “Just one more thing. Could you also give me a prescription for birth control pills? I’m going to camp with my boyfriend and want to be prepared. And please, please, don’t tell my parents!” The physician’s most appropriate reply would be which of the following? A. “Before I write you that type of prescription, I’d like to examine your boyfriend first.” B. “How long have you been having sexual relations with your boyfriend?” C. “I can only give you a prescription like that if I have your parents’ permission. Let’s set up a time to talk with them.” D. “I wish you would reconsider. Sex before marriage can be very complicated.” E. “I’ll be glad to give you the prescription, but I want you to discuss things with your parents first.” F. “I’ll be happy to give you a prescription, but let’s talk about some important issues first.” G. “I’ll make a decision about the prescription after I have a discussion with you and your boyfriend. When should we schedule that?” H. “I’m pleased that you are responsible enough to be prepared. I’ll be happy to give you the prescription and see no reason to tell your parents.” - Ans. C or F? 16) A 76-year-old man of Korean descent goes to see his primary care physician complaining of chest pains, difficulty breathing, and general fatigue. When interviewed, he answers the physician’s questions respectfully in a soft voice with little eye contact. When questioned about any pain, he indicates that it is always with him. Subsequent www.brain101.info 21 examination and testing leads the physician to suspect that the patient might have lung cancer. When the possibility is mentioned to the patient, he becomes very quiet, stares at the floor and says softly, but distinctly, “I do not think I want to know this if it is true.” At this point the physician’s best reply would be which of the following? A. “I know cancer can seem a bit frightening, but modern advances in treatment give us some options you may not be aware of.” B. “I understand the custom of your culture is not to discuss these things, but I’ll need you to work with me so we can beat this.” C. “I understand your hesitation, respected Grandfather. I will work with your family if that is what you direct.” D. “I’m sorry, but I will need your cooperation as we arrange treatment. I will need to tell you and you will have some decisions to make.” E. “If it is your wish, I will not tell you. Whom in your family should I talk to in your place?” F. “In this country, patients have to be told everything. I’ll let you know when we have more definitive information.” G. “Let’s wait until we know something definitive and then we can talk about things further.” H. “OK, I’ll respect your wishes and not tell you.” I. “Tell me a bit more about why you do not want to know.” - Ans. H or A? 17) A 25-year-old HIV-positive woman gives birth to a 6-pound baby boy at a local health clinic. The woman has received no prenatal care. She is ecstatic about the birth, holds the child closely, and talks softly to the child every chance she gets. Tests performed to assess the child’s HIV status return positive results. When told of these results, the new mother appears unfazed, and says that she will just have to be an even better mother to “help the child through this.” She requests a consultation with a breast-feeding counselor because she says, “I want to make sure I do this right.” The physician tells her that breast- feeding is not advisable, to which she replies, “I know that breast-feeding is best, and I want the best for my baby.” The physician’s best reply would be which of the following? A. “I’m pleased that you are taking your responsibilities so seriously. I’ll arrange an appointment with the breast-feeding counselor for you myself.” B. “If you insist on breast-feeding your child, the courts will remove the child from your custody.” C. “If you really love your child, you will do what is best and not breast-feed.” D. “It is important that you listen to me carefully. Breast-feeding increases the risk to your child. You must not do it.” E. “It’s wonderful to see how happy you are. We can talk a bit more about these things after you’ve has some rest and have recovered from the birth.” F. “Let me explain. A positive test when the child is this young is not definitive. But if you breast-feed your child, you greatly increase the chances of your child contracting HIV.” G. “Yes, breast-feeding is best in most circumstances, but given your HIV status, I strongly advise against it.” - Ans. F? www.brain101.info 22 18) A 55-year-old male begins group therapy. After the first session, he befriends one of the other clients, and begins telling her how extraordinarily intelligent and talented the facilitator is. At the next session, he and the facilitator disagree. After the session, he tells his fellow group member that the facilitator is utterly incompetent and that they should sue for malpractice. This is an example of A. displacement B. fixation C. reaction formation D. regression E. splitting - Ans. E? ============================================================== EPIDEMIOLOGY Glossary Accuracy: the extent to which a measurement or study result correctly represents the characteristic or relationship that is being assessed. Acquired immunodeficiency syndrome (AIDS): a disease characterized by a marked reduction in CD4+ T lymphocytes and associated defects in immune response caused by the human immunodeficiency virus (HIV). Acute: a disease of short duration. Acute myelogenous leukemia (AML): a heterogeneous group of disorders, also known as acute nonlymphocytic leukemia, each of which involves the uncontrolled proliferation of primitive blood-forming cells. Adjustment: a procedure for overall comparison of two or more populations in which background differences in the distribution of covariables are removed. (See also Standardization. ) Age adjustment: a procedure used to calculate summary rates for different populations in which underlying differences in the age distributions are removed. (See also Age standardization. ) Age-specific rate: a rate (usually incidence or mortality) for a particular age group. Age standardization (direct): a procedure for obtaining a weighted average of age-specific rates in which the weights are selected on the basis of a standard age distribution (eg, the population of the United States in 1940). Allele: an alternate form of a gene or a genetic locus that differs from other forms in its specific sequence of nucleotides; certain alleles may affect the structure and function of the corresponding protein coded for by that gene, in turn affecting the susceptibility to a particular www.brain101.info 23 condition. Alpha error: see Type I error. Alzheimer's disease: the most common form of dementia in many populations, first described in 1907 by Alois Alzheimer; affected individuals have characteristic abnormalities in their brains, including neurofibrillary tangles and plaques with a protein fragment, -amyloid, at their core. Analytic epidemiology: activities related to the identification of possible determinants of disease occurrence. Analytic study: a research investigation designed to test a hypothesis, often used in reference to a study of an exposure-disease association. Antibody: a protein, often produced in response to exposure to an antigen, that binds to the antigen and thereby stimulates its inactivation by the immune system. Antigen: a protein, usually foreign in origin, that is capable of generating an immune response in a host animal. Antigenic drift: mutation of a pathogen (eg, influenza A), such that the surface antigens differ from those of previously existing strains. Apgar score: a system of evaluating the health status of a newborn using five indicators, each assigned a maximum of two points; the score is named after its originator, Dr. Virginia Apgar. Arithmetic mean: see Mean. Arteriosclerosis: hardening of the arteries. Association: the extent to which the occurrence of two or more characteristics is linked either through a causal or noncausal relationship. Asymptomatic persons: individuals who have a particular disease but do not manifest abnormalities of function, appearance, or sensation typically associated with that disease. Atrophy: abnormal wasting of tissues, organs, or the entire body. Attack rate: the proportion of persons within a population who develop a particular outcome within a specified period of time. Attributable risk percent: the percentage of the overall risk of a disease outcome within exposed persons, related to the exposure of interest. Autoimmune disorder: a disease state in which affected individuals produce antibodies against their own cells or tissues. Benign: a mild illness; when applied to an abnormal growth of cells (viz, neoplasm), it connotes a slowly progressing defect that is not invading adjacent tissues (in contrast to the rapid growth and invasive behavior of a malignant neoplasm). Beta error: see Type II error. Bias: a nonrandom error in a study that leads to a distorted result. Biological marker: a measurable characteristic that helps to classify either level of exposure to a risk factor or susceptibility to (or presence of) a disease. Birth cohort effect: an unusual age-specific rate (either incidence or mortality) within cross- sectional data that reflects the shared experience of persons born in specific years (birth cohort). Blinding: assignment of treatment to individual subjects in a way such that subjects only (single blinding ) or both subjects and treating physicians (double blinding ) do not know the actual treatment allocation. Borrelia burgdorferi: a spirochete that is borne by a particular deer tick and when transmitted to humans can cause Lyme disease (a systemic illness characterized by a skin rash, joint pains, and, in advanced cases, cardiac and neurologic manifestations). Bronchoscopic examination: the insertion of an instrument (viz, bronchoscope) to help www.brain101.info 24 visualize the trachea and bronchi and to facilitate the collection of specimens from these tissues. Cancer: a heterogeneous group of diseases characterized by the abnormal, uncontrolled growth of cells, which are capable of crossing normal anatomic boundaries to invade other tissues and even spread to remote anatomic sites. Candidate region: a physical location on a chromosome believed to contain a potential disease susceptibility gene and identified by a genomewide scan and subsequent linkage analysis. Case: a person who has a disease of interest. (See also Incident case and Prevalent case. ) Case-control study: an observational study in which subjects are sampled based on the presence (cases) or absence (controls) of the disease of interest. Information is collected about earlier exposure to risk factors of interest. Case fatality: the proportion of persons with a particular disease who die from that disease within a specified period of time. Causality: the extent to which the occurrence of a risk factor is responsible for the subsequent occurrence of a disease outcome. Cerebral palsy: a disorder manifested by speech disturbances and lack of muscular coordination that arises from damage to the brain of a newborn before, during, or shortly after birth. Cerebrovascular accident: a deficit in the delivery of oxygenated blood to the brain that may occur because of a blood clot or a hemorrhage; a synonym for a stroke. Chance node: an element in a decision analysis that represents a point at which specified outcomes are determined on the basis of probability. Cholesterol: a steroid that is abundant in animal tissues and is necessary for normal function; elevated levels of total cholesterol circulating in the blood of a host are associated with increased risks of cardiovascular disease. Chronic: a disease of long duration. Chronic obstructive pulmonary disease (COPD): an abnormal and long-standing reduction in airflow in and out of the lungs, typically caused either by chronic bronchitis or emphysema. Clinical scenario: one of two or more alternative paths of management available in a decision analysis. Clinical trial: an experimental study that is designed to compare the therapeutic benefits of two or more treatments. Cluster: a group of cases of a disease closely linked in time, place of occurrence, or both. Cochrane Collaboration: an international organization dedicated to promoting well-informed health care decisions by preparing, maintaining, and ensuring accessibility to current, rigorous, systematic reviews of the benefits and risks of health care interventions. The organization is named in memory of Archie Cochrane, a physician epidemiologist who advocated using the best available evidence to guide health care decisions. Coefficient of determination: the square of the correlation coefficient; it represents the proportion of total variability in an outcome that can be explained by the predictors in a regression model. Cohort: a group of persons that shares a common attribute, such as birth in a particular year or residence in a particular town, and is followed over time. Cohort study: an observational study in which subjects are sampled based on the presence (exposed) or absence (unexposed) of a risk factor of interest. These subjects are followed over time for the development of a disease outcome of interest. (See also Prospective cohort study and Retrospective cohort study. ) Common-source exposure: contact with a risk factor that originates in the shared environment www.brain101.info 25 of multiple persons. Concordant results: the same outcome status for two or more individuals, as in a pair-matched case-control study in which both the case and the control are exposed (or unexposed). Confidence interval: a range of values for a measure that is believed to contain the true value within a specified level (eg, 95%) of certainty. Confounder: a variable that distorts the apparent relationship between an exposure and a disease of interest. Confounding: a systematic error in a study that arises from mixing of the effect of the exposure of interest with other associated correlates of the disease outcome. Control: in a case-control study, a subject without the disease of interest. (See also Adjustment. ) Control group: a population of comparison subjects in an analytic investigation. Coronary artery disease: complete or partial blockage of the blood vessels that bring oxygenated blood to the heart muscle (myocardium), usually arising from atherosclerosis; if the reduction in blood flow is severe, a myocardial infarction may result. Correlation coefficient: a statistical measure of the relatedness of two variables; it can range from -1 (perfectly related inversely to each other) to +1 (perfectly related in the same direction to each other). When the variables are unrelated to each other, the correlation coefficient has a value of zero. Correlation study: a hypothesis-generating investigation in which the values of two or more summary characteristics are associated across different population groups. Cosegregation: the tendency of alleles on the same chromosome to be inherited together. Cross-sectional study: an analytic investigation in which subjects are sampled at a fixed point or period of time, and the associations between the concurrent presence or absence of risk factors and diseases are then investigated. Crude mortality rate: the rapidity with which persons within a given population die from a particular disease, without adjustment for the underlying age distribution of the population. Cumulative incidence: the risk of developing a particular disease within a specified period of time. Cutoff point: a value on an ordinal or a continuous scale of measurement used to distinguish categories. For example, values above this threshold may be classified as "abnormal" and values below this point may be classified as "normal." Death rate: see Mortality rate. Decision analysis: a formal probabilistic process for making clinical decisions that incorporates information on medical options, anticipated likelihoods of various outcomes, and the uncertainty associated with clinical information. Decision diagram: a flow chart used in decision analysis that identifies the clinical management choices, probabilities of events, and likelihoods of outcomes. Decision node: an element of a decision tree that represents a choice between two or more competing alternative management approaches. Decision tree: see Decision diagram. Dementia: a condition characterized by impaired short- and long-term memory, along with disturbances of other cognitive functions, such as speech and the perception of spatial relationships. Dependent variable: see Outcome variable. Descriptive epidemiology: activities related to characterizing patterns of disease occurrence. www.brain101.info 26 Diabetes mellitus: a disorder of carbohydrate regulation caused by either a markedly reduced or absent production of insulin by the pancreas (Type I ) or a decreased sensitivity to the effects of insulin in the peripheral tissues (Type II ). Differential misclassification: incorrect categorization of the status of subjects with regard to one variable (eg, exposure) that is influenced by other characteristics of interest (eg, disease status). Discordant results: different outcome status for two or more individuals, as in a pair-matched case-control study, when one subject in a pair is exposed and the other individual is unexposed to the risk factor of interest. Disease outbreak: a sudden, unexpected increase in the occurrence of a disease within a relatively limited geographic area. Diuresis: an abnormally elevated volume of urine production. Dizygotic twin: fraternal twins resulting from the fertilization of two separate ova by two separate spermatozoa; the members of this pair are no more similar genetically than are two nontwin siblings. Dose-response relationship: an exposure-disease association in which the risk of developing a disease varies with respect to the intensity or duration of exposure. Eclampsia: the occurrence of one or more seizures that cannot be attributed to an underlying neurologic condition (such as epilepsy or a cerebral hemorrhage) in a patient with preeclampsia. Ecologic fallacy: an association between summary characteristics across populations without actual linkage of the characteristics within individual persons. Ecologic study: see Correlation study. enzyme-linked immunosorbent assay (ELISA); it can be used to test for antibodies to an infectious agent. Emerging infectious disease: an infection that has newly appeared within a population or has existed but is rapidly increasing in incidence or geographic range. Empiric treatment: in the context of infectious illness, the initiation of an antibiotic treatment against a spectrum of suspected potential pathogens, in the absence of a documented specific pathogen(s). Endemic rate: the usual rate of occurrence of particular events within a population. Eosinophilia-myalgia syndrome: a condition characterized by muscle pains and, in some patients, joint pains, skin thickening, hair loss, or intestinal disease, accompanied by an abnormally elevated level of eosinophil cells in the blood. Epidemic: a dramatic increase above the usual or expected rate of occurrence of particular events within a population. Epidemiology: the study of the distribution and determinants of disease within human populations. Escherichia coli O157:H7: a strain of bacteria that is a cause of the hemolytic uremic syndrome, which occurs in humans who consume food products contaminated with this pathogen. Etiology: the cause(s) of a disease or the study of disease causation. Evidence-based medicine: the integration of current best evidence from research with clinical expertise, pathophysiologic knowledge, and patient preferences, used to make health care decisions. Excess risk: the extra risk of the occurrence of a particular disease among persons exposed to a risk factor of interest. (See also Risk difference. ) Exclusions: persons who are eliminated from an analytic study because they do not satisfy the www.brain101.info 27 eligibility (inclusion) criteria. Expected utility: a numerical value that represents the average result if the decision maker follows a particular path in a decision analysis. Exposure: contact with or possession of a characteristic that is suspected to influence the risk of developing a particular disease. External validity: the extent to which the conclusions of a study can be correctly applied to persons beyond those who were investigated. (See also Generalize. ) False negative: a test result that is normal (negative) despite the true presence of the disease of interest or a study result that incorrectly fails to identify a true effect. (See also Type II error. ) False positive: a test result that is abnormal (positive) despite the true absence of the disease of interest or a study result that incorrectly suggests an effect, when, in truth, the purported effect does not exist. (See also Type I error. ) Familial aggregation: the extent to which the occurrence of a particular disease tends to cluster within families. Fixed effects model: a statistical approach to combining information from multiple sources in which it is assumed that the investigated relationship is constant across sources and any differences in individual results are attributable entirely to random variation. Follow-up study: see Cohort study. Food-borne disease: an illness that is caused by the ingestion of food or food products, often arising from contamination of the food with microbes or other toxic materials. Framingham Heart Study: a landmark prospective cohort study of risk factors for cardiovascular disease initiated in 1950 among residents of Framingham, Massachusetts. Generalize: the ability to extrapolate study results from the study subjects to other persons who were not investigated. Genetic epidemiology: the use of epidemiologic techniques to study hereditary determinants of disease in human populations. Genetic (linkage) map: a type of map of the genome in which markers are identified on the chromosomes and the relative distances between these markers are estimated by the frequency with which the markers are inherited together. Genome: the full complement of genes on all chromosomes. Genomewide scan: an approach to localizing candidate regions for genes contributing to the susceptibility for a specific disease by analyzing the extent to which the disease occurs in members of affected families in association with markers at known locations throughout the genome. Genotype: the genetic constitution of an individual; may be used in reference to the particular allele(s) present at one or more gene loci. Glucosuria: an abnormally elevated level of glucose in the urine, as my occur in diabetes mellitus. Granulocyte: a mature granular white blood cell, which includes neutrophils as well as other types of cells. Granulocytopenia: a condition marked by an abnormally low number of granulocytes in the blood, and which may predispose the host to infection. Hantavirus: a virus named for a river in South Korea, where human infection was first recognized; this pathogen is capable of causing a hemorrhagic fever and a separate pulmonary syndrome in infected human hosts. Hematologic: of or relating to the blood or blood-forming tissues. www.brain101.info 28 Heterogeneity: the statistical property of variation in an investigated relationship across individual studies or across subgroups within a particular study. Historical cohort study: see Retrospective cohort study. Historical controls: subjects in a clinical study who were previously treated with the standard therapy before the new experimental treatment was introduced. HIV: see Human immunodeficiency virus. Homogeneity: the statistical property of lack of variation of an investigated relationship across individual studies or across individual subgroups within a particular study. Human immunodeficiency virus: the cause of the acquired immunodeficiency syndrome (AIDS) and other HIV-related disorders. Hyperglycemia: an abnormally high level of glucose in the blood, as may occur in untreated patients with diabetes mellitus. Hypertension: an abnormal elevation in blood pressure. Hypoglycemia: an abnormally low level of glucose in the blood, as may result from an overly aggressive administration of insulin in patients with diabetes mellitus. Hypothesis-generating study: an exploratory investigation designed to formulate questions that are evaluated in subsequent analytic studies. Hypothesis-testing study: an analytic investigation in which one or more specific refutable suppositions are evaluated. Hypoxia: an abnormally low level of oxygen in the arterial blood. Immunity: a state in which a host is not susceptible to a particular infection or disease. Inbreeding study: a study in which the degree of selective breeding among members of a particular group is assessed with respect to risk of developing a particular disease. Incidence density: see Incidence rate. Incidence rate: the rapidity with which new cases of a particular disease arise within a given population. Incident case: a person who is newly diagnosed with a disease of interest. Incubation period: the time interval between contact with a risk factor (often an infectious agent) and the first clinical evidence of the resulting illness. Independent variable: a factor that is suspected to influence the outcome of an analytic study. Index case: in a disease outbreak, the first affected individual to be identified; in genetics, see Proband. Information (or observation) bias: a systematic error in a study that arises from the manner in which data are collected from participants. Informed consent: the process of providing a patient with information about the risks and benefits of a proposed treatment plan and then securing the patient's (or if the patient is a child, the guardian's) agreement to undergo the planned intervention recognizing the risks and benefits. Insulin: a peptide hormone produced in the pancreas and secreted into the blood, which delivers it to target organs to help regulate glucose utilization, protein synthesis, and formation and storage of lipids. Intention to treat: analysis of the results of a clinical trial based on initial treatment assignment regardless of whether the subjects completed the full course of treatment. Internal validity: the extent to which the conclusions of a study are correct for the subjects under investigation. Ketosis: a condition characterized by the enhanced production of ketone bodies, as may occur in the metabolic abnormalities associated with diabetes mellitus. www.brain101.info 29 Latent period: time between exposure to a risk factor and subsequent development of clinical manifestations of a particular disease. Lead-time bias: apparent increase in the length of survival of patients with a disease as a result of earlier detection of the disease through the use of a screening procedure. Length-biased sampling: preferential detection of less aggressive forms of a disease through the use of a screening procedure. Life expectancy: the expected, or average, duration of life for persons in a particular population, under the assumption that current age-specific mortality patterns continue to apply. Likelihood: the probability of the occurrence of a specified event. Likelihood ratio: the probability of a particular test result for a person with the disease of interest divided by the probability of that test result for a person without the disease of interest. Likelihood ratio for a negative test result: the probability of a negative test result for a person with the disease of interest divided by the probability of a negative test result for a person without the disease of interest. Likelihood ratio for a positive test result: the probability of a positive test result for a person with the disease of interest divided by the probability of a positive test result for a person without the disease of interest. Linkage: the proximity of multiple genes or genetic markers on the same chromosome, which is related to the probability that a certain combination of alleles at these sites will be inherited as a linkage group or haplotype. Linkage analysis: a statistical technique used to identify candidate regions for genes based on the examination of the closeness of association between the inheritance within affected families of the condition of interest and markers at known locations throughout the genome. Linkage disequilibrium: an excess or deficiency of certain combinations of alleles from genes or markers that are located on the same chromosome; alleles at tightly linked sites often are inherited together, and, therefore, linkage disequilibrium may help to identify the location of a particular susceptibility gene that is inherited as part of a linkage group with genes or markers at known locations within the genome. Longitudinal study: see Cohort study. Malignancy: the property of being malignant, often used interchangeably with the term cancer. Malignant: a severe disease that is resistant to treatment (eg, severe hypertension); the term often is used in relation to the behavior of cancers. Marker: in genetics, an identifiable physical location on a chromosome or deoxyribonucleic acid (DNA) segment useful in mapping genes and in performing linkage analysis. Matching: a procedure for sampling comparison subjects based on whether key attributes (ie, matching factors ) are similar to those of subjects in the index group. Mean: the arithmetic average of a distribution of values; calculated as the sum of the individual values divided by the number of observations. Meconium: the first intestinal discharges of a newborn; if passed prior to delivery, it may serve as a sign of fetal distress, and if aspirated by the newborn, may give rise to acute pulmonary distress. Median: a measure of central tendency of a distribution; calculated as the mid-point of the distribution when individual values are ordered from the smallest to the largest. Median survival time: the duration of time from diagnosis to death that is exceeded by exactly 50% of subjects with a particular disease. Medical outcome: See Outcome. www.brain101.info 30 Meta-analysis: a statistical combination or integration of the results of several independent research studies that are considered to be combinable. Metabolic acidosis: an abnormally high level of acid and low level of bicarbonate in the blood and other tissues resulting either from an accumulation of acids from metabolic processes (as in diabetes mellitus) or from an abnormally high loss of bases from the body (as in diarrhea or renal disease). Misclassification bias: incorrect characterization of the status of subjects with regard to a study variable, leading to a distorted conclusion. (See also Information bias. ) Mode: a measure of central tendency of a distribution; it is the value that occurs most frequently within the distribution. Monozygotic twin: genetically identical individuals arising from the division of a single fertilized ovum. Morbidity: a state of illness produced by a disease. Mortality: death, usually in reference to death caused by a particular disease (viz, cause-specific mortality ). Mortality rate: the rapidity with which persons within a given population die from a particular disease. Mycobacterium: a genus of bacteria, a member of which is M tuberculosis, also known as tubercle bacillus, the pathogen responsible for tuberculosis in humans. Myocardial infarction: a sudden diminution in the delivery of oxygenated blood to the heart muscle (viz, myocardium ), most commonly caused by partial or complete blockage of one or more of the coronary arteries. Natural history: the progression of a disease through successive stages, often used to describe the course of an illness for which no effective treatment is available. Negative predictive value: the probability that a person with a negative (normal) test result actually does not have the disease of interest. Neoplasm: a new growth that arises from the abnormal proliferation of cells; the proliferation may be benign or malignant (viz, cancer ). Nephropathy: a disorder of the kidney; among diabetics, the disorder arises because of damage to the small blood vessels of the kidney, which can lead to failure of the kidneys in an advanced stage. Neutropenia: the presence of an abnormally low level of neutrophils in the blood, placing the host at increased susceptibility to infection. Neutrophil: a mature white blood cell in the granulocyte series necessary for normal host defense responses. Nondifferential misclassification: incorrect categorization of the status of subjects with regard to one variable (eg, exposure) that is unrelated to another characteristic of interest (eg, disease status). Nosocomial infection: an illness caused by exposure to a pathogen during hospitalization of the host. Notifiable disease: a disease for which regular, frequent, and timely information on individual cases is considered necessary for the prevention and control of the disease. Null value: the point on the scale of a measure of association that corresponds to no association (eg, 1 for the risk ratio and the odds ratio and 0 for the risk difference and the attributable risk percent). Observation bias: see Information bias. www.brain101.info 31 Observational study: a nonexperimental analytic study in which the investigator monitors, but does not influence, the exposure status of individual subjects and their subsequent disease status. Odds: the probability that a particular event will occur divided by the probability that the event will not occur. Odds ratio: the odds of a particular exposure among persons with a specific disease divided by the corresponding odds of exposure among persons without the disease of interest. Opportunistic infection: an illness caused by a microorganism that is capable of causing disease only in a host whose resistance is lowered below normal levels. Outbreak: see Disease outbreak. Outcome: clinical events that result from patient management decisions (eg, morbidity, complications, quality of life, or mortality). Outcome variable: in an analytic study, the response of interest (eg, development of disease). Pandemic: an elevated occurrence of a disease across a wide geographic area, affecting a substantial proportion of the population. Pathogen: an agent responsible for the development of a particular disease. Pathophysiology: derangement of function associated with a disease process. Pedigree: the family members of a proband, identified with respect to their biological relationship to the proband and whether they are known to have the disease of interest. Penetrance: the proportion of individuals with a particular genotype that exhibits the same phenotype under similar environmental conditions. Perinatal asphyxia: an abnormally reduced level of oxygenation of a fetus during labor and delivery, or shortly thereafter. Person-time: a unit of measurement used in the estimation of rates that reflects the amount of time observed for persons at risk of a particular event. Person-to-person spread: propagation of a disease within a population by transfer from an affected person to susceptible persons. Person-years: a common unit for measuring person-time; one person-year corresponds to one person being followed for one year, or, alternatively, two persons each followed for one-half year, and so forth. Person-years of life lost: a measure of total life expectancy lost within a particular population because of premature death. Phenotype: a category or group to which an individual may be assigned on the basis of one or more characteristics observable clinically or by laboratory assessment that reflect genetic variation or gene-environment interactions. Placebo: an inert substance. Placebo effect: occurs when persons affected with a specific illness demonstrate clinical improvement when treated with an inert substance. Polymerase chain reaction: a laboratory technique for rapidly synthesizing large quantities of a particular portion of genetic material. Population at risk: persons who are susceptible to a particular disease but who are not yet affected. Population-based study: an analytic study in which subjects are sampled from the general population. Positive predictive value: the probability that a person with a positive (abnormal) test result actually has the disease of interest. Posttest odds of disease: the estimated probability, after the administration of a diagnostic test, www.brain101.info 32 that a patient has the disease of interest divided by the probability that the patient does not have the disease of interest. Posttest probability of disease: the estimated likelihood, after the administration of a diagnostic test, that a patient has the disease of interest. Power: see Statistical power. Precision: the extent to which a measurement is narrowly characterized. Statistical precision is inversely related to the variance of the measurement. Predictor variable: see Independent variable. Preeclampsia: the abnormal occurrence of hypertension accompanied by either an abnormal collection of fluid in body tissues or abnormally increased levels of protein in the urine, or both, due to pregnancy. Premature death: a death that occurs earlier than would be expected in the absence of a particular disease. Pretest odds of disease: the estimated probability, prior to the administration of a diagnostic test, that a patient has the disease of interest divided by the probability that the patient does not have the disease of interest. Pretest probability of disease: the estimated likelihood, prior to the administration of a diagnostic test, that a patient has the disease of interest. Prevalence: the proportion of persons in a given population that has a particular disease at a point or interval of time. Prevalent case: a person who has a disease of interest that was diagnosed in the past. Proband: the first affected individual who brings his or her family to the attention of a researcher or clinician for the purposes of medical care or investigation. Prognosis: the predicted rate of progression of a disease process and its likely outcome(s). Prognostic factor: an attribute anticipated to be related to the progression and outcome of a disease process. Proportion: one quantity divided by another quantity in which the population in the numerator is a subset of the population in the denominator. The possible values of a proportion range from zero to one. Prospective cohort study: a cohort study in which exposure status and subsequent occurrence of disease both occur after the onset of the investigation. Publication bias: a distortion in conclusions derived from published studies because of the selective factors associated with the likelihood of publication, including whether the findings were positive and statistically significant, and the potential proprietary interests of sponsors. Random effects model: a statistical approach to combining information from multiple sources in which it is assumed that the investigated relationship varies across individual sources, in addition to the influences of random variation in estimates. Randomization: procedure for assigning treatments to patients by chance. Rate: the rapidity with which health events such as new diagnoses or deaths occur. (See also Incidence rate and Mortality rate. ) Rate ratio: the rate of occurrence of a specified health event among persons exposed to a particular risk factor divided by the corresponding rate among unexposed persons. Ratio: one quantity divided by another quantity, in which the population in the numerator is not a part of the population in the denominator. The possible values of a ratio range from zero to positive infinity. Recurrence risk: in genetic epidemiology, the risk of developing a particular disease www.brain101.info 33 experienced by relatives of a subject with that disease. Relapse: the return of the manifestations of a disease after a period of diminished manifestations. Relative risk: see Risk ratio. Reliability: the extent to which multiple measurements of a characteristic are in agreement. Remission: elimination or reduction in the number or severity of the manifestations of a disease, which may be transient or permanent. Response variable: see Outcome variable. Retinopathy: a disorder of the retina of the eye; among diabetics the disorder arises from damage to the small blood vessels of the retina and can lead to blindness. Retrospective cohort study: a cohort study in which exposure status and subsequent development of disease both occur prior to the onset of the investigation. Risk: the probability that an event (eg, development of disease) will occur within a specific period of time. Risk difference: the risk of the occurrence of a particular disease among persons exposed to a given risk factor minus the corresponding risk among unexposed persons. Risk factor: an attribute or agent suspected to be related to the occurrence of a particular disease. Risk ratio: the likelihood of the occurrence of a particular disease among persons exposed to a given risk factor divided by the corresponding likelihood among unexposed persons. Sample: a subset of a target population that is chosen for investigation. Screening: the use of tests to detect the presence of a particular disease among asymptomatic persons prior to the time that the disease would be recognized through routine clinical methods. The Surveillance, Epidemiology and End Results (SEER) Program of the National Cancer Institute; it consists of 11 population-based cancer registries in various locations within the United States. Segregation analysis: a complex statistical technique used to assess whether a particular disease has, at least in part, a genetic origin, and, if so, the most likely mode of inheritance. Selection bias: a systematic error in a study that arises from the manner in which subjects are sampled. Sensitivity: the probability that a person who actually has the disease of interest will have a positive (abnormal) test result. Sensitivity analysis: (1) in systematic reviews, including meta-analyses, the evaluation of the pattern of results across subgroups of studies to characterize possible sources of heterogeneity and their respective influences on the overall summary effect; (2) in decision analysis, use of different values for an uncertain likelihood to determine whether the preferred course of action remains unchanged. Sentinel case(s): the initial person(s) affected by a particular illness during an outbreak. Seroconversion: change in a person's status from not having evidence of infection (such as antibodies) in the serum to having such evidence. Seronegative: absence of evidence of infection in a person's serum (synonym: antibody negative). Seropositive: presence of evidence of infection in a person's serum (synonym: antibody positive). Sib-pair analysis: statistical analysis for many genetic linkage studies attempting to locate susceptibility genes for a particular disease in which the fundamental unit of analysis is a pair of www.brain101.info 34 siblings. Specificity: the probability that a person who actually does not have the disease of interest will have a negative (normal) test result. Squamous cell carcinoma: a malignant neoplasm (cancer) arising from stratified squamous epithelium, but that may also occur in sites in which glandular or columnar epithelium normally occur. Standardization: an analytic procedure for obtaining a summary measure for a population by applying standard weights to the measures within subgroups of the population. Statistical power: the ability of a study to detect a true effect of a specified magnitude. The statistical power corresponds to 1 - Type II error. Statistical significance: the likelihood that a difference as large as or larger than that observed between study groups could have occurred by chance alone in a sample of the size investigated. Usually, the level of statistical significance is stated as a P -value (eg, P < 0.05). Stroke: a sudden derangement in function, as in sunstroke or heat stroke; often used in relation to a sudden neurologic deficit that occurs because of insufficient delivery of oxygenated blood to the brain, as may occur following a blood clot or hemorrhage. Subacute: a rate of progression of a condition that is intermediate between acute and chronic. Surveillance: ongoing observation of a population for rapid and accurate detection of changes in the occurrence of particular diseases. Survival: the likelihood of remaining alive for a specified period of time after the diagnosis of a particular disease. Systematic error: see Bias. Systematic review: a synthesis of medical evidence on a topic, in which the synthesis has been prepared using strategies to minimize errors. Terminal node: in a decision tree, an element that represents the outcome for a particular clinical scenario. T lymphocyte: a white blood cell that is responsible for cell-mediated immunity in the host. Transmission: the process by which a pathogen passes from one source of infection to a new host. True negative: a test result that is normal (negative) when the disease of interest is actually absent. True positive: a test result that is abnormal (positive) when the disease of interest is actually present. Tuberculosis: an infectious illness caused by Mycobacterium tuberculosis, characterized by a brief initial illness; in a minority of cases, a chronic active illness, primarily affecting the lungs, will occur months to years following infection. Tumor: a swelling that may occur from an inflammatory process or a benign or malignant neoplasm. Twin study: a study of genetic susceptibility in which concordance for occurrence of a particular disease is compared between dizygotic (fraternal) twins and monozygotic (identical) twins, or between twins reared together versus apart. Type I error: rejection of the null hypothesis when it is actually correct. Type II error: failure to reject the null hypothesis when it is actually incorrect. Underlying cause of death: (1) the disease or injury that initiated the train of morbid events leading directly to death, or (2) the circumstances of the accident or violence that resulted in fatal injury. www.brain101.info 35 Utility: in decision analysis, a patient's preference for one outcome over another, usually graded on a scale of zero, representing death, to one, representing perfect health. Validity: the extent to which a measurement or a study result correctly represents the characteristics or relationship of interest. Variability: the property of having a spread of values, which may arise from random sources (viz, the operation of chance) or from systematic influences (viz, bias). Viremia: the presence of virus particles in the blood of a host. Vital statistics: information concerning patterns of registered life events, such as births, marriages, divorces, and deaths. Weighted average: a summary measure in which some of the component data values are assigned greater influence than others. For example, precision-based weighting is the calculation of a summary measure in which the relative influence of individual results is based on statistical confidence in the respective results. Withdrawals: subjects who are initially included in a study but later voluntarily or involuntarily terminate participation. Years of potential life lost (YPLL): a measure of total life lost to a particular age (eg, 75 years) within a population because of premature deaths. Questions 1-3: For each numbered situation below, select the most appropriate term from the following lettered options. Each option can be used once, more than once, or not at all. A. Birth cohort effect B. Ecologic fallacy C. Latent period D. Endemic occurrence E. Epidemic occurrence 1. Populations in which infection from Helicobacter pylori is common have a persistent, steady elevation in gastric cancer when compared with other communities in which this infection is uncommon. 2. H pylori infection increases the risk of gastric cancer, but the time from initial infection to cancer occurrence is more than a decade. 3. A correlation is shown between national consumption of coffee and gastric cancer mortality, but individual coffee drinkers do not have an elevated risk of developing gastric cancer. Questions 4-7: For each measure discussed in the numbered statements below, select the most appropriate value from the following lettered options. Each option can be used once, more than once, or not at all. A. 0.30 B. 0.50 C. 0.90 D. 6.5 E. 11.0 F. Cannot be determined from the information provided 4. In a correlational study, the average annual per capita consumption of alcohol (in gallons) is used to predict annual mortality rates (per 100,000 persons) from cirrhosis of the liver across states in the United States. The resulting regression equation is www.brain101.info 36 The correlation coefficient is 0.55 and the coefficient of determination is 0.30. What is the predicted increase in the liver cirrhosis annual mortality rate (per 100,000 persons) for 1 gallon per capita annual alcohol consumption? 5. Given the equation in question 4, what is the predicted annual mortality rate (per 100,000 persons) for cirrhosis of the liver in the absence of alcohol consumption? 6. Given the equation in question 4, what is the predicted annual mortality rate (per 100,000 persons) for cirrhosis of the liver in a state with an average annual per capita alcohol consumption of 5 gallons? 7. Given the equation in question 4, what proportion of the variability in average annual mortality rates (per 100,000 persons) for cirrhosis of the liver can be accounted for by knowing the corresponding per capita annual consumption of alcohol? Questions 8-11: For each migration discussed in the numbered statements below, select the lettered option that describes the most likely effect on the incidence rate of disease among offspring. Each option can be used once, more than once, or not at all. A. Greater B. Smaller C. About the same D. Cannot be determined from the information provided 8. When compared with the corresponding rate for nonmigrants in a high-risk country, the incidence rate for an environmentally determined disease among offspring of migrants to a low- risk country is 9. When compared with the corresponding rate for nonmigrants in a low-risk country, the incidence rate for an environmentally determined disease among offspring of migrants to a high- risk country is 10. When compared to the corresponding rate for nonmigrants in a high-risk country, the incidence rate for a genetically determined disease among offspring of migrants to a low-risk country is 11. When compared to the corresponding rate for nonmigrants in a low-risk country, the incidence rate for a genetically determined disease among offspring in a high-risk country is 1. D 2. C 3. B 4. C 5. D 6. E 7. A 8. B 9. A 10. C 11. C For each question, select the single best answer. Questions 1-5: www.brain101.info 37 A cohort study is conducted to evaluate the relationship between dietary fat intake and the development of prostate cancer in men. In the study, 100 men with a high fat diet are compared with 100 men who are on a low fat diet. Both groups start at age 65 and are followed for 10 years. During the follow-up period, 10 men in the high fat intake group are diagnosed with prostate cancer and 5 men in the low fat intake group develop prostate cancer. 1. What is the risk of developing prostate cancer in the high fat group? A. 0.05 B. 0.10 C. 0.15 D. 0.20 E. 0.25 2. What is the risk of developing prostate cancer in the low fat group? A. 0.05 B. 0.10 C. 0.15 D. 0.20 E. 0.25 3. What is the risk ratio (high fat consumers compared to low fat consumers) for the occurrence of prostate cancer? A. 0.05 B. 0.75 C. 1.0 D. 1.5 E. 2.0 4. The point estimate for the risk ratio in question 3 suggests that the risk of prostate cancer associated with consumption of a high fat diet is A. Decreased B. Increased C. Not affected D. Cannot be determined from the information provided 5. The 95% confidence interval is 0.95 to 3.5. For statistical significance at an alpha level of 0.05, the correct interpretation of these results is that A. A statistically significant association exists between high dietary fat intake and an increased risk for prostate cancer. B. A statistically significant association exists between high dietary fat intake and a decreased risk for prostate cancer. C. It can be concluded with 95% confidence that high dietary fat intake protects against prostate cancer. D. It can be concluded with 95% confidence that high dietary fat intake increases the risk of prostate cancer. E. The risk of prostate cancer is not statistically significantly different between men with high fat intake and men with low fat intake. Questions 6-10: A cohort study is conducted to evaluate the relationship between serum cholesterol level and the occurrence of myocardial infarction in women. In the study, 500 women with high serum cholesterol levels and 500 women without high serum cholesterol levels are followed over a 10- www.brain101.info 38 year period. During the study, 40 of the women with high serum cholesterol levels and 15 of the women with normal serum cholesterol levels develop a newly diagnosed myocardial infarction. 6. The incidence rate (per 10,000 person-years) for a myocardial infarction among women with high serum cholesterol is A. 30 B. 50 C. 60 D. 80 E. 100 7. The incidence rate (per 10,000 person-years) for a myocardial infarction for women with normal serum cholesterol is A. 30 B. 50 C. 60 D. 80 E. 100 8. The (incidence) rate ratio for myocardial infarction is A. 0.37 B. 1.33 C. 2.67 D. 3.15 E. 3.75 9. The risk difference is A. 0.002 B. 0.005 C. 0.006 D. 0.01 E. 0.05 The attributable risk percent is A. 25.5% B. 35.0% C. 47.5% D. 55.5% E. 62.5% 1. B 2. A 3. E 4. B 5. E 6. D 7. A 8. C 9. E 10. E www.brain101.info 39 Questions 1-4: For each numbered situation below, select the best descriptor from the following lettered options. Each option can be used once, more than once, or not at all. A. Ecologic fallacy B. Confounding C. Random error D. Misclassification E. Cohort effect F. Selection bias 1. In a case-control study of environmental exposure to tobacco smoke and risk of sudden infant death syndrome (SIDS) in children, low socioeconomic status is associated with exposure to environmental tobacco smoke and is a risk factor for SIDS. 2. In the study cited in question 1, cases are sampled from death certificates and controls are sampled from registrants in a managed care health insurance plan. 3. In the study cited in question 1, parents of children who have died from SIDS are more likely to overestimate their child's exposure to environmental tobacco smoke than are parents of controls. 4. A correlation analysis across 15 states reveals an association between mortality rates from SIDS and smoking prevalence among young adults, but parents of children who die from SIDS are not more likely than other parents to be smokers. Questions 5-7: For each numbered situation below, select the most appropriate advantage of the case-control design from the following lettered options. Each option can be used once, more than once, or not at all. A. Confounding is unlikely because of randomization B. Efficient for the study of rare diseases C. Efficient for the study of diseases that develop slowly D. The risk among exposed persons can be estimated directly E. The temporal relationship between exposure and disease is clearly defined F. The ability to conduct an assessment quickly allows rapid intervention 5. Assessing risk factors for infection in a community-wide outbreak of cryptosporidiosis. 6. Assessing risk factors for congenital defects of the neural tube. 7. Assessing the role of antioxidants in reducing the risk of atherosclerotic coronary artery disease. Questions 8-10: For each numbered measure below, select the most appropriate calculation from the following lettered options. Each option can be used once, more than once, or not at all. A. 80/150 B. 20/280 C. 20/300 D. (80 ª 280)/(20 ª 70) E. 80/70 F. (20 ª 70)/(80 ª 280) G. (80/100)/(70/350) 8. The odds of exposure among cases in an unmatched case-control study of risk factors for adenocarcinoma of the esophagus, in which chronic heartburn was found in 80 of 150 cases and www.brain101.info 40 20 of 300 controls. 9. The odds of exposure among controls in the study described in question 8. 10. The odds ratio for exposure in the unmatched case-control study described in question 8. Questions 11-13: For each numbered situation below, select the best descriptor from the following lettered options. Each option can be used once, more than once, or not at all. A. Concordant pair B. Discordant pair C. Not applicable D. Cannot be determined from the information provided 11. In a pair-matched case-control study of traumatic head injury as a risk factor for Alzheimer's disease, a patient with Alzheimer's disease had a prior history of head trauma and the corresponding control did not have prior head trauma. 12. In a pair-matched case-control study of cognitive impairment as a risk factor for hip fracture in the elderly, a patient with a hip fracture has cognitive impairment as does the corresponding control. 13. In an unmatched case-control study of infection from Helicobacter pylori as a risk factor for stomach cancer, a patient with stomach cancer has serum antibody to H pylori and a control also is antibody seropositive for H pylori. 1. B 2. F 3. D 4. A 5. F 6. B 7. C 8. E 9. B 10. D 11. B 12. A 13. C  Accuracy vs. Precision - Accuracy and validity are the same. (hitting the target) Precision and reliability are the same. (hitting the target in the same place every time) - if you have both: accuracy/validity AND precision/reliability then you hit the target in the middle (bull's eye) everytime... - use this analogy for analyzing tests in biostats. so accuracy/validity would mean you get the same answer most of the time, not necessarily the right one and precision/reliability means you get the right answer most of the time. www.brain101.info 41 - CORRECTION:see the FA part of BS... they have a picture and explain it well. Correction: accuracy and validity (hitting the target- but all over the target) Precision and reliability (hitting one area of the target all the time- not necessarily the bulls eye) if you have both: accuracy/validity AND precision/reliability then you hit the target in the middle (bull's eye) everytime... - Validity: whether the test truly measures what it purports to measure Reliability: Reproducibility of a test. (getting around the same ans./results every time) Achild begins to copy: A Circle at 3yr A Cross at 4yr A Square at 5yr A triangle at 6yr  Stranger Anexiety:7-11mo Separation Anexiety:12-15mo  aging decrease both REM&stages 3,4,wake early in the morning MDD:short REM latency,long first REM,total REM%INCREASE(>25%)BUT REM decrease toward morning leads to waking early in the morning,decrease stages3,4,normal sleep oneset elderly and depressed people awake more than 3times during the night bipolar and anxious people have difficulty with sleep begining alcohol,benzo and barbiturates decrease REM&delta sleep(stage3,4) Recall:TOTAL REM INCREASE IN MDD!  In what group age is illness percieved as a punishment??????? Ans. 0-5 yrs.  EMANCIPATION: Under 18 years is legally incompetent and considered minor.But the boy is emancipated minor as he is serving military.Other facts that make minors emancipated are-1)living and supporting on their own 2)Marriage.Please note being pregnant or having child does not make child emancipated. Partial emancipation is considered for substance abuse,STD treatment,birth control and prenatal care as these issues have impact on public health.  1]diff bet schizotypal and schizoaffective? 2]diff bet autistic and asperger disorder? Ans. schizotypal disorder:in DSMIV:a personality disorder:with odd behavioral and magical thinking but!schizoaffective disorder:in DSMIV:psychotic disorder that characterized by psychosis+mood symptoms Autistic disorder:problems with communication and social relationship,also MR,repeating www.brain101.info 42 behavioral some of them have unusual abilities in one field like math!asperger the same but no delay in language development,infact it's mild form of autistic disorder  schizophreniform- - symptoms less than 6 months schizophrenia-symptoms more than 6 months schizoid--avoidant,socially isolated,very few friends schizotypal- - magical thinking  in addition: schizoid--"voluntary",not a psychosis(what's that guy's name in "Angel eyes"??) schizotypal--thinks weird but looks weird too!! (Robbin Williams in "Fisher King" will be a good example) schizoaffective--schizophrenia + mood disorder(that will be me..:)  neurotransmitter of REM sleep? Ans. acetylcholine..  and what's the role of serotonin in sleep cycle? Ans. I think it initiates sleep!… yes,serotonin regulates sleep pattern,increase level of serotonin associated with increase in total sleep and delta sleep,do u know the location of REM sleep system? Ans. Pons  Anaclitic Depression: when a baby between the age of 6 and 15 months is separated from its primary care giver, it can suffer from anaclitic depression characterised by depression and episodes of screaming and loud protests  ON HARDY-WEINBERG: That's used to calculate the prevalence of each genotype(AA, Aa, or aa), given the prevalence of each gene(A or a). If A accounts for 70% and a for the rest, AA will account for 49%(0.7x0.7), Aa for 42%(0.3x0.7x2), and aa for 9%(0.3x0.3). Just draw a 2x2 table, and that'll make it very clear. > Basically you have to calculate the positive predictive value (PPV), for which prevalance is required which in this case is 10%. Assuming the population to 100 construct a 2x2 table using the sensitivity and specificity for the test which is this problem is 80% and 90% respectively. --- Disease No Disease Total pos 8 -----------9 ----------17 neg 2 ----------81 ---------83 tot 10 ---------90 ---------100 So the PPV will be true positive divided by total positive ie 8 divided by 17 which is 47%  Emphasized Topics: - STATISTICS Lots of computations on odds ratio, relative risk, Hardy-Weinberg, values for sensitivity and specificity of two studies and you will be asked to compare. May questions also about biases. The study will be described and you have to determine what is the bias of study. Questions on alpha and beta errors. Same thing as before, the study will be described and you'll be asked what error is it. Questions on what is the best thing to say to a patient given a particular situation. For this you have to study ethics. Kaplan is OK www.brain101.info 43 - BEHAVIORAL SCIENCE - ELDERLY What are the cognitive changes? Physiologic changes Alzheimers, Dementia, Delirium etc. Know the common Differential Diagnosis like Picks etc. - PERSONALITY DISORDERS - COMMON PSYCHIATRIC ILLNESS (Depression, Schizophrenia etc.) - DRUG ADDICTION/DEPENDENCY - DEFENSE MECHANISMS - NEUROTRANSMITTERS --------------------------------------------------------------------  MISCELLANEOUS TOPICS  PROGNOSTIC FACTORS IN SCHIZOPHRENIA. Mark each one of the following choices with "g" for good prognisis and "p" for poor prognosis for a patient with Schizophrenia. 1. onset of psychotic symptoms within a month of the first change in behavior. 2. withdrawn autistic behavior 3. flat affect 4. positive family history of schizophrenia. 5. confusion or perplexity at the height of the psychotic episode. 6. good premorbid social and occupational functioning. Ans. 1, 5, 6 –G : 2, 3, 4 -P > 1.17 YR old male visits the physician bcos his parents are worried about his behaviour.he is always in the shower taking hours in the morning.he spends lot of time washing his hands which he says is to get rid of the germs.which of the foll neuro transmitter abnormalities is the cause for the disease a.dopaminergic b.cholinergic c.noradrenergic d.serotonergic 2.A 30 yr old man is brought to the ER by a friend bcos he was turning blue.On exam he is pale,perioral cyanosis,shallow resp of 3-5/min.he is unresponsive .his friend reports he must have taken something to "relax".no track marks are seen.the drug most likely the patient has taken is www.brain101.info 44 a.benzodiazepine b.alcohol c.cocaine d.ketamine e.opiate Ans. 1.d ....diag is OCD ......NT is serotonin 2.e opiate........this is the one that causes respiratory depression  confidence interval The formula is CI= X +- Z( S/under root N) where CI is conf interval +- is plus minus Z= z score which is 2 for 95% confidence and 2.5 for 99% (this will be mentioned in the question) S= Standard deviatin N= sample size  Case 1 A 16-year-old female presents to a family physician to obtain a referral for family therapy. She is estranged from her mother and stepfather, who see the same physician. For many years, this patient responsibly cared for her four younger siblings while their single mother worked. Since her mother's marriage, the family has become involved in a fundamentalist church. The patient moved out when she felt the social and moral restrictions of the family's religion were too burdensome for her. The patient seemed quite mature; she maintained a 3.5 GPA, along with a part-time job. She demonstrated a genuine desire for reconciliation, and the therapy referral was provided. She also requested and obtained a prescription for contraceptives during the visit, with the assurance that her sexual activity would be kept confidential. In follow-up, she reported that the therapist had informed her that if she mentioned anything about being sexually active with her adult partner, he would be obliged to report her to the state. The patient was very concerned about the conflict between this statement and the family physician's prior assurance of confidentiality. Should this patient's confidentiality be broken? While the physician has a moral obligation to obey the law, he must balance this against his responsibility to the patient. In researching the Criminal Code of Washington, the physician learned that sexual intercourse with a minor, at least 16, but under 18, is a class C felony, and a reportable offense, if the offender is at least 90 months older than the victim. This patient's relationship did not actually meet the criteria for mandatory reporting. Had this not been the case however, the physician could be justified in weighing the balance of harms arising from the filing of such a report. There is little justification for informing the family of the young woman's sexual activity. Due to the family's strong fundamentalist beliefs, significant damage would have occurred in the family reconciliation process with this discovery. Although they would clearly disapprove of the patient's actions, her choices carry no risk of harm to them.  Case 2: A 55-year-old man has a 3-month history of chest pain and fainting spells. You feel his symptoms merit cardiac catheterization. You explain the risks and potential www.brain101.info 45 benefits to him, and include your assessment of his likely prognosis without the intervention. He is able to demonstrate that he understands all of this, but refuses the intervention. Can he do that, legally? Should you leave it at that? This patient understands what is at stake with his treatment refusal. As he is competent to make this decision, you have a duty to respect his choice. However, you should also be sure to explore his reasons for refusing treatment and continue to discuss your recommendations. A treatment refusal should be honored, but it should also not be treated as the end of a discussion.  Real Case: Feb. 01, 2002 - Two physicians are faced with their worst fear their own nine- month old baby presents in the ED with a severe drug overdose. Instead of following his colleagues' recommendations, the father refuses to intubate the child until it is clear that she won't last much longer without the procedure. The mother pushes away the on-call critical care pediatrician, demanding another doctor, even if it means wasting precious minutes. Meanwhile, the baby is screaming, struggling for life on a trauma table. This week's episode of ER presents a puzzling, but not uncommon, paradox of the medical profession. If the physician's moral obligation is to always act in the best interests of his patient by providing the most appropriate treatment, why will he sometimes deviate from the standard of care when treating a family member or close friend? Just as Drs. Mark Greene and Elizabeth Corday were reluctant to pursue the traditional and effective method of treatment when their baby fell ill, many physicians provide treatments for their family members that they might not consider for the typical patient. It has been argued that physicians always have their patients' best interests in mind when caring for family members, but may lose sight of this "best interests" standard when treating patients with whom they have no personal connection. While the human urge to protect one's own is admittedly strong, applying this view to the doctor-patient relationship seems to undermine physician integrity. It seems far more likely that physicians, whose careers are founded on the Hippocratic Oath, diligently look to the best interests of all their patients whenever possible. However, when personal interests cloud the lens of objectivity (as when treating family members), a physician may fail to act in the best interests of the patient. Indeed, evidence suggests that physicians treating family members sometimes lose their sense of impartiality and do not act in the patient's best interests. One study reported that 33% of physicians have observed another physician "inappropriately involved" in a family member's care. (1) Such inappropriate involvement may lead to overtreatment or undertreatment, strained professional relations between colleagues with differing views on treatment, improper allocation of hospital resources, and, most significantly, severe emotional strain on the treating physician. Understanding the reasons why physicians sometimes act inappropriately when treating family members is essential when faced with the sort of situation presented in this week's episode. Not only does such understanding help Dr. Greene's and Dr. Corday's colleagues in negotiating the proper course of treatment, but it also may have helped Mark and Elizabeth themselves understand their own biases and take a slightly more objective view towards the care of their child. www.brain101.info 46 There are three common situations in which physicians are often unable to maintain their objectivity when treating family members or children - when the situation is too emotionally charged, when the physician's personal knowledge of the patient makes objectivity impossible, or when the situation is too personal for the physician to feel comfortable as a primary caregiver. (2) Of these, the first situation is by far the most common, and is the one faced by Drs. Greene and Corday this week. When their baby accidentally overdoses on their teenaged daughter's stash of illegal drugs, Mark and Elizabeth's emotions are running high. These parents respond in an understandable fashion - with fear, panic, and a desperate desire to do everything possible to get their child through this emergency. However, unlike most parents, these two are in a position to direct their baby's treatment based on their own medical expertise. Many physicians have a psychological need to feel omnipotent, and any setback in treating a family member may result in "doubts, guilt, or an exaggerated sense of responsibility that may become pathological." (3) For this reason, physicians caring for their own children may be tempted to bypass traditional means of treatment in an effort to avoid exposing the patient to further stress or trauma. While many physician-parents tend to overtreat by taking all possible measures to improve their child's health, Mark and Elizabeth err on the side of undertreatment. Like most clinicians, they recognize how unpleasant and stressful intubation can be on a patient. In an effort to spare their baby this ordeal, and possibly because they are in denial about the severity of her condition, they delay the procedure for far longer than is medically indicated. When the two finally step aside, Dr. Kerry Weaver tells Mark that the course of treatment would have been different had it not been for his and Elizabeth's involvement. "If this wasn't your baby, we would have intubated a long time ago," she emphasizes. Dr. Weaver's statement makes it clear that the baby's best interests were not necessarily served by involving the emotionally-strained parents in her care. When physician-parents are too emotionally involved in their child's care, treatment may not only be rendered less effective, but other related problems may result. Interpersonal relations between physician-parents and colleagues may become strained when colleagues are asked to deviate from the typical standard of care when treating physicians' children. Physician-parents may invoke the hostility of administrators and caregivers by bypassing what seem to them like exasperating administrative procedures - everyday delays in treatment and test results, visitors' hours, insurance paperwork, etc. At the request of the parent, resources may be directed towards the ill child even if they would be better utilized elsewhere, and the care of other patients on the unit may consequently be delayed. Finally, the emotional strain on the parent himself is evidenced by conflicting responsibilities - to the child, to the medical profession, to colleagues or family members, and to his own conscience - which make it impossible for him to look at his child's care objectively. When your own child is in danger, it is often impossible to separate the role of "parent" from that of "physician." Understanding and accepting this struggle of identity helps some physician-parents to step back and leave their child's care in the hands of a trusted colleague, who may be better able to serve both the child's and parent's best interests.  High Yield Re: Inform Consent What is informed consent? Informed consent is the process by which a fully informed patient can participate in choices about her health care. It originates from the legal and ethical right the patient has to direct what happens to her body and from the ethical duty of the physician to involve the patient in her health www.brain101.info 47 care. What are the elements of full informed consent? The most important goal of informed consent is that the patient have an opportunity to be an informed participant in his health care decisions. It is generally accepted that complete informed consent includes a discussion of the following elements: · the nature of the decision/procedure · reasonable alternatives to the proposed intervention · the relevant risks, benefits, and uncertainties related to each alternative · assessment of patient understanding · the acceptance of the intervention by the patient In order for the patient's consent to be valid, he must be considered competent to make the decision at hand and his consent must be voluntary. It is easy for coercive situations to arise in medicine. Patients often feel powerless and vulnerable. To encourage voluntariness, the physician can make clear to the patient that he is participating in a decision, not merely signing a form. With this understanding, the informed consent process should be seen as an invitation to him to participate in his health care decisions. The physician is also generally obligated to provide a recommendation and share her reasoning process with the patient. Comprehension on the part of the patient is equally as important as the information provided. Consequently, the discussion should be carried on in layperson's terms and the patient's understanding should be assessed along the way. Basic consent entails letting the patient know what you would like to do and asking them if that will be all right. Basic consent is appropriate, for example, when drawing blood. Decisions that merit this sort of basic informed consent process require a low-level of patient involvement because there is a high-level of community consensus. How much information is considered "adequate"? How do you know when you have said enough about a certain decision? Most of the literature and law in this area suggest one of three approaches: · reasonable physician standard: what would a typical physician say about this intervention? This standard allows the physician to determine what information is appropriate to disclose. However, it is probably not enough, since most research in this area shows that the typical physician tells the patient very little. This standard is also generally considered inconsistent with the goals of informed consent as the focus is on the physician rather than on what the patient needs to know. · reasonable patient standard: what would the average patient need to know in order to be an informed participant in the decision? This standard focuses on considering what a patient would need to know in order to understand the decision at hand. · subjective standard: what would this patient need to know and understand in order to make an informed decision? This standard is the most challenging to incorporate into practice, since it requires tailoring information to each patient. Most states have legislation or legal cases that determine the required standard for informed consent. In the state of Washington, we use the "reasonable patient standard." The best approach to the question of how much information is enough is one that meets both your professional obligation to provide the best care and respects the patient as a person with the right to a voice in health care decisions. (See also Truth-Telling and Law and Medicine.) What sorts of interventions require informed consent? Most health care institutions, including UWMC, Harborview, and VAMC have policies that state which health interventions require a signed consent form. For example, surgery, anesthesia, and www.brain101.info 48 other invasive procedures are usually in this category. These signed forms are really the culmination of a dialogue required to foster the patient's informed participation in the clinical decision. For a wide range of decisions, written consent is neither required or needed, but some meaningful discussion is needed. For instance, a man contemplating having a prostate-specific antigen screen for prostate cancer should know the relevant arguments for and against this screening test, discussed in layman's terms. (See also Research Ethics.) When is it appropriate to question a patient's ability to participate in decision making? In most cases, it is clear whether or not patients are competent to make their own decisions. Occasionally, it is not so clear. Patients are under an unusual amount of stress during illness and can experience anxiety, fear, and depression. The stress associated with illness should not necessarily preclude one from participating in one's own care. However, precautions should be taken to ensure the patient does have the capacity to make good decisions. There are several different standards of decision making capacity. Generally you should assess the patient's ability to: · understand his or her situation, · understand the risks associated with the decision at hand, and · communicate a decision based on that understanding. When this is unclear, a psychiatric consultation can be helpful. Of course, just because a patient refuses a treatment does not in itself mean the patient is incompetent. Competent patients have the right to refuse treatment, even those treatments that may be life-saving. Treatment refusal may, however, be a flag to pursue further the patient's beliefs and understanding about the decision, as well as your own. What about the patient whose decision making capacity varies from day to day? Patients can move in and out of a coherent state as their medications or underlying disease processes ebb and flow. You should do what you can to catch a patient in a lucid state - even lightening up on the medications if necessary - in order to include him in the decision making process. What should occur if the patient cannot give informed consent? If the patient is determined to be incapacitated/incompetent to make health care decisions, a surrogate decision maker must speak for her. There is a specific hierarchy of appropriate decision makers defined by state law (also see the DNR topic page). If no appropriate surrogate decision maker is available, the physicians are expected to act in the best interest of the patient until a surrogate is found or appointed. Is there such a thing as presumed/implied consent? The patient's consent should only be "presumed", rather than obtained, in emergency situations when the patient is unconscious or incompetent and no surrogate decision maker is available. In general, the patient's presence in the hospital ward, ICU or clinic does not represent implied consent to all treatment and procedures. The patient's wishes and values may be quite different than the values of the physician's. While the principle of respect for person obligates you to do your best to include the patient in the health care decisions that affect his life and body, the principle of beneficence may require you to act on the patient's behalf when his life is at stake. A 64-year-old woman with MS is hospitalized. The team feels she may need to be placed on a feeding tube soon to assure adequate nourishment. They ask the patient about this in the morning and she agrees. However, in the evening (before the tube has been placed), the patient becomes disoriented and seems confused about her decision to have the feeding tube placed. She tells the www.brain101.info 49 team she doesn't want it in. They revisit the question in the morning, when the patient is again lucid. Unable to recall her state of mind from the previous evening, the patient again agrees to the procedure. Is this patient competent to decide? Which preference should be honored? A 64-year-old woman with MS is hospitalized. The team feels she may need to be placed on a feeding tube soon to assure adequate nourishment. They ask the patient about this in the morning and she agrees. However, in the evening (before the tube has been placed), the patient becomes disoriented and seems confused about her decision to have the feeding tube placed. She tells the team she doesn't want it in. They revisit the question in the morning, when the patient is again lucid. Unable to recall her state of mind from the previous evening, the patient again agrees to the procedure. Is this patient competent to decide? Which preference should be honored? This patient's underlying disease is impairing her decision making capacity. If her wishes are consistent during her lucid periods, this choice may be considered her real preference and followed accordingly. However, as her decision making capacity is questionable, getting a surrogate decision maker involved can help determine what her real wishes are. ---------------------------------------------------------------------------------------------------  please clarify my doubt what happen s to the levels of Ach in alzeheimers and what is the role of Acetyl choline esterase inhibitors in this. - Ans. in Alzheimer:Ach decreased coz of cholinergic neurons degeneration Achesterase:breaks down Ach to choline&acetate,so blocking the action of this enzyme by tacrine,donepezil provides more Ach&can delay the progression of Alzheimer dementia  The incidence of cystic fibrosis in caucasian population is 1 in 1600.the expected frequecy of heterozygote carriers in this population is a)1 in 10 b)1 in 20 c)1 in 40 d)1 in 80 e)1 in 160 Ans. B?  In kaplan page 7 behavioral science says "Question: what is the effect of increased prevalence on sensitivity? On Positive Predictive Value"? Answer : Sensitivity stays the same but positive value increases" But WHY? Would you explain to me?And also would you explain in a very simple way what is the relation between Sensitivity, specificity, PPV and NPP and accuracy? - Ans. sensitivity measures how well A TEST identifies truely ill people(so test feature!),it's not correlated to prevalence(number of individuals with disease/total population)but PPV is a probability that someone with+test actually has www.brain101.info 50 disease,and it increased when number of affected people increased in population or in other words prevalence increased. - If a test more sensitive=e.g Elsia=very sensivtive ...it include all case hiv+ve.+but not SPECIFIC.it means if sensitive test it has some cases whoare not HIV +ve only confirm by WESTRON BLOT.THATS why it goes with NPP> IF SPECIFIC....e.g WESTREN BLOT...if positive...it mean all are +ve.no chances that in those ppl one donot have disease...so it goes wih PPV. - I will like to illustrate this question with the case of diabetes diagnosis using FBS. Let us assume that the sensitivity of FBS to detect diabetes is 90%. Therefore if the test is applied to a population with a prevalence of diabetes of 20% then we will expect to identify 9/10 of the 20%.If the total population of of such a community is 200 we well get 36 diabetic based on FBS. This leaves 4 diabetic unidentified. If prevalence of diabetes were to increase due to poor eating habits to about 30% our test will then identify 90% of the 30% of the diabetics which will be 54. Now positive predictive value tells us if you are identified as being diabetic from the test what is the probability that you are diabetic. Every test except for one with a 100% sensitivity and 100% specificity is bound to pick up some false positives. In a highly prevalent community the number of false positives is reduced by virtue of the fact that there are fewer negatives in the population. Thus the positive predictive value which is the number of true positives/(true positives+false positives) will be increased.  tell me the association between sensitivity or specificity to PPV or NPV...i got mixed up sensitivity usually measures all those who truely have disease.then it should be ass. with PPV ..right! but the books say the other way round..plz clarify my concept..any one? - Ans. seN for N sPe for P - sensitivity goes with NPV.Cos if test is more sensitive and you have a + result,you may still be normal(FP).But if u get a - then you sure dont have the disease. Specificity goes with PPV.Ie if u get a + result then you sure are +.But if u get a - you might still have the disease(FN). - PPV probability that someone with+test actually has disease,as it calculated by true+s/sum of true+s&false+s,so it's correlated to specificity (specificity measures how well a test identifies people without disease)and false+:normal patients that incorrectly identified as being ill NPV probability that a person with a -test is actually well,as it calculated by true- s/sum of true-s&false-s,it correlated to sensitivity(sensitivity measures how well a test identifies truely ill people)and false-:ill people that test shows they're normal in summery: sensitivity........false negative.........NPV specificity........false positive.........PPV this is what i know,hope it helps,if not i can explain more - Mnemonic: +PV=true+s/sum of true+s & FALSE+s specificity=true-s/sum of true-s & FALSE+s www.brain101.info 51 - -PV=true-s/true-s & FALSE-s sensitivity=true+s/sum of true+s & FALSE-s -  BQ: which test is used for IQ testing in 2 yrold? - Ans. Stanford Binet scale, coz verbal skill not component of a test, for children <4 yr also: 4-6yr WPPSI 6-17yr WISC-R >=17yr WAIS-R - i meant Stanford Binet <4 yr old, after 4-6yr:WPPSI  What do objective and projective tests mean in beh? Ans. objective test based on Qs easily scored: like MMPI, projective test requires subject to interpret the Qs: like Rorschach test, TAT. --------------------------------------------------------------------------------------------------  in operant conditioning the rate of extinction is most effectively slowed wen the response has been maintained on a reinforcement scedule of 1]fixed ratio 2]variable ratio 3]fixed interval reinforcement 4]continuous reinforcement 5]peicework reinforcement B] extinction is easiest with? Ans. 2]variable ratio forb:4]continuous reinforcement  a behaviour pattern that increases in frequency wen followed by a reward is an exampl of 1]classical conditioning 2]shaping 3]respondent conditionig 4]operant conditionig 5]generalization ans. 4 ]operant conditioning  in a programme aimed at cessation of smoking, a 45yr old woman is required to smoke as many cigarettes in a small booth i short period of time, the mech used here is? 1] habituation www.brain101.info 52 2]aversive conditionig 3]stimulus control 4]variable ratio reinforcement 5]systematic desensitization Ans. 2  an obese women on a wt loss programme is not losing wieght n confesses not to have followed the schedule for the first few weeks n then gone on a crash diet during the week prior to the check up, it is. 1]stimulus control 2]variable ratio reinforcement 3]fixed interval reinforcement 4]variable interval reinforcement Ans. 3, fixed interval ratio , i.e she follows the schedule everytime just befor the planned check up  the best self management procedure in a weight loss intervention programme is? 1]inforamtion control 2]self monitoring 3]self punishment 4]self reward 5]enlisiting social support Ans. 4  An old lady is brought to the hospital in coma by her daughter with whom she lives. The old lady is in coma and is diagnosed to be brain dead. When the daughter is spoken to, she says, “knowing mom, I feel that should something happen to her, she’d prefer to be withdrawn from life support”. Just then , the old lady’s estranged son comes there after hearing the news and he looks at the doc and says “why haven’t u done anything yet, I want the best treatment for my mother?” if I were the doctor, I would have been in a fix, what would you do? Ans. follow daughter's advise (as the caregiver in this case), son is in state of denial!  A pt comes to ur office and tells u ‘doc, I just want to tell someone this, but I’m going to go home and kill my wife’ and then walks out your door… what do you do? Ans. this is tarasoff decision: stop harm so, you need to go after the patient in case if patient gets away - call911, then you call wife and ask her to come to your office /here you need to remember tarasoff 2 www.brain101.info 53  A 16 yr old girl comes to you and tells you she is planning to have sex with her boyfriend and does not want to get pregnant. She asks you to write a prescription for birth control pills. What will you do as a physician ? - Ans. parental consent not reqd for contraceptive prescription.Also treatment in emergency,drugor alcohol addict,STD treatment. So i guess you could give OC but counsel them. - Hey I just verified the contraceptive usage thing. These are the 4 categories where a minor living with his parents does not need parental consent --> STD treatment, contraceptives, substance abuse treatment, prenatal care.  A 15 yr old mother of one, comes with genitourinary complaints. Whose consent will the doctor have to take to examine her ? - the 15 year old again. theses are two fields.....Pregnant girl <18, mother less than 18, and an independant minor who can give consent solely. and Correct me if I am wrong..is it the same rule for a boy or a girl,<18 who ask for the advise in contraception with out telling their parents! - a married minor too... - One more thing is a pregnant lady or a teenaged mom under 18 yrs of age does NOT automatically become emancipated(not sure of the spelling!) ie doesn't become independent to give consent unless she is ACTIVELY TAKING CARE OF THE CHILD  Well, here’s a 17 yr old boy in the army and he comes with a bullet wound in his thigh. U need to remove it surgically. Who do you have to take consent from? Ans. the boy himself because he is independant minor.  TRUE OR FALSE? Feeding tube is a medical treatment and can be withdrawn at the patient's request - Ans. True - How about in anorexia nervosa? - Ans. false in case of anorexia nervosa. anorexia nervosa usually seen in girls teens e.g. 15y.o., in this case doesn't matter what she said if this pt >18 - go to the cort /this is an exeption when on the test answer is go to the cort.  TRUE OR FALSE ? a competent patient has the right to refuse even life saving hydration (oral or I.V fluids) Ans. True if >18  An 8 yr old child with IDDM needs to undergo treatment with insulin but the parents refuse surgery and say they will take the child home and pray for its welfare… what should the doctor do? www.brain101.info 54 (a) Don’t start treatment (b) decide to go ahead against the parents’ wishes and start Rx ( c) try to convince them to agree threatening them with the child’s death (d) go to court with the dispute. - Ans. Could have been b,But patient is not serious enough immediately.(like DKA). So now could have been c but you cant threaten them.Only convince. So D is best i guess. - we have to go to court with the dispute. This is the only reason to go to court , other decisions should be taken outside the court  If a patient, a 52 yr old male with no other illnesses, comes with signs of ischemic heart disease and you, as a doctor know he needs an emergency coronary artery bypass graft surgery but he REFUSES surgery. What next? (a) ignore him and perform surgery (b) take consent from his spouse (c) don’t do surgery (d) threaten him that he will die if he doesn’t undergo surgery Ans. C, and mention in the notes that you have explained to the patients the need of surgery and he refuses,  how do you assess for visual retention of presented objects? how do you differentiate from Bendervisualmotor Gestalt test ans. The Bender test is a brief, nonverbal assessment consisting of nine stimulus cards, each showing a figure. The client is asked to reproduce the figures, a task requiring visual association, motor coordination, and the ability to integrate perceptual and motor skills to achieve accurate reproductions. AND Dynamic Visual Retention Test (W.H. Gaddes, 1988 Revision) has been shown to be sensitive to left frontal lobe damage/dysfunction and to discriminate dyslexic readers  BQ: by decreasing the upper limit of normal of a test,what changes are seen in: 1)sensitivity? 2)PPV? 3)false negative rate? Ans. sensitivity:increased PPV:decreased false negative rate:decreased www.brain101.info 55  what are alpha and beta errors in stat? Ans. Alpha error is we reject(fail to accept) the null hypothesis when the null is true. Beta error is when we accept the null hypothesis though it is false. To have an anology, alpha error occurs when you convict an innocent person and beta error occurs when you free a guilty person. Both these errors have to be kept very low. The value 1-beta should be very high because that indicates the power of the test.  Why is Stanford binet is best for young children? Ans. it's the ratio of mental age to chronological age multiplied by 100 as it's not completey based on language performance best for children 2-4yr  Mnemonic on Alzheimer’s disease: - RONALD (Ronald Reagan, a famous victim): R Reduction of Ach O Old age N Neurofibrillary tangles A Atrophy of cerebral cortex (diffuse) L Language impairment D Dementia (MC in elderly)/ Down’s syndrome - very important... and what congenital anomaly is it associated with in those over the age of 40?....ans. Down’s syndrome - so what do you see in the brain?...ans. amyloid deposits..and neurofibrillary tangles - we see plaques due to amyloid deposition. in down syndrome patient beta amyloid protein production increses which causes development of plaqes.  most likely method used by both males and females to commit suicide is? Ans. Pills to try to kill themselves... Gun to complete the act...  most admissions in the hospitalisation r due to? Ans. psych reasons including drug abuse  in which subspeciality of physicians highest suicide rate is seen? Ans. Psychiatrists  this disease diagnosis involves more malpractice suits than any other? Ans. Breast cancer diagnosis.  highest suicide rates are seen in which profession? - Ans. Dentists #1 Psychiatrists #2  which neurotransmitter is most imp for sleep serotonin or acytlcholine? Ans. Both. In causing sleep its Seratonin and Ach. In Waking up its Norepi, and Dopamine.  Which is the most common form (Nicotine, Caffine, Alcohol) of drug use in US? Ans. ALCOHOL! Alcohol is the most abused drug. Therefore FAS is the most common cause of MR. www.brain101.info 1 CARDIAC TESTS • Chest X-ray o X-rays are a form of electromagnetic radiation (like light); they are of higher energy, however, and can penetrate the body to form an image on film. There is low radiation exposure. X-rays are monitored and regulated to provide the minimum amount of radiation exposure needed to produce the image. o A chest X-ray may be ordered when an person's symptoms include a persistent cough, coughing up blood, chest pain, a chest injury, or difficulty in breathing. The test is also used when tuberculosis, lung cancer, or other chest or lung disease is suspected. o A serial chest X-ray (repeated or sequential) may be used to evaluate changes over time if an abnormality found on a chest X-ray (for example, an increase in the size of an abnormality over a period of weeks). • EKG o What is it? An electrocardiogram – often abbreviated, as EKG or ECG – is a test that measures the electrical activity of the heartbeat. With each beat, an electrical impulse (or “wave”) travels through the heart. This wave causes the muscle to squeeze and pump blood from the heart. o Why is it done? An EKG gives two major kinds of information.  1 st , find out how long a wave takes to travel from one part of the heart to the next shows if the electrical activity is normal or slow, fast or irregular.  Second, by measuring the amount of electrical activity passing through the heart muscle, a pediatric cardiologist may be able to find out if parts of the heart are too large or are overworked. • HDP o Swan catheter • Exercise Stress Test o Definition  An exercise stress test is a general screening tool to test the effect of exercise on your heart. The test gives a general sense of how healthy your heart is.  During the test, the electrical activity of the heart is measured while you walk on a treadmill or pedal a stationary bicycle. This measures the heart's reaction to your body's increased demand for oxygen. (See also sestimibi and thallium stress tests.) o How the test is performed  You will be asked to walk or pedal on an exercise machine. An electrocardiogram (ECG) is used to record the activity of your heart and blood pressure readings are taken. The response of the heart to this increased workload is monitored.  The test continues until you reach a target heart rate, unless complications such as chest pain or an exaggerated rise in blood pressure develop with activity. Monitoring continues after exercise for 10 to 15 minutes or until the heart rate returns to baseline. • Thallium Stress Testing o Nuclear imaging methods that provide a view of the blood flow into the heart muscle. The thallium and sestamibi tests are also called “MIBI stress test” and “myocardial perfusion scintigraphy”, and are used to evaluate how well your heart is perfused (supplied with blood) at rest as compared with activity. o During these tests, heart images can be obtained because the patient receives an injection of a substance that is labeled with a radioactive marker or radiotracer to make it visible in the bloodstream. These substances are also called radiopharmaceuticals, and include thallium-201 and technetium-99m MIBI or sestamibi. www.brain101.info 2 o In comparison to the standard treadmill stress test, thallium and sestamibi stress tests are more accurate and provide additional information. o How the test is performed  Exercise as hard as you can on a treadmill or bicycle  When you reach your maximum level of exercise, a nurse will inject in your vein a small amount of a radioactive substance (radiotracer), either thallium or sestamibi  The radiotracer will travel in the bloodstream and, through the coronary arteries, will enter into the heart muscle as you complete your exercise session.  After you finish exercising, you will lie down on a special table under a bulky camera called a gamma camera. The gamma camera can scan your heart and detect the radiotracer in it.  The distribution of the radiotracer in your heart will be processed by a computer to create pictures of your heart. The first pictures are made shortly after the exercise test, to show the circulation of blood to your heart during exercise. This is the part considered "a stress test" and is the most challenging for your heart.  Then you will need to lie quietly for 2-3 hours, and at that point the scanner will make another series of pictures of your heart. These images will show the circulation of blood through your heart muscle at rest. • Myoview or Cardiolite Stress Testing; Sestamibi o Advantage  Technetium instead of Thallium which is brighter and better  Used for women w/ large breasts or men with big guts  Fewer false positive than Thallium, better for screening  Specificity is 96% (Thallium is 85%)  Good screening tool o Disadvantage  Not as good for viability as Thallium  Not as good for people who have had an infarct • Persantine Stress Test o What is an IV Persantine-Thallium Stress Test?  This test is an alternative procedure for patients with coronary artery disease who can not exercise on a treadmill.  An IV line is started to administer the persantine and the thallium.  Persantine helps to expand the coronary arteries increasing the blood flow to the area. This effect is similar to what happens during vigorous exercise.  Thallium (a radioactive isotope) is detected by X-rays and shows which parts of the heart muscle are receiving enough oxygen.  EKG used to moniter.  A special camera takes pictures of the heart.  2 phases, with each phase lasting 30-60 minutes o Why is a Persantine Stress Done?  This test will help the doctor to evaluate the patient's cardiac condition related to:  Irregular heart rhythms  Which areas of the heart are not getting enough blood and oxygen.  How quickly the heart recovers after exercise. o Good area dilates more to Persantine and shows up as bright, bad area is dull. o Steal can be created, but only occurs w/ very high grade stenosis, pt will become symptomatic w/ chest pain • Multigated Acquisition Wall Motion Scan (MUGA) o During the MUGA test, a radioactive isotope (Technetium) is injected into the vein o Radioactive isotopes attach to red blood cells and pass through the heart in the circulation o The isotopes can be traced through the heart using special cameras or scanners. Ejection fraction (normal=55%, 30% after one MI, <30% major risk) can be measured by observing volumes during systole and diastole. www.brain101.info 3 o The test is often given at rest, then repeated with exercise, or after administering certain medications. The test is performed to detect certain heart conditions. o Easily done and reproducible • First-pass radionuclide angiocardiography o Nuclear imaging, same information as MUGA, but harder to do o Injection of a bolus of a technetium-99m-labelled RBCs o Nuclear scan images acquired in rapid sequence during the injection o Images recorded allow the calculation of the ventricular ejection fraction (EF), the determination of shunt volumes and wall motion abnormalities o First-pass radionuclide angiocardiography is less frequently used nowadays, as gated blood pool scanning yields much better results. Gating of the data to the cardiac cycle permits addition of the five or so cardiac cycles observed during the first pass of the radionuclide, thereby improving the statistical accuracy of the EF determination. • Spect o Single photon emission computed tomography (SPECT) is a noninvasive technique for creating very clear, three-dimensional pictures of a major organ (e.g., the heart) o Uses radionuclide imaging – a technique that involves the injection of very small amount of a radionuclide substance called a tracer. Energy from the tracer in the body is detected by a gamma ray camera, which then takes the pictures. A tracer is not a dye (contrast medium) • Gated Spect o In addition to aquiring images, you play them back like a movie?? o You get wall motion • Echo Doppler o You get ejection fraction, wall motion o An instrument that transmits high-frequency sound waves called a transducer is placed on your ribs near the breast bone and directed toward the heart. The transducer picks up the echoes of the sound waves and transmits them as electrical impulses. The echocardiography machine converts these impulses into moving pictures of the heart. o Best test to test DYASTOLIC DYSFUNCTION o This test is performed to evaluate the valves and chambers of the heart in a noninvasive manner for  heart murmurs  check the pumping function of the heart  evaluate patients who have had heart attacks o It is a very good screening test for heart disease in certain groups of patients. • TEE o Swallow a probe o Sedate patient, use Lidocaine o You get a better view of the heart o Sometimes you can’t see the Left Atrium too well o Use on pts:  Suspected descending aortic dissection  Endocarditis  Need a closer look • Non-Invasive Vascular Testing o Plethysmography (Pulse Volume Recording w/ Segmental blood pressures)  A test is performed by placing blood pressure cuffs on the extremities to measure the systolic pressure  The cuffs are then attached to a pulse volume recorder (plethysmograph) that displays each pulse wave.  Compares the systolic blood pressure of the lower extremity to the upper extremity, to help rule out disease that blocks the arteries in the extremities.  Use for diabetic pts over 70, non-healing ulcers, cellulitis, claudication Sx, reduced pulses www.brain101.info 4 o Venous Duplex  Use when there is a venous problem • Vericous veins • Edema • Venous stasis • Stasis ulcers • Carotid Doppler o Carotid duplex is an ultrasound procedure performed to assess blood flow through the carotid artery to the brain. High-frequency sound waves are directed from a hand-held transducer probe to the area. These waves "echo" off the arterial structures and produce a 2-dimensional image on a monitor, which will make obstructions or narrowing of the arteries visible. o Carotid duplex is a procedure that uses ultrasound to look for  Plaques (Intimal thickness)  Blood clots  Aneurisms  Other problems with blood flow o Predict TIAs and strokes (CVAs) and if pt would benefit from stent • Holter o Definition: 24 hours of ECG recordings o Used to check for  Arrythmias (tachy, brady, V-tach)  Post MI  Palpataions, dizziness  Sudden death risk • Signal Average ECG o Non-invasive o Identifying risks for potentially fatal heart rhythm problems o The procedure involves obtaining electrocardiograph signals from the heart, amplifying them, and then filtering and averaging them by computer o The procedure may detect "late potentials," low amplitude signals associated with serious rhythm abnormalities, which can lead to sudden cardiac death. o Useful in same pts as holter • HRV (Heart Rate Variability) Test o High variability is good (HR quickens as you run up stairs, for instance, and slows as you nap) • T Wave Alterans o Low level stress test (bring hear to 110) o Look at T wave  If they are stable → good  If they invert, change → bad o Helps predict risk of sudden death o Used in same pts as holter • Transtelephonic monitering o Holter through the telephone o Loop recorder records heart beat for 30 days o Used in same pts as holter • Pacemaker Testing o Monitor function of pacemaker over the phone • Ambulatory Blood Pressure and HR o Monitor blood pressure for white coat syndrome pts • EPS o Intracardiac electrophysiology study (EPS) involves placing wire electrodes within the heart to find the location of a known arrhythmia and determine the best therapy www.brain101.info 5 o Abnormal electrical activity can occur anywhere along the conduction system, including in the muscle cells of either the atria or ventricles. The electrodes inserted during EPS will map the type of arrhythmia you have and where the problem arises in your heart. o Allows for determination of the severity of the problem (including whether you are at risk for sudden cardiac death) as well as appropriate treatment. o Good for pts w/ syncope, posts MI, arrhythmias • Ablation o Nonsurgical method o Insert a thin tube (catheter) through a blood vessel (in the upper thigh, wrist or arm) and all the way up to the heart o At the tip of the tube is a small wire, which can deliver radiofrequency energy to burn away the abnormal areas of the heart o Success rate over 90 percent o Treats  Tachycardia  Wolff-Parkinson-White syndrome: episodes of tachycardia caused by abnormal electrical pathways (circuits) in the heart  V-tach w/ normal ventricle (Not good for people post MI)  A-fib • Most you can’t burn b/c its due to stretch fibers • Some you can burn b/c it’s from one irritable focus • Pacemakers o Biventricular pacemaker  New type of artificial pacemaker designed to treat heart failure  In many heart failure patients, the walls of the left ventricle are no longer synchronized, these pacemakers resynchronize  Standard pacemakers pace either the lower-right chamber of the heart (single chamber pacemaker) or both the lower-right and the upper-right chambers (dual chamber pacemaker). In contrast, biventricular pacemakers pace both of the lower chambers of the heart (the ventricles).This enables the device to stimulate the left and right ventricles simultaneously, which can enable the left ventricle to pump blood more efficiently  Used to treat heart failure  Used for pts w/ • Stage 3 / 4 heart failure (sx w/ rest or minimal activity) • Ejection fractions <30% • Have an IVCD? • QRS interval >130 msec  Buy stock in Guidance? • ICDs o An implantable cardioverter defibrillator (ICD) is a device that is implanted in the chest to monitor for and, if necessary, correct episodes of rapid heartbeat. o If the heartbeat gets too fast (ventricular tachycardia), the ICD will stimulate the heart to restore a normal rhythm (anti-tachycardia pacing). o In cases where the heartbeat is so rapid that the person may die (ventricular fibrillation), the ICD will also give an electric shock (defibrillation) to “reset” the heartbeat. o Similar to an artifical pacemaker, which is another type of device that corrects an abnormal heart rhythm. However, pacemakers are usually chosen to correct a heart rhythm that is too slow (bradycardia), whereas ICDs are used to correct a heart rhythm that is too fast (tachycardia). And there are patients who need both bradycardia pacing and anti- tachycardia pacing. In these patients, an ICD will be used to pace the heart. o Minor surgical procedure (not open-heart surgery) • AEDs www.brain101.info 6 o A defibrillator is a device that attempts to restore a normal heart rhythm by delivering an electrical shock to the heart. A defibrillator is used when the heartbeat is dangerously fast due to ventricular tachycardia or ventricular fibrillation. Either of these conditions can be life- threatening, because the heart may abruptly stop pumping blood to the body (cardiac arrest). Some defibrillators are external (e.g., defibrillator paddles in an emergency room), and some are surgically implanted in the patient’s chest (e.g., an implantable cardioverter defibrillator). o Recent advances in technology have allowed people with little training to use automatic external defibrillators (AEDs) in an emergency when medical professionals are not present. o The American Heart Association estimates that 20,000 lives would be saved every year in the United States if AEDs were more widely available. The chance of surviving cardiac arrest decreases by 10 percent with every minute that ticks by without defibrillation. • Cardiac Catheterization o Used to study the various functions of the heart or to obtain diagnostic information about the heart or its vessels o A small incision is made in an artery or vein in the arm, neck, or groin. The catheter is threaded through the artery or vein into the heart. X-ray images called fluoroscopy are used to guide the insertion. o When the catheter is in place, dye is injected to visualize the structures and vessels within the heart. o Using different techniques, the coronary arteries can be viewed by injecting dye or opened using balloon angioplasty. The oxygen concentration can be measured across the valves and walls (septa) of the heart and pressures within each chamber of the heart and across the valves can be measured. The technique can even be performed in small, newborn infants. • PTCA (POBA) o Balloon angioplasty is one of three standard treatments for coronary artery disease (CAD) — a disease in which the blood flow to the heart and the body is restricted due to hardened arteries (atherosclerosis). The other standard treatments for CAD are medication and bypass surgery. o The goal of balloon angioplasty is to push the fatty plaque back against the artery wall to make more room for blood to flow through the artery. This improved blood flow reduces the risk of heart attack and sudden cardiac death. o Physician uses an artery into which a thin tube with an uninflated balloon at the tip (balloon- tipped catheter) will be inserted. He inserts the balloon-tipped catheter through the femoral artery all the way up to the heart o Once the balloon-tipped catheter is at the site of the blockage, the balloon at the tip of the catheter is inflated, pushing the plaque in the artery back against the wall of the artery o The balloon-tipped catheter is then removed or replaced with a stent (a wire mesh tube used to hold the artery open). The patient is then given time to recover o Most patients are free to go home after about 24 hours. • Stents o w/ PTCA o everybody gets them o The artery must be <2 mm • Heart Surgery o Off pump, better to do surgery this way (let the heart beat naturally) o Annuloplasty: Reconstruction of the ring (or annulus) of an incompetent cardiac valve. • INDICATIONS for doing an ECG o Class I You have to do the tests o Class 2A You should do the tests o Class 2B You don’t have to o Class 3 You cannot do the tests www.brain101.info 1 BIOCHEMISTRY • Proteins and Enzymes • Electrolytes and Acid-Base Balance • Case Studies PROTEINS and ENZYMES Erythrocyte Sedimentation Rate (ESR): A high ESR occurs when the body is infected or under stress, and the liver is releasing acute-phase proteins into the blood. ACUTE-PHASE PROTEINS: Proteins released by the liver when the body is under stress. • alpha 1 -Antitrypsin: Protease inhibitor. When there is tissue damage, the dead tissue releases proteases, so the anti-proteases help to prevent further damage. • alpha 2 -Macroglobulin: Indirect anti-protease that fixes proteases and allows macrophages to engulf them. • C-Reactive Protein (CRP): Opsonin, help to fix antibodies to antigen to facilitate phagocytosis. • Ceruloplasmin: Copper-carrying protein, and anti-oxidant. • Complement Proteins: Inflammatory mediators. • Ferritin: Iron protein-carrier. • Fibrinogen: Clotting factor. • Haptoglobin: Binds to hemoglobin in blood. • Serum Amyloid A Protein: Apolipoprotein. PROTEIN ELECTROPHORESIS: alpha 1 , alpha 2 , beta, and gamma zones all have distinct proteins. • alpha 1 Zone: Closest to the anode (right). o Albumin: Albumin is the tall peak closest to the anode.  Normal properties:  50% of liver protein production; primary determinant of oncotic pressure.  20-day lifespan in circulation. If albumin decreases, it won't show up until 20 days later.  MW = 68 kDa, which is right at the margin for glomerular filtration. That's why even mild glomerular disease leads to albuminuria.  Albumin is decreased under a lot of circumstances:  Renal disease, proteinuria.  Times of stress or disease.  Malnutrition, Kwarshiorkor.  Albumin binds to bilirubin and Ca +2 .  A decreased albumin levels can significantly alter the laboratory values for bilirubin and calcium. If albumin is low, then these lab-values will be falsely low, and you must adjust them upward to get the real value. o Pre-Albumin: Fetal albumin is called pre-albumin. It consists of two proteins.  alpha-Fetoprotein (AFP):  Anencephaly, Spina Bifida: AFP leaks out of the fetus and into the maternal circulation, thus AFP is increased in maternal blood.  Liver Cancer, Endodermal Sinus (Yolk-Sac) Tumor: The tumors contain immature tissue thxat releases pre-albumin, thus AFP is increased.  Transtherytin: Fetal form of TBG that carries T 3 and T 4 in fetal blood. • alpha 2 Zone: o alpha 2 -Macroglobulin: Huge molecule that binds to proteases and thus allows macrophages to engulf them, getting rid of the proteases. o Haptoglobin: Binds to free hemoglobin in the plasma. Its maximum binding-capacity is about 10% of all hemoglobin in blood. www.brain101.info 2  If free haptoglobin is decreased (all bound up) and free hemoglobin is increased, then that indicates intravascular hemolysis, such as that caused by blood-type incompatibility or artifical heart valves. o Ceruloplasmin o GC Globulin • beta Zone: o LDL Lipoprotein o Transferrin: Iron transporting protein. o C3 Complement Factor o beta 2 -Microglobulin: Part of the Major Histocompatibility Complex. o Hemopexin: Binds free heme (hemoglobin degradation product) -- not hemoglobin itself, as in Haptoglobin. • gamma Zone: Closest to cathode (left). o Immunoglobulins (Ig): o C-Reactive Protein (CRP): Good marker during wound-healing. If it increases during wound- healing, then the wound is probably getting infected.  Originally discovered as a protein that binds to Streptococcus Pneumoniae. o Fibrinogen: o Lysozyme: GAMMOPATHIES: • Polyclonal Gammopathy: Broad gamma peak, indicating infection. • Monoclonal Gammopathy: Narrow gamma peak. Differential: o Multiple Myeloma, 60%. Malignancy of IgG-secreting plasma cells. o Waldenstrom Macroglobulinemia, 10%. Hypersecretion of IgM. o Lymphomas, Leukemias, 10% o Monoclonal Gammopathy of Unknown Significance (MGUS), 10% o Rare causes: Heavy chain disease, primary amyloidosis, solitary plasmacytoma. • Hypogammaglobulinemia: No peak or shallow peak in gamma range. o Due to inherited immune deficiency:  X-Linked IgA Deficiency: Common, 1/750 births.  Agammaglobulinemia: Rare. o Acquired causes: Malignancies, immunosuppressive drugs, HIV, measles, malnutrition. PLASMA ENZYMES: • Alkaline Phosphatase (Alk.Phos.): Increased Alk.Phos. indicates: o Cholestasis o Increased bone growth or reformatuion. Osteoblasts secrete Alk.Phos. • Alanine Aminotransferase (ALT): Increased ALT indicates liver damage. It is released into circulation from damaged or necrotic liver cells. • Aspartate Aminotransferase (AST): AST is released from a variety of damaged cells. Increased ALT indicates: o Liver damage o Post-MI o General cellular injury. • Myocardial Infarct (MI): CAL is a mnemonic to remember the order in which enzymes increase: o Creatinine Kinase (CK): 4-8 hrs. post-MI o Aspartate Aminotransferase (AST): Goes up next. o Lactate Dehydrogenase (LDH): Last one to go up. • Creatinine Kinase (CK): Isozymes o CK-MM: 99% of skeletal muscle, and about 77% of myocardium. o CK-MB: About 22% of myocardium, but it is not found in any other tissues, so CK-MB is a significant marker for myocardial infarct. o CK-BB: Forms greater than 90% of CK in other tissues, such as CNS, colon, and ileum. • Lactate Dehydrogenase (LDH): Isozymes www.brain101.info 3 o LDH-1: The predominant isozyme in myocardial tissue. High LDH-1 indicates MI. • LDH-Flip: LDH-5 is normally highest, but in cases of MI LDH-1 may be higher. This is called an LDH- flip and is suggestive of MI. o LDH-2 thru LDH-4: Minor isozymes. o LDH-5: The predominant isozyme in liver and skeletal muscle. It is normally the highest, except in cases of MI LDH-1 may be higher. PROTEINURIA: Can be caused by three mechanisms: • OVERFLOW: o Normal proteins in blood: hemoglobinuria, myoglobinuria. o Abnormal proteins in blood: Bence-Jones protein (IgG light-chains found in Mutiple Myeloma). • GLOMERULAR: Primarily albuminuria. o Fever, glomerulonephritis cause higher renal permeability. o Altered hemodynamics (such as exercise) can transiently cause proteinuria. • TUBULAR: o Tubular damage due to heavy-metal poisoning, drug toxicities. o Interstitial nephritis, pyelonephritis. o beta 2 and alpha 1 Microglobulin will be found in urine. They are normally filtered and reabsorbed, but with tubular disease they won't be reabsorbed. ELECTROLYTES and ACID-BASE TOTAL BODY WATER: • TBW is normally 60% of body weight. 60% of 70 kg = 42L o INTRACELLULAR: Intracellular fluid is normally two thirds of TBW. 67% of 42L = 28L o EXTRACELLULAR: Extracellular fluid is normally one third of TBW. 33% of 42L = 14L  PLASMA VOLUME: Plasma is normally about 5% of TBW. 5% of 42L = 2-3L  INTERSTITIAL VOLUME: ISF is the rest of the volume. 14L - 3L = 10-11L. POTASSIUM: Reference range 3.5 - 5.0 mEq / L • HYPOKALEMIA: Decreased K + in plasma o Hypokalemia is usually accompanied by metabolic alkalosis. • HYPERKALEMIA: Increased K + in plasma o Hyperkalemia is usually accompanied by metabolic acidosis. SODIUM: Reference range 135 - 146 mEq / L • HYPONATREMIA: o Pitting Edema: Fluid has moved from vascular space into interstitial space. The intracellular spaces are not affected.  It occurs because of an off-balance of Starlnig's Forces:  Too much hydrostatic pressure: CHF  Too little oncotic pressure: Nephrotic Syndrome, Liver Cirrhosis  One usually finds hyponatremia with these conditions, because the patient has gained more water than sodium, so the sodium levels are diluted. o Cerebral Edema: In hyponatremia, water enters into neuron cells in brain ------> cerebral edema. Potential for herniation if it is not corrected.  Idiogenic Molecules are osmotically active molecules created by the cerebrum, to try to compensate for the cerebral edema. They are excreted into the ISF to try to suck the water out of the cells. www.brain101.info 4 o Syndrome of Inappropriate ADH (SIADH): It is the most common cause of hyponatremia with a normal physical exam (no edema, no lost skin turgor).  CAUSES: Ectopic production by a tumor, such as small-cell carcinoma of the lung.  TREATMENT: Restrict intake of water. Electrolyte balance remains normal; no electrolyte adjustments are needed. Treat with ADH antagonists. • HYPERNATREMIA: o Dehydration: Pure water loss, infantile diarrhea.  TREATMENT: Don't give the calculated amount of fluid back to the patient. Always give less, to prevent cerebral herniation. The brain will make osmotically active idiogenic molecules to try to compensate for the dehydration. Then if you give too much fluid, the brain can herniate. o Diabetic Ketoacidosis (DKA):  Due to increased osmotic force of hyperglycemia, fluid will move from the intracellular space into the plasma.  Sodium Concentration must be adjusted for the presence of hyperglycemia (which isn't normally taken into account). The sodium concentration will be actually higher than what is reported. This is true with blood sugar > 500. • TOTAL BODY SODIUM (TBNa): o Decreased TBNa ------> decreased fluid in interstitial space ------> decreased skin turgor. You can pull on skin and it isn't as elastic or tight. o Increased TBNa ------> increased fluid in interstitial space ------> pitting edema. Fluid accumulated in interstitial space.  Patient may still have Hyponatremia, because the sodium concentration in blood is decreased. But, the total body sodium will be increased. DEHYDRATION: • Adult Diarrhea is isotonic, thus adult dehydration will show normal sodium levels. o To rehydrate, give them what they lost -- an isotonic saline solution. • Infantile Diarrhea is hypotonic, thus infantile diarrhea will show hypernatremia. o To rehydrate, give them what they lost -- a pedialyte, or hypotonic saline solution. EDEMA: • Cellular Edema: Tissue Hypoxia leads to cellular edema. Hypoxia ------> Na + /K + -ATPase Pump failure -- ----> Na + collects inside cells and brings water with it ------> hydropic swelling of cells and cellular edema. • Pitting Edema: CHF, Nephrosis, Cirrhosis lead to pitting edema. Transudate collects in interstitial space, either due to increased hydrostatic pressure (CHF) or decreased oncotic pressure (Nephrosis, cirrhosis, malnutrition). • Cerebral Edema: In hyponatremia, water enters into neuron cells in brain ------> cerebral edema. Potential for herniation if it is not corrected. o Idiogenic Molecules are osmotically active molecules created by the cerebrum, to try to compensate for the cerebral edema. They are excreted into the ISF to try to suck the water out of the cells. OSMOLALITY: Normal value is about 289 mOsm. ACID-BASE: • METABOLIC ACIDOSIS: Decrease the HCO 3 - ------> the pH goes down. Compensation: Respiratory Alkalosis will bring the pH back near normal. • METABOLIC ALKALOSIS: Increase the HCO 3 - ------> the pH goes up. Compensation: Respiratory Acidosis (hypoventilation) can help to bring the pH back near normal. o CAUSES:  Vomiting: Lose enough stomach acid to produce alkalosis. www.brain101.info 5  Diuretics: Loop diuretics and thiazides can lead to hypokalemia ------> secondary metabolic alkalosis. • RESPIRATORY ACIDOSIS: Increase the PCO 2 ------> the pH goes down. Hypoventilation. Compensation: Metabolic Alkalosis can help bring the pH back near normal. • RESPIRATORY ALKALOSIS: Decrease the PCO 2 ------> the pH goes up. Hyperventilation. Compensation: Metabolic Acidosis can help bring the pH back near normal. ANION GAP: Essentially, the difference between between the concentrations of cations (Na + primarily) and anions (Cl - , HCO 3 - ) in the blood. • High Anion Gap: Metabolic Acidosis. It indicates that you have added acids to the blood: salicylic acid, formic acid, lactic acid, oxalic acid, sulfuric acid. • Normal Anion Gap: Respiratory Acidosis. It occurs when you ultimately become acidotic because of losing HCO 3 - . NORMAL VALUES of ARTERIAL BLOOD-GASES: Item Value pH 7.4 [HCO 3 - ] 22 - 28 mEq / L PaCO 2 33 - 44 mEq / L PaO 2 90 - 100 mEq / L CASE STUDIES ACID-BASE CASE STUDIES: Case # pCO 2 PO 2 HCO 3 - pH Explanation Case 1 70 low 27 7.2 Acute Barbiturate Overdose. PCO 2 is high ------> respiratory acidosis from hypoventilation. It is uncompensated because the HCO 3 - is normal and the pH is low. Case 2 70 100 12 7.0 Code Arrest. High PCO 2 ------> respiratory acidosis. Also, low HCO 3 - ------ > metabolic acidosis. It's a mixed disorder. Case 3 59 50 31 7.34 COPD. Partially compensated respiratory acidosis. High PCO 2 , high HCO 3 - (metabolic alkalosis) in compensation, near normal but slightly low pH. Case 4 29 100 22 7.50 Hyperventilation. Uncompensated respiratory alkalosis. Case 5 50 80 12 7.27 Chronic Renal Failure. Patient shows partially compensated metabolic acidosis with high anion gap. Patient can't excrete all the acid he is creating. Case 6 50 80 42 7.52 Diuretics in a non-smoking female. Metabolic Alkalosis (high HCO 3 - ) with partially compensated respiratory acidosis (low PCO 2 ). Case 7 62 50 36 7.37 COPD, loop diuretic. Mixed disorder. Respiratory acidosis from COPD, and metabolic alkalosis from loop diuretic. The pH is near normal but it should not be called compensated, because full compensation never occurs, and the pH is the result of two unrelated processes. www.brain101.info 6 CLINICAL BIOCHEMISTRY CASE STUDIES: Case Pertinent Lab Values Explanation 1 Potassium Lab Error, Addison's Disease High K + High Urea Low Na + K + was high becuase of partial hemolysis of blood, because blood was aged. Labs could indicate Addison's Disease, but they need to be retaken. 2 Potassium Lab Error High K + K + of 45 is incompatible with life. 3 IDDM Glucose tolerance test: young kid most likely has a transitory hyperglycemia, because he just ate. Next day glucose is normal 4 Starvation, Dehydration ICF and ECF will shrink to the same extent. Drink seawater: death due to hypernatremia, diarrhea from magnesium in the sea-water. 5 Dehydration High Na + , high Cl - High urea (pre-renal failure) Low HCO 3 - (acidosis) Man lost pure water ------> dehydration with hypernatremia. He had hypotension, high pulse. Pre-renal failure: Due to inadequate perfusion of kidneys; uremia (high urea) is more prominent than high creatinine. 6 Paraneoplastic SIADH Low serum osmolality, low urine osmolality. Low Na + , low Cl - High K + (aldosterone is not being secreted at all) Differential should include Diabetic Ketoacidosis. 7 Dehydration High urea All electrolytes are low. Low HCO 3 - , acidosis. Uremia: pre-renal failure due to hypotension. These labs would not be found in end-stage kidney failure. 8 Injury with Lactic Acidosis High Na + High K + Low HCO 3 - , metabolic acidosis Hyperkalemia is often associated with metabolic acidosis. Give calcium chloride immediately to prevent arrhythmias associated with the hyperkalemia. 9 Volume depletion after surgery Low Na + Her sodium was depleted from surgery. Her responses to the low sodium included all things except reduced GFR. 10 Creatinine Clearance Erroneous collection of urine is most common mistake in measuring creatinine. 11 Diabetes Insipidus Normal electrolytes (more or less). Lack of ADH. ADH effects osmolality and plasma volume, but not electrolyte balance. www.brain101.info 7 (more or less). High serum osmolality. Low urine osmolality. volume, but not electrolyte balance. 12 Septicemia with acidosis, pre- renal failure High K + High urea High serum osmolality High K + is associated with acidosis. Renal disturbance is due to pre-renal failure. Serum urea is also increased because patient is in a state of excessive catabolism. 13 Renal Osteodystrophy High urea, high creatinine Chronic renal failure ------> low, calcium and Vitamin-D ------> high PTH. 14 Compensated Metabolic Alkalosis High HCO 3 - (alkalosis), low PCO 2 (compensatory acidosis). pH is high, but variable. Low K + Patient had severe vomiting, and later had shallow respirations. Low K + is associated with alkalosis. 15 Respiratory Acidosis High PCO 2 (respiratory acidosis). High HCO 3 - (compensatory alkalosis) 16 Compensated Respiratory Acidosis High PCO 2 (respiratory acidosis). High HCO 3 - (compensatory alkalosis) pH is closer to normal, hence compensated. 17 Diuretic-induced hypokalemia with Metabolic Alkalosis Low K + High HCO 3 - 18 Membranous Nephropathy, Albuminuria Albuminuria 19 Multiple Myeloma Hypercalcemia 20 Myocardial Infarct Increased CK, AST, LDH. Creatinine Kinase MB (CK-MB) is most useful isoenzyme for diagnosis. 21 Metastatic Breast Cancer High Alk.Phos, AST, ALT. Normal albumin Cancer metastases to bone. 22 Obstructive Jaundice, caused by Carcinoma of Head of Pancreas Very high alk.phos, indicated of cholestasis. www.brain101.info 8 High AST and ALT High Bilirubin 23 Acute Viral Hepatitis Very high AST and ALT Moderate Alk.Phos. High gamma-GT Dark color of urine is due to conjugated bilirubin. Patient should recover from the hepatitis without consequences. 24 NIDDM glucose tolerance test 25 Diabetic Ketacidosis Odor on breath 26 Nocturnal Hypoglycemia in a Diabetic Low blood sugar at night after taking insulin. Measuring blood sugar during a hypoglycemia attack isn't practical. Can measure catecholamines in the blood to estabolish diagnosis. Treatment: adjust insulin levels. 27 Osteomalacia Low Ca +2 Low adjusted Ca +2 High alk.phos. would be found if ordered, to establish diagnosis. 28 Paraneoplastic Hypercalcemia High Ca +2 Low phosphate Normal PTH Normal PTH was found on further investigation, so they took X-rays looking for metastases. 29 Hypomagnesemia with scondary Hypoparathyroidism. Low Mg +2 Low PTH secondarily Mg +2 is required for PTH secretion! 30 Paget's Disease of Bone High alk.phos. 31 Lactotrope Adenoma with Pan- hypopituitarism High Prolactin The rest of the pituitary hormones are low Compression atrophy of the rest of the pituitary. 32 Possible Growth Deficiency Repeat test. GH levels can fluctuate, and erroneous results can happen after a single random measurement. 33 Cystic Cold Thyroid Nodule in woman on ERT. High T 4 Low TSH Taking estrogen ------> TBG is higher ------> T 4 baseline must be higher to compensate for the increased TBG. Perform fine-needle aspiration biopsy to evaluate the nodule. 34 Hypothyroidism 35 Thyrotoxicosis Order free T 3 and T 4 tests to evaluate status. 36 Acute Adrenal Cortical Failure Low Na + , High K + Low Na + and high K + result from no aldosterone. www.brain101.info 9 Hypotension Acidosis Acidosis is secondary to the hyperkalemia. Give ACTH (Synachten) test to confirm diagnosis. 37 Auto-immune Adrenalitis (Addison's Disease) Low Na + , High K + Hypotension Acidosis 38 ACTH-Secreting Carcinoma of Lung, Cushing's Disease Carcinoid tumor. 39 Polycystic Ovary Syndrome High testosterone, High LH, low FSH 40 Chronic Malnutrition Vitamin-K malabsorption 41 Pernicious Anemia with Hypothyroidism 42 Total Parenteral Nutrition, secondary hyperglycemia High blood sugar Can see hyperglycemia in patients who are on TPN, due to poor or no stimulation of insulin release. 43 MVA with tissue injury High K + High K + is released from tissues, from tissue injury. Measure creatinine kinase to document muscle cell necrosis (rhabdomyolsysis). 44 Osteomalacia High alk.phos. Low Ca +2 , low Vit-D Most likely caused y malnutrition, or malabsorption of Vitamin-D. 45 Iron-Deficiency Anemia Low Fe +2 Low transferrin saturation (high binding capacity) Low ferritin. 46 Wilson's Disease Liver failure. 47 Digoxin Toxicity, Renal Failure Patient had elevated serum urea due to pre-renal failure, secondary to heart failure. 48 Salicylate Poisoning Low HCO 3 - High anion gap Metabolic Acidosis with Respiratory Alkalosis. Anion gap is increased because it is metabolic acidosis. 49 Lead Poisoning Measure protoporphyrin in blood cells to confirm diagnosis. 50 Alcoholism There is no lab test that is specific for alcoholism. gamma-GT comes close but is not diagnostic. www.brain101.info 10 51 Diabetic Hypoglycemia after drinking alcohol Patient was hypoglycemia, due to mixing alcohol with insulin. Treat with IV glucose. 52 Hyperlipidemia Low electrolytes High amylase High triglycerides Pseudohyponatremia: Low Na + due to abnormally low water-content of plasma (i.e. plasma had way too much lipid in it). Genetic disorder involves Apolipoprotein-B Patient is at risking of forming a volvulus. 53 Obesity, hyperlipidemia, NIDDM, Alcohol High cholesterol, lipids High glucose High gamma-GT Treat with dietary measures. Man is at increased risk for coronary artery disease. 54 Heterozygous Familial Hypercholesterolemia High fasting cholesterol, Normal lipids, Low HDL Hypercholesterolemia is also found in patients with Hypothyroidism. 55 Pheochromocytoma VMA in urine. HTN 56 ACTH-secreting tumor, Cushing's Disease High HCO 3 - Low K + , High Na + High creatinine Metabolic Alkalosis secondary to hypokalemia, from increased aldosterone activity. Probably comes from oat-cell carcinoma of lung. 57 Alcoholic Liver Disease, Hepatoma High liver enzymes High gamma-GT alpha-Fetoprotein was normal in this case (it's usually elevated) Can also measure Carcinoembryonic Antigen (CEA) 58 Thyroid Carcinoma Severe headache High Ca +2 59 Septic Arthritis posing as Gout Uric acid came back normal. Give antibiotics to treat septic arthritis. 60 Hemolysis, Tissue Damage High LDH, high CK Low haptoglobin LDH, CK = damage to: muscle, liver, or erythrocytes. 61 Cystic Fibrosis High Cl - in sweat 62 Rh-Incompatibility Disease Measure bilirubin in amniotic fluid to diagnose erythroblastosis fetalis. High bilirubin would indicate hemolysis in the fetal blood. 63 Pre-Eclampsia Progressive albuminuria, HTN 64 Cretinism Baby came back normal. TSH must be above 100 before follow-up test is required. 65 IRDS in premature infant www.brain101.info 1 Cranial Nerves Nerve CN Source Branches Motor Sensory Notes Olfactory I Filaments of the bipolar olfactory epithelial cells constitute the Olfactory n. Second order Olfactory nerve cell bodies located in the olfactory bulb None Smell (SVA) Multiple olfactory filaments pass through the cribriform plate to exit the anterior cranial fossa and synapse in the olfactory bulb; the olfactory tract carries the signal from the bulb to olfactory cortex of the forebrain. Optic II Ganglion layer of the retina to the forebrain. None None Vision (SSA) The course of the optic nerve is: through the optic canal to the optic chiasma, then the optic tract to the lateral geniculate body and optic radiation Oculomotor III Oculomotor nuclei of the midbrain (extraocular muscles); accessory oculomotor nucleus (nucleus of Edinger- Westphal - preganglionic parasympathetic) Superior br., Inferior br. GSE: * Superior br.: levator palpebrae, superioris m., superior rectus m. * Inferior br: medial rectus m., inferior rectus m., inferior oblique m.; GVE: ciliary m. & sphincter pupillae m. (preganglionic parasympathetic axons go to the ciliary ganglion via the parasympathetic root, postganglionic parasympathetic go from the ciliary ganglion to the eyeball via short ciliary nn.) None Passes through the superior orbital fissure to exit the middle cranial fossa Trochlear IV Trochlear nucleus of the midbrain None Superior oblique m. of the eye (GSE) None Passes through the superior orbital fissure to exit the middle cranial fossa; it is the smallest cranial nerve and the only cranial nerve to arise from dorsum of brainstem www.brain101.info 2 middle cranial fossa; it is the smallest cranial nerve and the only cranial nerve to arise from dorsum of brainstem Trigeminal V Motor root arises from the trigeminal motor nucleus in the pons (SVE); sensory part arises from the trigeminal ganglion (GSA) and projects into the pons to the primary sensory nucleus of V or more inferiorly to the nucleus of the spinal root of V (medulla and upper spinal cord) Ophthalmic, Maxillary & Mandibular divisions SVE: anterior belly of the digastric m., mylohyoid m., tensor veli palatini m., tensor tympani m.; muscles of mastication: temporalis m., masseter m., lateral pterygoid m., medial pterygoid m. Skin of the face; mucous membranes of the nasal and oral cavities; general sensation (GSA) to the anterior 2/3 of the tongue Some brs. carry pre- or postganglionic parasympathetic fibers; the trigeminal n. divides into three divisions at the trigeminal ganglion; SVE supplies muscles of 1st pharyngeal arch origin Ophthalmic division of the trigeminal n. V1 Trigeminal ganglion Meningeal br., lacrimal n., frontal n., nasociliary n. None (GSA) skin of the forehead, upper eyelid and nose; mucous membrane of the upper nasal cavity, frontal sinus, ethmoid air cells and sphenoid sinuses Passes through the superior orbital fissure to exit the middle cranial fossa; the lacrimal n. Receives postganglionic parasympathetic axons to the lacrimal gland from the zygomaticotemporal br. of zygomatic n. Maxillary division of the trigeminal n. V2 Trigeminal ganglion Meningeal br., posterior superior alveolar n., pharyngeal, posterior superior medial and lateral nasal brs., nasopalatine n., greater and lesser palatine nn., zygomatic n., infraorbital n. None GSA: skin of the upper lip, cheek, lower eyelid; mucous membrane of the palate; teeth and gingiva of the maxillary alveolar arch; the mucous membrane lining most of the nasal cavity; the mucous membrane lining the maxillary sinus Passes through the foramen rotundum to enter the pterygopalatine fossa; the pterygopalatine ganglion is associated with it in the pterygopalatine fossa; postganglionic parasympathetic fibers distribute with branches of the maxillary division to mucous glands of the nasal cavity and palate; the zygomatic n. & its brs. carry postganglionic parasympathetic axons to the orbit to reach the lacrimal n. and lacrimal gland www.brain101.info 3 mucous glands of the nasal cavity and palate; the zygomatic n. & its brs. carry postganglionic parasympathetic axons to the orbit to reach the lacrimal n. and lacrimal gland Mandibular division of the trigeminal n. V3 Trigeminal ganglion; motor root arises from the pons Meningeal br., medial pterygoid and lateral pterygoid nn., masseteric n., anterior and posterior deep temporal nn., buccal n., auriculotemporal n., lingual n., inferior alveolar n. SVE: mylohyoid m., anterior belly of the digastric m.; tensor tympani m., tensor veli palatini m.; muscles of mastication (temporalis, masseter, medial pterygoid and lateral pterygoid) GSA: skin of the lower lip and jaw extending superiorly above level of the ear; mucous membrane of the tongue and floor of the mouth; lower teeth and gingiva of the mandibular alveolar arch Passes through the foramen ovale to exit the middle cranial fossa; the otic ganglion is associated with the medial side of V3 below the foramen ovale; the auriculotemporal n. carries postganglionic parasympathetic axons to the parotid gland; the submandibular ganglion is associated withe the lingual n. near the submandibular gland; postganglionic parasympathetics from the submandibular ganglion supply the submandibular gland and the sublingual gland Abducens VI Pons: abducens nucleus None GSE: lateral rectus m. None Passes through the superior orbital fissure Facial VII Pons and medulla: nucleus solitarius of medulla via nervus intermedius (SVA sensory root) from geniculate ganglion; superior salivatory nucleus (GVE preganglionic parasympathetic) of pons via nervus intermedius; facial motor nucleus of pons Greater petrosal n. (preganglionic parasympathetic to pterygopalatine ganglion, postganglionic parasympathetic travels with brs. of maxillary division of V), chorda tympani (SVA taste from anterior 2/3 of the tongue; preganglionic parasympathetic to the submandibular ganglion, Stapedius m., stylohyoid m., posterior belly of digastric m., muscles of facial expression; secretomotor to lacrimal, submandibular, sublingual, and mucous glands of the nasal and oral cavities Taste (SVA) from the anterior 2/3 of the tongue; part of the skin of the external auditory meatus Exits the posterior cranial fossa by passing into the internal acoustic meatus, goes through the facial canal; motor to muscles of facial expression exits the skull at the stylomastoid foramen www.brain101.info 4 superior salivatory nucleus (GVE preganglionic parasympathetic) of pons via nervus intermedius; facial motor nucleus of pons via motor root division of V), chorda tympani (SVA taste from anterior 2/3 of the tongue; preganglionic parasympathetic to the submandibular ganglion, postganglionic parasympathetic to the submandibular and sublingual glands), n. to stapedius, posterior auricular n., intraparotid plexus with temporal, zygomatic, buccal, marginal mandibular & cervical brs. mucous glands of the nasal and oral cavities stylomastoid foramen Vestibulocochlear VIII Pons & medulla: vestibular nuclei from the vestibular ganglion of the semicircular ducts; cochlear nuclei in the inferior cerebellar peduncle Divides within the temporal bone into vestibular and cochlear parts None Vestibular: balance/proprioception (SSA); cochlear: hearing (SSA) Auditory nerve; passes into the internal auditory meatus Glossopharyngeal IX Medulla: spinal trigeminal nucleus from the superior ganglion (GVA); nucleus solitarius from the inferior ganglion (SVA); nucleus ambiguus (GVA); inferior salivatory nucleus (GVE - preganglionic parasympathetic) Tympanic nerve to the tympanic plexus and lesser petrosal n., carotid sinus n., stylopharyngeus brs., pharyngeal brs. GSE: stylopharyngeus; GVE: secretomotor to the parotid gland (preganglionic parasympathetic via the tympanic n. to the lesser petrosal n. to the otic ganglion; postganglionic parasympathetic via the auriculotemporal n.) GVA: carotid body, carotid sinus, pharynx, middle ear; GSA: skin of the external ear; SVA: taste from the posterior 1/3 of the tongue Exits the posterior cranial fossa by passing through the jugular foramen;it may penetrate the stylopharyngeus m. Vagus X Medulla: dorsal motor nucleus (GVE preganglionic parasympathetic); inferior ganglion (GVA); nucleus ambiguus (SVE); superior ganglion (GSA); inferior ganglion(SVA) Auricular br., pharyngeal br., superior laryngeal, superior and inferior cervical cardiac brs., recurrent laryngeal n., thoracic cardiac brs., brs. to the SVE: intrinsic muscles of the larynx, pharynx (except stylopharyngeus), and palate (except tensor veli palatini); GVE: smooth muscle of the respiratory tree & gut (proximal to the GSA: skin of the external auditory meatus; GVA: viscera of head, neck, thorax & abdomen proximal to the left colic flexure; SVA: taste from the epiglottis Passes through the jugular foramen to exit the posterior cranial fossa; vagus means "wanderer" in reference to its extensive distribution to the body cavities www.brain101.info 5 motor nucleus (GVE preganglionic parasympathetic); inferior ganglion (GVA); nucleus ambiguus (SVE); superior ganglion (GSA); inferior ganglion(SVA) pharyngeal br., superior laryngeal, superior and inferior cervical cardiac brs., recurrent laryngeal n., thoracic cardiac brs., brs. to the pulmonary plexus, brs. to the esophageal plexus, anterior and posterior vagal trunks muscles of the larynx, pharynx (except stylopharyngeus), and palate (except tensor veli palatini); GVE: smooth muscle of the respiratory tree & gut (proximal to the left colic flexure), heart; secretomotor: mucous glands of the larynx, respiratory tree, pharynx and gut; secretomotor to digestive glands external auditory meatus; GVA: viscera of head, neck, thorax & abdomen proximal to the left colic flexure; SVA: taste from the epiglottis jugular foramen to exit the posterior cranial fossa; vagus means "wanderer" in reference to its extensive distribution to the body cavities Accessory XI Cranial root: medulla - nucleus ambiguous. Spinal root: spinal nucleus of the upper cervical spinal cord None GSE: sternocleidomastoid and trapezius mm. None Spinal root enters cranial cavity by passing through the foramen magnum. Exits skull by passing through the jugular foramen. Accessory n. is motor only; the subtrapezial plexus of nerves receives proprioceptive fibers: for the sternocleido-mastoid m. from the ventral primary rami of spinal nn. C2 and C3 - for trapezius via ventral primary rami of C3 and C4 Hypoglossal XII Medulla: hypoglossal nucleus No named branches. Branches of the ventral primary ramus of spinal nerve C1 are carried by this nerve and are not considered to be branches of the hypoglossal nerve Intrinsic and extrinsic muscles of the tongue (except the palatoglossus m.) None Exits the posterior cranial fossa by passing through the hypoglossal canal; the superior root of the ansa cervicalis travels with the hypoglossal n. for a short distance www.brain101.info 1 Medical Student’s Amnesia A transient selective loss of memory during an exam that prevents one from remembering the eponymically-named diseases discovered by old, dead doctors. Addison’s Disease 1. Primary adrenocortical deficiency Addisonian Anemia 2. Pernicious anemia (antibodies to intrinsic factor or parietal cells → ↓IF → ↓Vit B12 → megaloblastic anemia) Albright’s Syndrome 3. Polyostotic fibrous dysplasia, precocious puberty, café au lait spots, short stature, young girls Alport’s Syndrome 4. Hereditary nephritis with nerve deafness Alzheimer’s 5. Progressive dementia Argyll-Robertson Pupil 6. Loss of light reflex constriction (contralateral or bilateral) 7. “Prostitute’s Eye” – accommodates but does not react 8. Pathognomonic for 3°Syphilis 9. Lesion pretectal region of superior colliculus Arnold-Chiari Malformation 10. Cerebellar tonsil herniation through foramen magnum = see thoracolumbar meningomyelocele Barrett’s 11. Columnar metaplasia of lower esophagus (↑ risk of adenocarcinoma)- constant gastroesophageal reflux Bartter’s Syndrome 12. Hyperreninemia Becker’s Muscular Dystrophy 13. Similar to Duchenne, but less severe (mutation, not a deficiency, in dystrophin protein) Bell’s Palsy 14. CNVII palsy (entire face; recall that UMN lesion only affects lower face) Berger’s Disease 15. IgA nephropathy causing hematuria in kids, usually following infection Bernard-Soulier Disease 16. Defect in platelet adhesion (abnormally large platelets & lack of platelet-surface glycoprotein) Berry Aneurysm 17. Circle of Willis (subarachnoid bleed) Anterior Communicating artery 18. Often associated with ADPKD Bowen’s Disease 19. Carcinoma in situ on shaft of penis (↑ risk of visceral ca) [compare w/ Queyrat] Brill-Zinsser Disease 20. Recurrences of rickettsia prowazaki up to 50 yrs later Briquet’s Syndrome 21. Somatization disorder 22. Psychological: multiple physical complaints without physical pathology Broca’s Aphasia 23. Motor Aphasia (area 44 & 45) intact comprehension Brown-Sequard 24. Hemisection of cord (contralateral loss of pain & temp / ipsilateral loss of fine touch, UMN / ipsi loss of consc. Proprio) Bruton’s Disease 25. X-linked agammaglobinemia (↓ B cells) Budd-Chiari 26. Post-hepatic venous thrombosis = ab pain; hepatomegaly; ascites; portal HTN; liver failure Buerger’s Disease 27. Acute inflammation of medium and small arteries of extremities → painful ischemia → gangrene 28. Seen almost exclusively in young and middle-aged men who smoke. Burkitt’s Lymphoma 29. Small noncleaved cell lymphoma EBV 30. 8:14 translocation 31. Seen commonly in jaws, abdomen, retroperitoneal soft tissues 32. Starry sky appearance Caisson Disease 33. Nitric gas emboli Chagas’ Disease 34. Trypansoma infection - cardiomegaly with apical atrophy, achlasia Chediak-Higashi Disease 35. (AR) Phagocyte Deficiency = defect in microtubule polymerization 36. Neutropenia, albinism, cranial & peripheral neuropathy & repeated infections w/ strep & staph Conn’s Syndrome 37. Primary Aldosteronism: HTN; retain Na + & H2O; hypokalemia (causing alkalosis); ↓ renin Cori’s Disease 38. Type III Glycogenosis – Glycogen storage disease (debranching enz: amylo 1,6 glucosidase def. ↑ Glycogen) Creutzfeldt-Jakob 39. Prion infection → cerebellar & cerebral degeneration Crigler-Najjar Syndrome 40. Congenital hyperbilirubinemia (unconjugated) 41. Glucuronyl transferase deficiency. Can progress to Kernicterus 42. Less severe form will respond to Phenobarbital therapy Crohn’s 43. IBD; ileocecum, transmural, skip lesions, cobblestones, lymphocytic infiltrate, granulomas 44. (contrast to UC: limited to colon, mucosa & submucosa, crypt abscesses, pseudopolyps, ↑ colon cancer risk) 45. Clinically: ab pain & diarrhea; fever; malabsorption; fistulae b/t intestinal loops & abd structures Curling’s Ulcer 46. Acute gastric ulcer associated with severe burns Cushing’s 47. Disease: Hypercorticism 2° to ↑ ACTH from pituitary (basophilic adenoma) 48. Syndrome: hypercorticism of all other causes (1° adrenal or ectopic) 49. - moon face; buffalo hump; purple striae; hirsutism; HTN; hyperglycemia www.brain101.info 2 Cushing’s Ulcer 50. Acute gastric ulcer associated with CNS trauma de Quervain’s Thyroiditis 51. Self-limiting focal destruction (subacute thyroiditis) DiGeorge’s Syndrome 52. Failure of 3 rd & 4 th pharyngeal pouches formation: Thymus & Parathyroid 53. Thymic hypoplasia → T-cell deficiency 54. Hypoparathyroidism  Tetany Down’s Syndrome 55. Trisomy 21 or translocation – Simian Crease Dressler’s Syndrome 56. Post-MI Fibrinous Pericarditis autoimmune Dubin-Johnson Syndrome 57. Congenital hyperbilirubinemia (conjugated) = bilirubin transposrt is defective not conjugation 58. Striking brown-to-black discoloration of the liver Duchenne Muscular Dystrophy 59. Deficiency of dystrophin protein → MD X-linked recessive Edwards’ Syndrome 60. Trisomy 18 61. Rocker-bottom feet, low ears, small lower jaw, heart disease Ehler’s-Danlos 62. Defective collagen Eisenmenger’s Complex 63. Late cyanotic shunt (R→L) pulmonary HTN & RVH 2° to long-standing VSD, ASD, or PDA Erb-Duchenne Palsy 64. Trauma to superior trunk of brachial plexus Waiter’s Tip Ewing Sarcoma 65. Malignant undifferentiated round cell tumor of bone in boys <15yoa - t11;22 Eyrthroplasia of Queyrat 66. Carcinoma in situ on glans penis Fanconi’s Syndrome 67. Impaired proximal tubular reabsorption 2° to lead poisoning or Tetracycline (glycosuria, hyperphosphaturia, aminoaciduria, systemic acidosis) Felty’s Syndrome 68. Rheumatoid arthritis, neutropenia, splenomegaly Gardner’s Syndrome 69. AD = adenomatous polyps of colon, osteomas & soft tissue tumors Gaucher’s Disease 70. Lysosomal Storage Disease glucocerebrosidase deficiency – glucocerebroside accumulation 71. Hepatosplenomegaly, femoral head & long bone erosion, anemia Gilbert’s Syndrome 72. Benign congenital hyperbilirubinemia (unconjugated) = ↓d glucuronyl transferase activity Glanzmann’s Thrombasthenia 73. Defective glycoproteins on platelets = deficient platelet aggregation Goodpasture’s 74. Autoimmune: ab’s to glomerular & alveolar basement membranes. Seen in men in their 20’s Grave’s Disease 75. Autoimmune hyperthyroidism (TSI): IgG Ab reactive w/ TSH receptors. Low TSH & TRH – High T3 / T4 Guillain-Barre 76. Polyneuritis following viral infection/ autoimmune (ascending muscle weakness & paralysis; usually self-limiting) Hamman-Rich Syndrome 77. Idiopathic pulmonary fibrosis. Can see honey comb lung. Hand-Schuller-Christian 78. Chronic progressive histiocytosis Hashimoto’s Thyroiditis 79. Autoimmune hypothyroidism. May have transient hyperthyroidism. Low T3 /T4 & High TSH Hashitoxicosis 80. Initial hyperthyroidism in Hashimoto’s Thyroiditis that precedes hypothyroidism Henoch-Schonlein purpura 81. Hypersensivity vasculitis = allergic purpura. Lesions have the same age. 82. Hemmorhagic urticaria (with fever, arthralgias, GI & renal involvement) 83. Associated with upper respiratory infections Hirschprung’s Disease 84. Aganglionic megacolon Horner’s Syndrome 85. Ptosis, miosis, anhidrosis (lesion of cervical sympathetic nerves often 2° to a Pancoast tumor) Huntington’s (Chromosome 4) 86. AD: Progressive degeneration of caudate nucleus, putamen (striatum) & frontal cortex ↓ GABA Jacksonian Seizures 87. Epileptic events originating in the primary motor cortex (area 4) Job’s Syndrome 1. Immune deficiency: neutrophils fail to respond to chemotactic stimuli 2. Defective neutrophilic chemotactic response = repeated infections 3. Commonly seen in light-skinned, red-haired girls 88. ↑’d IgE levels Kaposi Sarcoma 89. Malignant vascular tumor (HHV8 in homosexual men) Kartagener’s Syndrome 90. Immotile cilia 2° to defective dynein arms infection, situs inversus, sterility Kawasaki Disease 91. Mucocutaneous lymph node syndrome in kids (acute necrotizing vasculitis of lips, oral mucosa) Klinefelter’s Syndrome 92. 47, XXY: Long arms, Sterile, Hypogonadism Kluver-Bucy 93. Bilateral lesions of amygdala (hypersexuality; oral behavior) Krukenberg Tumor 94. Adenocarcinoma with signet-ring cells (typically originating from the stomach) metastases to 95. the ovaries Laennec’s Cirrhosis 96. Alcoholic cirrhosis www.brain101.info 3 Lesch-Nyhan 97. HGPRT deficiency 98. Gout, retardation, self-mutilation Letterer-Siwe 99. Acute disseminated Langerhans’ cell histiocytosis Libman-Sacks 100. Endocarditis with small vegetations on valve leaflets 101. Associated with SLE Lou Gehrig’s 102. Amyotrophic Lateral Sclerosis degeneration of upper & lower motor neurons Mallory-Weis Syndrome 103. Bleeding from esophagogastric lacerations 2° to wretching (alcoholics) Marfan’s 104. Connective tissue defect: defective Fibrillin gene Dissecting aortic aneurysm, subluxation of lenses McArdle’s Disease 105. Type V Glycogenosis - Glycogen storage disease (muscle phosphorylase deficiency = ↑ Glycogen) Meckel’s Diverticulum 106. Rule of 2’s: 2 inches long, 2 feet from the ileocecum, in 2% of the population 107. Embryonic duct origin; may have ectopic tissue: gastric/pancreatic remnant of vitteline duct/yolk stalk Meig’s Syndrome 108. Triad: ovarian fibroma, ascites, hydrothorax – associated w/ fibroma of ovaries Menetrier’s Disease 109. Giant hypertrophic gastritis (enlarged rugae; plasma protein loss) Monckeberg’s Arteriosclerosis 110. Calcification of the media (usually radial & ulnar aa.) Munchausen Syndrome 111. Factitious disorder (consciously creates symptoms, but doesn’t know why) Nelson’s Syndrome 112. 1° Adrenal Cushings → surgical removal of adrenals → loss of negative feedback to pituitary → Pituitary Adenoma Niemann-Pick 113. Lysosomal Storage Disease (sphingomyelinase deficiency – sphingomyelin accumulation) 114. “Foamy histiocytes” Osler-Weber-Rendu Syndrome 115. Hereditary Hemorrhagic Telangiectasia. Seen in the Mormon’s of Utah. Paget’s Disease 116. Abnormal bone architecture (thickened, numerous fractures → pain) Pancoast Tumor 117. Bronchogenic tumor with superior sulcus involvement → Horner’s Syndrome Parkinson’s 118. Dopamine depletion in nigrostriatal tracts Peutz-Jegher’s Syndrome (AD) 119. Melanin pigmentation of lips, mouth, hand, genitalia + hamartomatous polyps of small intestine Peyronie’s Disease 120. Subcutaneous fibrosis of dorsum of penis Pick’s Disease – 2 Different Diseases - 121. 1. Progressive dementia similar to Alzheimer’s 122. 1. Constrictive pericarditis – sequel to mediastinal tuberculosis 123. Calcium-frosting, unyielding layer – heart chambers may be unable to dilate to receive blood during diastole Plummer’s Syndrome 124. Hyperthyroidism, nodular goiter, absence of eye signs (Plummer’s = Grave’s - eye signs) Plummer-Vinson 125. Esophageal webs & iron-deficiency anemia, spoon-shaped nails, ↑ SCCA of esophagus Pompe’s Disease 126. Type II Glycogenosis – Glycogen storage disease → cardiomegaly (α 1,4 Glucosidase deficiency: ↑ Glycogen) Pott’s Disease 127. Tuberculous osteomyelitis of the vertebrae Potter’s Complex 128. Renal agenesis → oligohydramnios → hypoplastic lungs, defects in extremities Raynaud’s 129. Disease: recurrent vasospasm in extremities = seen in young, healthy women 130. Phenomenon: 2° to underlying disease (SLE or scleroderma) Reiter’s Syndrome 131. Urethritis, conjunctivitis, arthritis non-infectious (but often follows infections), HLA-B27, polyarticular Reye’s Syndrome 132. Microvesicular fatty liver change & encephalopathy 133. 2° to aspirin ingestion in children following viral illness, especially VZV Riedel’s Thyroiditis 134. Idiopathic fibrous replacement of thyroid Rotor Syndrome 135. Congenital hyperbilirubinemia (conjugated) 136. Similar to Dubin-Johnson, but no discoloration of the liver Sezary Syndrome 137. Leukemic form of cutaneous T-cell lymphoma (mycosis fungoides) Shaver’s Disease 138. Aluminum inhalation → lung fibrosis Sheehan’s Syndrome 139. Postpartum pituitary necrosis = hemorrhage & shock usually occurred during delivery Shy-Drager 140. Parkinsonism with autonomic dysfunction & orthostatic hypotension Simmond’s Disease 141. Pituitary cachexia – can occur from either pituitary tumors or Sheehan’s Sipple’s Syndrome 142. MEN type IIa = pheochromocytoma, thyroid medullary CA, hyperparathyroidism Sjogren’s Syndrome 143. Triad: dry eyes, dry mouth, arthritis ↑ risk of B-cell lymphoma Spitz Nevus 144. Juvenile melanoma (always benign) Stein-Leventhal 145. Polycystic ovary: see amenorrhea; infertility; obesity; hirsutism = ↑↑LH secretion Stevens-Johnson Syndrome 146. Erythema multiforme, fever, malaise, mucosal ulceration (often 2° to infection = mycoplasma or sulfa drugs) www.brain101.info 4 Still’s Disease 147. Juvenile rheumatoid arthritis (absence of rheumatoid factor) Takayasu’s arteritis 148. Aortic arch syndrome 149. Loss of carotid, radial or ulnar pulses = pulseless disease. Night sweats. 150. Common in young Asian females Tay-Sachs (AR) 151. Gangliosidosis (hexosaminidase A deficiency → GM2 ganglioside) Cherry Red Spots of the Macula Tetralogy of Fallot 152. 1.VSD, 2.overriding aorta, 3.pulmonary artery stenosis, 4.right ventricular hypertrophy Tourette’s Syndrome 153. Involuntary actions, both motor and vocal Txt w/ Pimozide Turcot’s Syndrome 154. Colon adenomatous polyps plus CNS tumors Turner’s Syndrome 155. 45, XO = most common cause of Primary Amenorrhea. No Barr body on buccal smear. Vincent’s Infection 156. “Trench mouth” – acute necrotizing ulcerative gingivitis due to Fusobacterium Von Gierke’s Disease 157. Type I Glycogenosis – Glycogen storage disease (G6Ptase deficiency) – Glycogen accumulaiton Von Hippel-Lindau 158. Hemangioma (or hemangioblastoma) = cerebellum, brain stem, & retina 159. Adenomas of the viscera, especially ↑ Renal Cell Carcinoma 160. Chromosome 3p Von Recklinghausen’s 161. Neurofibromatosis & café au lait spots & Lisch nodules (Chromosome 17) Von Recklinghausen’s Disease of Bone 162. Osteitis fibrosa cystica (“brown tumor”) 2° to hyperparathyroidism = osteoclastic resorption w/ 163. fibrous replacement Von Willebrand’s Disease (AD) 164. Defect in platelet adhesion 2° to deficiency in vWF. ↑aPPT, ↑ Bleed time Waldenstrom’s macroglobinemia 165. Proliferation of IgM-producing lymphoid cells in men 50-70 yoa; PAS(+) Dutcher bodies Wallenberg’s Syndrome 166. Posterior Inferior Cerebellar Artery (PICA) thrombosis “Medullary Syndrome” 167. Ipsilateral: ataxia, facial pain & temp; Contralateral: body pain & temp Waterhouse-Friderichsen 168. Adrenal insufficiency 2° to DIC 169. DIC 2° to meningiococcemia Weber’s Syndrome 170. Paramedian Infarct of Midbrain 171. Ipsilateral: mydriasis; Contralateral: UMN paralysis (lower face & body) Wegener’s Granulomatosis 172. Necrotizing granulomatous vasculitis of paranasal sinuses, lungs, kidneys, etc. Weil’s Disease 173. Icteric Leptospirosis non-icteric prgresses to renal failure & myocarditis 174. Dark field microscopy for dx Wermer’s Syndrome 175. MEN type I = thyroid, parathyroid, adrenal cortex, pancreatic islets, pituitary Wernicke’s Aphasia 176. Sensory Aphasia impaired comprehension Wernicke-Korsakoff Syndrome 177. Thiamine deficiency in alcoholics; bilateral mamillary bodies (mediodorsal nucleua) (confusion, ataxia, ophthalmoplegia) Whipple’s Disease 178. Malabsorption syndrome (with bacteria-laden macrophages) & polyarthritis Wilson’s Disease 179. Hepatolenticular degeneration (copper accumulation [Txt w/ Penicillamine ] & decrease in ceruloplasmin) 180. Mallory Bodies in the Liver & also w/ alcoholic hepatitis & Hyaline change 181. Chromosome 13 Wiskott-Aldrich Syndrome 182. Immunodeficiency: combined B- &T-cell deficiency (thrombocytopenia & eczema) 183. ↓ IgM w/ ↑ IgA Wolff-Chaikoff Effect 184. High iodine level (−)’s thyroid hormone synthesis Zenker’s Diverticulum 185. Esophageal; cricopharyngeal muscles above UES Zollinger-Ellison 186. Gastrin-secreting tumor of pancreas (or intestine) → ↑ acid → recurrent ulcers Roger’s Disease 187. Interventricular septal defect Barlow’s Syndrome 188. Floppy vale syndrome – women b/t 20-40 yoa Bracht-Wachter Lesions 189. Minute abscesses found in subacute bacterial endocarditis Lutembacher’s Syndrome 190. Combination of septum secundum atrial septal defect w/ mitral stenosis Schmidt’s Syndrome 191. Autoimmnue thyroid Disease (Hashimoto’s ) & insulin-dependent diabetes Hallmark Findings Albumino-Cytologic Dissociation 192. Guillain-Barre (markedly increased protein in CSF with only modest increase in cell count) Antiplatelet Antibodies 193. Idiopathic thrombocytopenic purpura Arachnodactyly 194. Marfan’s Aschoff Bodies 195. Rheumatic fever www.brain101.info 5 Auer Rods 196. Acute promyelocytic leukemia (AML type M3) Autosplenectomy 197. Sickle cell anemia: switch a glu  val in β chain 198. Low O2 ↑ sickling 199. Aplastic crisis w/ B19 (Parvovirus ssDNA) infection 200. Salmonella osteomyelitis 201. Vaso-occlusive painful crisises 202. Hydroxyurea as Txt (↑ Hb F ) & Bone marrow transplant Babinski 203. UMN lesion Basophilic Stippling of RBCs 204. Lead poisoning Bence Jones Protein 205. Multiple myeloma free light chains (either kappa or lambda) 206. Waldenstrom’s macroglobinemia Birbeck Granules 207. Histiocytosis X (eosinophilic granuloma) Blue Bloater 208. Chronic Bronchitis (at least 3 months for at least 2 years of ecessive mucus secretion & chronic recurrent productive cough) Boot-Shaped Heart 209. Tetralogy of Fallot Bouchard’s Nodes 210. Osteoarthritis (Proximal IP joint of the fingers) Boutonniere’s Deformity 211. Rheumatoid arthritis flex proximal & extend distal IP joints Brown Tumor 212. Hyperparathyroidism Brushfield Spots 213. Down’s Call-Exner Bodies 214. Granulosa cell tumor: associated w/ endometrial hyperplasia & carcinoma 215. Granuloma-Theca cell tumor Cardiomegaly with Apical Atrophy 216. Chagas’ Disease Chancre 217. 1° Syphilis Chancroid 218. Haemophilus ducreyi Charcot Triad 219. Multiple sclerosis = nystagmus, intention tremor, scanning speech Charcot-Leyden Crystals 220. Bronchial asthma Cheyne-Stokes Breathing 221. Cerebral lesion Chocolate Cysts 222. Endometriosis Chvostek’s Sign 223. Hypocalcemia facial spasm in tetany Clue Cells 224. Gardnerella vaginitis Codman’s Triangle 225. Osteosarcoma Cold Agglutinins 226. Mycoplasma pneumoniae 227. Infectious mononucleosis Condyloma Lata 228. 2° Syphilis 229. New coffee flavor at Bagel & Bagel Cotton Wool Spots 230. HTN 231. Aka, cytoid bodies seen w/ SLE (yellowish cotton wool fundal lesions) Councilman Bodies 232. Dying hepatocytes – HepB Crescents In Bowman’s Capsule 233. Rapidly progressive (crescentic glomerulonephritis) Currant-Jelly Sputum 234. Klebsiella Curschmann’s Spirals 235. Bronchial asthma Depigmentation Of Substantia Nigra 236. Parkinson’s Donovan Bodies 237. Granuloma inguinale (STD) Eburnation 238. Osteoarthritis (polished, ivory-like appearance of bone) Ectopia Lentis 239. Marfan’s Erythema Chronicum Migrans 240. Lyme Disease Fatty Liver 241. Alcoholism Ferruginous Bodies 242. Asbestosis - & Iron laden Ghon Focus / Complex 243. Tuberculosis (1° & 2°, respectively) Glitter Cells 244. Acute Pyelonephritis Gower’s Maneuver 245. Duchenne’s MD use of arms to stand Heberden’s Nodes 246. Osteoarthritis (Distal IP joint of the fingers) www.brain101.info 6 Heinz Bodies 247. G6PDH Deficiency Heterophil Antibodies 248. Infectious mononucleosis (EBV) Hirano Bodies 249. Alzheimer’s Hypersegmented PMNs 250. Megaloblastic anemia Hypochromic Microcytic RBCs 251. Iron-deficiency anemia or β Thalassemia Jarisch-Herxheimer Reaction 252. Syphilis over-aggressive treatment of an asymptomatic pt. that causes symptoms 2° to rapid lysis Joint Mice 253. Osteoarthritis (fractured osteophytes) Kaussmaul Breathing 254. Acidosis / Diabetic Ketoacidosis Keratin Pearls 255. Squamous Cell CA of skin Actinic Keratosis is a precursor Keyser-Fleischer Ring 256. Wilson’s Kimmelstiel-Wilson Nodules 257. Diabetic nephropathy: Nodular Glomerulosclerosis nodules of mesangial matrix Koilocytes 258. HPV 6 & 11 (condyloma acuminatum - benign) and HPV 16 & 18 (malignant association) Koplik Spots 259. Measles Lewy Bodies 260. Parkinson’s (eosinophilic inclusions in damaged substantia nigra cells) Lines of Zahn 261. Arterial thrombus Lisch Nodules 262. Neurofibromatosis (von Recklinhausen’s disease) = pigmented iris hamartomas Lumpy-Bumpy IF Glomeruli 263. Poststreptococcal glomerulonephritis – prototype of nephritic syndrome Mallory Bodies 264. Alcoholic hepatitis McBurney’s Sign 265. Appendicitis (McBurney’s Point is 2/3 of the way from the umbilicus to anterior superior iliac spine) Michealis-Gutmann Bodies 266. Malakoplakia lesion on bladder due to macros & calcospherites (M-G Bodies): usually due to E. Coli Monoclonal Antibody Spike 267. Multiple myeloma this is called the M protein (usually IgG or IgA) 268. MGUS Myxedema 269. Hypothyroidism Negri Bodies 270. Rabies Neuritic Plaques 271. Alzheimer’s Neurofibrillary Tangles 272. Alzheimer’s Non-pitting Edema 273. Myxedema 274. Anthrax Toxin Notching of Ribs 275. Coarctation of Aorta Nutmeg Liver 276. CHF = causing congested liver Owls Eye Cells 277. CMV 278. Reed Sternburg Cells (Hodkins Lymphoma) 279. Aschoff cells seen w/ Rheumatic Fever Painless Jaundice 280. Pancreatic CA (head) Pannus 281. Rheumatoid arthritis, also see morning stiffnes that ↓ w/ joint use, HLA-DR4 Pautrier’s Microabscesses 282. Mycosis fungoides (cutaneous T-cell lymphoma), Sezary Philadelphia Chromosome 283. CML Pick Bodies 284. Pick’s Disease 2 types of COPD 285. Pink Puffer – Type A: Emphysema 286. Blue Bloater – Type B: Bronchitis 287. Emphysema Centroacinar – smoking Panacinar - α1-antitrypsin deficiency Podagra 288. Gout (MP joint of hallux) Port-Wine Stain 289. Hemangioma Posterior Anterior Drawer Sign 290. Tearing of the ACL Psammoma Bodies 291. Papillary adenocarcinoma of the thyroid 292. Serous papillary cystadenocarcinoma of the ovary 293. Meningioma 294. Mesothelioma Pseudohypertrophy 295. Seen w/ Duchenne muscular dystrophy @ the claf muscles, due to ↑ fat Punched-Out Bone Lesions 296. Multiple myeloma www.brain101.info 7 Rash on Palms & Soles 297. 2° Syphilis 298. RMSF 299. Coxsackie virus infection: Hand-Foot-Mouth Disease Red Morning Urine 300. Paroxysmal nocturnal hemoglobinuria. You would use Ham’s test to confirm. Reed-Sternberg Cells 301. Hodgkin’s Disease Reid Index Increased 302. Chronic bronchitis = ↑d ratio of bronchial gland to bronchial wall thickness Reinke Crystals 303. Leydig cell tumor Rouleaux Formation 304. Multiple myeloma RBC’s stacked as poker chips S3 Heart Sound 305. L→R Shunt (VSD, PDA, ASD) 306. Mitral Regurg 307. LV Failure S4 Heart Sound 308. Pulmonary Stenosis 309. Pulmonary HTN Schwartzman Reaction 310. Neisseria meningitidis impressive rash with bugs Smith Antigen 311. SLE (also anti-dsDNA) 312. Malar Rash, Wire loop kidney lesions, Joint pain, False (+) syphilis test (VDRL) 313. 90% 14-45 yo females 314. also seen w/ use of INH; Procainamide; Hydralazine = SLE-like syndrome Soap Bubble on X-Ray 315. Giant cell tumor of bone Spike & Dome Glomeruli 316. Membranous glomerulonephritis = Nephrotic syndrome 317. Spike = basement membrane material & Dome = immune complex deposits (IgG orC3) String Sign on X-ray 318. Crohn’s bowel wall thickening Target Cells 319. Thalassemia in α Thalassemia w/ no α gene: Hydrops Fetalis & Intrauterine death associations = HbBarts Tendinous Xanthomas 320. Familial Hypercholesterolemia Thyroidization of Kidney 321. Chronic pyelonephritis Tophi 322. Gout Tram-Track Glomeruli 323. Membranoproliferative GN: Nephritic syndrome – basement membrane is duplicated into 2 layers Trousseau’s Sign 324. Visceral ca, classically pancreatic (migratory thrombophlebitis) 325. Hypocalcemia (carpal spasm) 326. These are two entirely different disease processes and different signs, but they unfortunately have the same name. Virchow’s Node 327. Supraclavicular node enlargement by metastatic carcinoma of the stomach Warthin-Finkeldey Giant Cells 328. Measles WBC Casts 329. Pyelonephritis Whipple’s Triad 330. CNS disfunction – Hypoglycemic episodes – glu injection reverses CNS Sympt’s Wire Loop Glomeruli 331. Lupus nephropathy, type IV (diffuse proliferative form) ↑ AFP in amniotic fluid or mother’s serum 332. Spina Bifida 333. Anencephaly ↑ Uric Acid 334. Gout 335. Lesch Nyhan 336. Myeloproliferative Disorders 337. Diuretics (Loop & Thiazides) ↓ FEV1/FVC 338. COPD “Ground Glass” on chest x-ray (Hyaline) 339. Due to Pneumocystis carinii 340. Seen w/ Atelectasia Honey Combing of the lung 341. Seen w/ Asbestosis (a restrictive lung disease) Crescents 342. Goodpastures syndrome (pneumonia w/ hemoptysis & rapidly progressive glomerulonephritis) Linear Ig Deposits 343. Goodpastures syndrome 45 Degree Branch Points 344. Aspergillosis PAS(+) Dutcher Bodies 345. Waldenstrom’s Macroglobulinemia = ↑IgM = Hyperviscosity “Ground Glass” in Abdomen(Hyaline) 346. Seen in the hepatocytes of healthy carriers of HBsAg in liver biopsies “Signet Ring” Cells 347. Cells that replace the ovaries, due to Krukenberg’s tumor that has metastasized from the stomach www.brain101.info 8 Ground Glass Appearance (Hyaline) 348. Seen w/ Progressive Multifocal Leukoencephalopathy oligodendrocytes 349. Nuclei seen in Papillary CA of the thyroid (malignant) Congo Red 350. Shows amyloid deposition in plaques & vascular walls Meningiomas & Progesterone 351. Some meningiomas have Progesterone receptors = rapid growth in pregnancy can occur Tuberous Sclerosis Triad 352. Seizures; Mental retardation; Leukoderma (congenital facial white spots or macules): angiofibromas Cowdry A Inclusions 353. Seen w/ Herpes Simplex Encephalitis – in oligodendroglia Devic’s Syndrome 354. “Neuromyelitis Optica” 355. A variant of multiple sclerosis: rapid demyelination of the optic nerve & spinal cord w/ paraplegia c-erb B2 356. Breast Cancer association Foster-Kennedy Syndrome 357. A tumor causing blindness & loss of smell w/ papilloedema Hoffman’s Sign 358. Flicking of the middle finger’s nail Red Nucleus Destruction 359. Intention tremors of the arm Ventral Spinocerebellar tr. 360. Unconscious proprioception of lower extremities Dorsal Spinocerebellar tr. 361. Unconscious prorpioception & fine motor movements Cuneocerebellar tr. 362. Unconscious proprioception & fine motor movements of upper extremities Dorsal Column 363. Conscious proprioception of the body Lateral Spinothalamic tr. 364. Pain & Temperature sensation Ventral Spinothalamic tr. 365. Light touch perception SVA 366. Taste & Smell GSE 367. Muscles of the eye & of the tongue SSA 368. Vision; Hearing; Equilibrium GVA 369. Sensation of tongue; soft palate. Carotid Body & Sinus innervation GVE 370. Edinger Westphal = parasympathetic eye innervation 371. Gland innervation = secretions 372. Viscera GSA 373. Pain & temperature of face 374. Sensation of external ear SVE 375. Innervation of muscles of masticaiton, facial expressions, larynx & pharynx LMN Lesion 376. Werndig Hoffman (progressive infantile muscular atrophy) 377. Poliomyelitis Sensory Pathway Lesion 378. Subacute Combined Degeneration = Friedrich’s Ataxia = B12 deficiency 379. Tabes Dorsalis (Neurosyphilis) Both UMN & LMN Lesion 380. ALS = Lou Gherig’s Disease Both Sensory & Motor Lesion 381. Brown Sequard 382. Anterior Spinal artery Occlusion Suprachiasmatic Nucleus 383. Controls circadian rhythm Ventromedial Nucleus 384. Satiety center. Savage behavior & obesity when lesioned Lateral Nucleus 385. Induces eating. Starvation when lesioned Arcuate Nucleus 386. Releases PIF (dopa-ergic neurons) Mamillary Body 387. Can have hemorrhages as seen in Wernicke’s Encephalopathy Acanthocytes 388. RBSc w/ spiny projections. Seen in Abetalipoproteinemia. Most Common… 1 o Tumor arising from bone in adults 389. Osteosarcoma Adrenal Medullary Tumor – Adults 390. Pheochromocytoma: 5 P’s: ↑ Pressure; Pain (Headache); Perspiration; Palpitations; Pallor/Diaphoresis Adrenal Medullary Tumor – Children 391. Neuroblastoma Agent of severe viral encephalitis 392. Herpes simplex www.brain101.info 9 Aggressive lung tumor 393. Small cell or oat cell Associated with gallstones 394. Adenocarinoma Bacterial Meningitis – adults 395. Strep pneumoniae & in young adults = Neisseria meningitidis Bacterial Meningitis – elderly 396. Neisseria meningitidis Bacterial Meningitis – newborns 397. E. coli / Group BStrep. Bacterial Meningitis – toddlers 398. Hib Benign epithelial tumor of oral mucosa 399. Papilloma Benign fallopian tube tumor 400. Adenomatoid Benign ovarian tumor 401. Mature(Native) Teratoma = benign dermatoid Benign tumor of soft tissue 402. Lipoma Benign tumor of the breast <25yoa 403. Fibroadenoma Benign tumor of the liver 404. Hemangioma Benign tumor of the vulva 405. Hidroadenoma Benign uterine tumor 406. Leiomyoma: estrogen sinsitive: changes size during pregnancy & menopause Bone Tumors 407. Metasteses from Breast & Prostate Brain Tumor – Child 408. Medulloblastoma (cerebellum) Brain Tumor –Adult 409. Astrocytoma (including Glioblastoma Multiforme) then: mets, meningioma, Schwannoma Breast Carcinoma 410. Invasive Duct Carcinoma Breast Mass 411. Fibrocystic Change: premenopausic women (Carcinoma is the most common in post-menopausal women) Bug in Acute Endocarditis 412. Staph aureus Bug in debilitated, hospitalized pneumonia pt 413. Klebsiella Bug in Epiglottitis 414. Hib Bug in GI Tract 415. Bacteroides (2 nd – E. coli) Bug in IV drug user bacteremia / pneumonia 416. Staph aureus Bug in PID 417. N. Gonnorrhoeae Bug in Subacute Endocarditis 418. Strep Viridans CA of urinary collecting system 419. Transitional cell CA (assoc. w/ benzidine; βnaphthylamine; analine dyes; long term txt w/ cyclophosphamide) Cardiac 1 ry Tumor – Adults 420. Myxoma: “Ball Valve” Cardiac 1 ry Tumor – Child 421. Rhabdomyoma – associated w/ Tuberous sclerosis Cardiac Tumor – Adults 422. Metasteses Cardiomyopathy 423. Dilated (Congestive) Cardiomyopathy: Alcohol, BeriBeri, Cocaine use, Coxsackie B, Doxorubicin 424. Systolic Dysfunction Cause of 2 ry HTN 425. Renal Disease Cause of Addison’s 426. Autoimmune (2 nd – infection) Cause of breast lumps 427. CA of the breast Cause of chronic endometriosis 428. TB Cause of Congenital Adrenal Hyperplasia 429. 21-Hydroxylase Deficiency: NaCl lost & Hypotension (then, 11- NaCl retention & HTN) Cause of Cushings 430. Exogenous Steroid Therapy (then, 1 ry ACTH, Adrenal Adenoma, Ectopic ACTH) Cause of Death in Alzheimer pts 431. Pneumonia Cause of Death in Diabetics 432. MI Cause of Death in premature 433. NRDS = hyaline membrane disease Cause of Death in SLE pts. 434. Lupus Nephropathy Type IV (Diffuse Proliferative) = Renal Disease Cause of Dementia 435. Alzheimer’s www.brain101.info 10 Cause of Dementia (2 nd most common) 436. Multi-Infarct Dementia Cause of Dwarfism 437. Achondroplasia Cause of Food poisoning 438. Staph aureus Cause of Hematosalpynga 439. Ectopic pregnancy Cause of Hypoparathyroidism 440. Throidectomy Cause of Hypothyroidism 441. Corrective surgery I31 treatment Cause of Kidney infections 442. E. coli Cause of Liver disease in US 443. Alcohol consumption Cause of Malignancy in children 444. Acute leukemia Cause of Mental retardation 445. Down’s Cause of Mental retardation (2 nd most common) 446. Fragile X Cause of NaCl loss and Hypotension 447. 21 hydroxylase deficiency Cause of PID 448. N. ghonorrhea Cause of Portal cirrhosis 449. Alcohol Cause of Preventable Blindness 450. Chlamydia (serotypes A,B,Ba,C) Cause of Pulmonary HTN 451. COPD Cause of Secondary Hypertension 452. Renal disease Cause of SIADH 453. Small Cell Carcinoma of the Lung Cause of UT Obstruction in men 454. BPHyperplasia Cause Pernicious Anemia 455. Chronic atrophic gastritis = no production of intrinsic factor Chromosomal Disorder 456. Down’s Common Tumor of the Appendix 457. Carcinoid tumor: flushing; diarrhea; bronchospasm; RHeart valvular lesions 458. Txt: Methysergide (5HT antagonist) Congenital Cardiac Anomaly 459. VSD (membranous > muscular) Congenital Early Cyanosis 460. Tetralogy of Fallot =right to left shunt Coronary Artery Thrombosis 461. LAD artery: MI Demyelinating Disease 462. Multiple Sclerosis: (Charcot Triad = nystagmus, intention tremor, scanning speech) 463. Periventricular plaques w/ ↓ Oligodenrocytes 464. ↑ IgG in CSF, Optic Neuritis, MLF Syndorme = Internuclear Ophthalmoplegia, bladder incontinence Dental Tumor 465. Odontoma Dietary Deficiency 466. Iron Disease of the Breast 467. Fibrocystic disease Disseminated Opportunistic Infection in AIDS 468. CMV (Pneumocystis carinii is most common overall) Esophageal Cancer 469. SCCA Fallopian Tube Malignancy 470. AdenoCA Fatal Genetic Defect in Caucasians 471. Cystic Fibrosis (chromosome 7q) Female Tumor 472. Leimyoma Form of Amyloidosis 473. Immunologic (Bence Jones protein in multiple myeloma is also called the Amyloid Light Chain) Form of Tularemia 474. Ulceroglandular Germ Cell Tumor of Testes 475. Seminoma (analogous to dysgerminoma of ovaries) Gynecological Malignancy 476. Endometrial Carcinoma Gynecological Finding 477. Endometrial CA Heart Murmur 478. Mitral Valve Prolapse www.brain101.info 11 Heart Valve in Bacterial Endocarditis 479. Mitral Heart Valve in Bacterial Endocarditis in IV drug users 480. Tricuspid Heart Valve involved in Rheumatic Fever 481. Mitral then Aortic Hereditary Bleeding Disorder 482. Von Willebrand’s Disease Hormone secreted in Pituitary Adenoma 483. Prolactin Inherited disease of the Kidney 484. Adult polycystic kidney disease: associated w/ polycystic liver, Berry aneurysms, Mitral prolapse 485. APD1 – chromosome 16 Intracranial tumor in adults 486. Glioblastoma mulitforme Islet Tumor 487. Insulinoma = β cell tumor Liver 1 ry Tumor 488. Hepatoma Liver Disease 489. Alcoholic Liver Disease Location of Adenocarcinoma of the Pancreas 490. Head (99%) Location of Adult Brain Tumors 491. Above Tentorium Location of Childhood Brain Tumors 492. Below Tentorium Lung Tumor, malignant or benign 493. Malignant Lung Tumor, primary or secondary 494. Secondary Lysosomal Storage Disease 495. Gaucher’s Malignancy in Women 496. Lung (2 nd breast) Malignancy of the Larynx 497. Glottic CA (squamous cell) Malignancy of the Small Intestine 498. Adenocarcinoma Malignancy Vulva 499. Squamous cell CA Malignant Eye Tumor in Kids 500. Retinoblastoma Malignant Tumor of the Liver 501. Hepatocellular CA Motor Neuron Disease 502. ALS Muscular Dystrophy 503. Duchenne’s: Dystrophin deletion. Presents <5yoa weakness at pelvic girdles w/ upward progression Nasal Tumor 504. Squamous cell CA Neoplasm – Child 505. Leukemia Neoplasm – Child (2 nd most common) 506. Medulloblastoma of brain (cerebellum) Neoplasm of the West 507. Adeno CA of the rectum and/or colon Neoplastic Polyp 508. Tubular adenoma Nephrotic Syndrome in Adults 509. Membranous Glomerulonephritis Nephrotic Syndrome in Children 510. Minimal Change (Lipoid Nephrosis) Disease (responds well to steroid txt) Non Hodgkin’s Lymphoma 511. Follicular small clear cell Number of Deaths per year in Women 512. Lung CA Skin tumor 513. Basal cell CA Opportunistic infection in AIDS 514. PCP Ovarian Malignancy 515. Serous Cystadenocarcinoma Ovarian Tumor 516. Hamartoma Pancreatic Tumor 517. Adeno (usually in the head) www.brain101.info 12 Patient with ALL / CLL / AML / CML 518. ALL – Child / CLL – Adult over 60 / AML - Adult over 60 / CML – Adult 35-50 Patient with Goodpasture’s 519. Young male Patient with Reiter’s 520. Male Pituitary Tumor 521. Prolactinoma (2 nd – Somatotropic “Acidophilic” Adenoma) Place for Primary Squamous Cell CA of esophagus 522. Mid 1/3 Place for Peptic Ulcer Disease 523. Lesser curvuture in antrum – associated w/ blood group O Primary Benign Salivary Tumor 524. Pleomorphic Adenoma (Mixed) – 90% localized to the parotid Primary Hyperparathyroidism 525. Adenomas (followed by: hyperplasia, then carcinoma) Primary Malignancy of Bone 526. Osteosarcoma Primary Malignancy of Small Intestine 527. Lymphoma Pt. with Hodgkin’s 528. Young Male (except Nodular Sclerosis type – Female) Pt. with Minimal Change Disease 529. Young Child Renal Malignancy 530. Renal cell CA Renal Malignancy of Early Childhood 531. Wilm’s tumor (neohroblastoma) – chromosome 11p Salivary Tumor 532. Pleomorphic adenoma Secondary Hyperparathyroidism 533. Hypocalcemia of Chronic Renal Failure Sexually Transmitted Disease 534. Chlamydia (sero types D-K) Site of Diverticula 535. Sigmoid Colon Site of Embolic Occlusion 536. Middle cerebral aa: contralateral paralysis; aphasias; motor & sensory loss Site of Metastasis 537. Regional Lymph Nodes Site of Metastasis (2 nd most common) 538. Liver Sites of Atherosclerosis 539. Abdominal aorta > coronary > popliteal > carotid Skin CA of Fair Skinned People 540. Malignant melanoma Skin Cancer 541. Basal Cell Carcinoma Small Intestine Congenital Anomaly 542. Meckel’s diverticulum Stomach Cancer 543. Adeno – associated w/ blood group A Testicular Tumor 544. Seminoma = malignant painless testes growth Thyroid Anomaly 545. Thryoglossal duct cyst Thyroid CA 546. Papillary CA Tracheoesophageal Fistula 547. Lower esophagus joins trachea / upper esophagus – blind pouch – polyhydramnios association Tumor in men <20 548. Germ cell tumor Tumor of Infancy 549. Benign vascular tumor = port wine stain = Hemangioma Tumor of the Stomach >50 years of age 550. CA of stomach (adeno CA) Type of Hodgkin’s 551. Mixed Cellularity (versus: lymphocytic predominance, lymphocytic depletion, nodular sclerosis) Type of Non-Hodgkin’s 552. Follicular, small cleaved Type of Portal Cirrhosis 553. Micronodular Type of Soft Tissue Tumor of Childhood 554. Rhabdomyosarcoma Vasculitis (of medium & small arteries) 555. Temporal Arteritis (branch of Carotid Artery) Viral Encephalitis 556. HSV Worm Infection in US 557. Pinworm (2 nd – Ascaris) www.brain101.info 13 Worst Prognosis in Thyroid Cas 558. Follicular CA Cause of Lobar Pneumonia 559. Strep. Pneumoniae Cause of Death b/t 24-44 yoa 560. AIDS Cause of Pneumonia in Cystic Fibrosis 561. Pseudomonas Cause of Osteomyelitis in IV Drug Users 562. Pseudomonas Cause of Infection in Burn Pts 563. Pseudomonas Mental Problem in Males 564. Specific phobia Intelligence Test 565. Stanford Binet (ages 6 & under) 566. WIPSI (ages 4-6) 567. WISK-R (for ages 6-17) 568. WAIS-R (for > 17 yoa) Paraphilia 569. Pedophilia Metabolite seen w/ Pheochromocytoma 570. VMA: vanillylmandelic acid (NE metabolite) Severe Shigella 571. Dysenteriae Bug in Otitis Media & Sinusitis in Kids 572. Strep. Pneumoniae Cause of a Solitary Brain Abscess 573. A. Israelli Cause of Bacterial Diarrhea in U.S. 574. Campylobacter jejuni Shigella Type 575. S. Sonnei Cause of Non-Ghonococcal Urethritis 576. Chlamydia trichomonas Pneumonia 577. Strep. Pneumoniae Urethritis 578. N. ghonorrhea Cause of Glomerulonephritis 579. IgA Nephropathy = Berger’s Disease Cause of Viral Pneumonia 580. RSV – infants 581. Parainfluenza – kids 582. Influenza virus – adults 583. Adeno virus – military recruits Complication of COPD 584. Pulmonary infections Cause of Death w/ SLE 585. Renal failure Atrial Septal Defect 586. Ostium Secundum Type Warm Antibody 587. Most common form of immune hemolytic anemia 588. IgG auto antibodies to RBC 589. See spherocytosis; (+) Coombs’ test; complication to CLL Immunodeficiency 590. IgA Deficiency Congenital GIT Anomaly 591. Meckel’s Diverticulum: persistence of vitelline duct/yolk sac stalk Cause of Congenital Malformation 592. Fetal Alcohol Syndrome Pharmacology Autonomic Nervous System Epinephrine 1. α1, α2, β1, β2 Norepinephrine 2. α1, α2, β1 (no β2 activity) GABA 3. Causes an inhibitory cell hyperpolarization Muscarinic-r 4. Uses DAG & IP3 as 2 nd messengers 5. Parasympathetic control www.brain101.info 14 Bethanechol 6. Cholinergic. ↑ GI & Bladder motility. Txt atonic bladder post-op Pilocarpine 7. Cholinergic. Pupillary constriciton= miosis. Ciliary constriction= accomodation. 8. Txt acute glaucoma Isoflurophate 9. Organophosphate. Irreversible acetylcholinesterase (-)r Pralidoxime 10. “2PAM”. Reverses organophosphate binding to acetylcholinesterase Neostigmine 11. Reversible acetylcholinesterase (-)r 12. Txt Myasthenia Gravis Myasthenia Gravis 13. Anitbodies to Ach-r. ↑’g muscular weakness due to Ach’s weak postsynaptic effect @ NMJ. Inactivates-r Tubocurium 14. Nondepol. Competitive cholinergic N-r (-)r. 15. Prevents Ach binding but does not activate NMJ 16. ↑ Histamine release= ↓ BP & ↑ bronchospasm Trimethaphan 17. Nonselectively binds N-r of the PS- and SNS Pancurium 18. More potent than tubocurium w/o histamine release Succinylcholine 19. Depol. Non competitive (-)r of muscle aciton 20. Opens Na Ch.= fasciculations. Closes Na Ch.= paralysis. Continuous infusion. α1 & Eye 21. Mydriasis due to norepinephrine. Prazosin (-). M-r & Eye 22. Miosis due to Ach. Atropine (-). Sympathetic 23. Post ganglionic symapthetic fibers releases norepinephrine Parasym. 24. Post ganglionic parasympathetic fibers release Ach M3-r & Eye 25. Contracts sphincter = miosis. Contracts ciliary = accomodation. M2-r & Heart 26. Negative chronotropy: ↓ HR = vagal arrest 27. Negative inotropy: ↓ contractility M3-r & Lung 28. Bronchospasm ↑secretions M3-r & GI 29. ↑ motility (cramps & diarrhea). Involuntary defecation Tacrine 30. Acetylcholine esterase (-)r. Txt Alzheimer’s Atropine 31. DOC w/ vagal arrest Glycoperrolate 32. M-r(-). Antispasmodic. Txt peptic ulcers. Pirenzepine 33. M-r(-). Antispasmodic. Txt peptic ulcers. Doxacurium 34. Most potent competitive non-depol NMJ (-)r. No cardiovascular side effects. No Histamine release. β bungarotoxin 35. Prevent the releasal of Ach from vesicles @ the pre synaptic nerve ending α bungarotoxin 36. Irreversible N-r (-)r = ↓ action potentials α1 & Eye 37. Contracts radial muscle = mydriasis (pupil dilation) α1 & Arterioles 38. Constiction: ↑TPR = ↑ Diastolic pressure = ↑ Afterload α1 & Venules 39. Constriction: ↑ Venous return = ↑ Preload α1 & Sex Function 40. Ejaculation ↑ Diastolic 41. ↑ α1 = ↑TPR ↓ Diastolic 42. ↑ β2; Direct acting vasodilators; (+)Cholinergics β1 & Heart 43. (+)chronotropism = ↑HR. 44. (+)inotropism = ↑ contractility; ↑SV; ↑CO; ↑O2 consumption. 45. ↑ conduction velocity Phenylephrine 46. α1 (+) Nasal decongestant. β2(+) Asma Drugs 47. Metaproterenol; Albuterol; Terbutaline; Ritodrine; Salmeterol Ritodrine/Turbutaline 48. Relaxes myometrium used in pre-mature labor pains Phentolamine 49. Epi reversal. Blocks α, vasodilation occurs. Pt goes from HyperTN to HypoTN. 50. Txt pheochromocytoma = ↓BP Terazosin 51. Txt BPH Yohimbine 52. ↑ sympathetic outflow = α2 (-). Txt impotence. Cardioselective NMJ 53. Pancuronium = ↑HR due to atropine-like anti muscarinic vagolytic effect & Gallamine (-)r Ecothiophate 54. Irreversible cholinesterase (-)r. Pyridostigmine 55. Cholinomimetic that ↑s M & N-r effects. (-) acetylcholinesterase & plasma cholinesterase 56. DOC for the oral Txt of MG www.brain101.info 15 Cardio Digoxin 1. ↓ AV nodal conduction/ inh. Na/K/Atpase = inc. Ca conc. in heart cells = inc. contraction force Diltiazem 2. Txt black men. Txt AV nodal re entrance Quinidine 3. ↓ AV nodal conduction. Cinchonism. Anticholinergic= aggravate MG. Hypotension= α block Verapamil 4. ↓ AV nodal conduction. ↓ BP. Negative inotrope= no CHF use Propranolol 5. ↓ AV nodal conduction. ↓ BP. Negative inotrope(= β block) Aggravates Asthma and Diabetes Melitus via β2 block. Diazoxide 6. Balanced vasodilator. Niroprusside 7. Balanced vasodilator. Unloads heart. ↑s cyanide= pre-txt w/ thiosulfate. Txt Acute HTN’v Crisis Reserpine 8. Txt severe & resistant HTN. Depletes CA. See stuffy nose. No to pts w/ peptic ulcers. Dobutamine 9. At high doses β2(+) offsets α1 = β1 ↑ CO w/o systemic vascular resistance Dopamine 10. At low doses Txt Shock= dilates renal and mesenteric aa= maintain urine output Esmolol 11. Short acting β(-) Captopril 12. Balanced vasodilator. Txt Outpt. CHF see dry cough(bradykinin induced) Digoxin 13. Txt CHF & Atrial Flutter - inotropic - ↓ K+ levels= dig. Toxicity Dig. Toxicity 14. Fatal ventricular arrhythmias w/ sever AV block Quinidine 15. ClassIa anti arrhythmic. Moderate Na Ch. Block Lidocaine 16. ClassIb anit arrhythmic. Normalizes conduction. Txt initial MI= control arrhythmias Flecanide 17. ClassIc anti arrhythmic. Marked conduction slowing Amiodarone 18. Long t1/2= need potent doses to obtain desired level for action. See blue skin, ocular deposits, Pulmonary Fibrosis. NE 19. ↑ AV nodal conduction via β1. Metoprolol(-) β1 Ach 20. ↓ AV nodal conduction via M receptor. Atorpine(-) M-r Atenolol 21. Controls catecholamine induced arrhythmias Bretylium 22. Txt Malignant Ventricular Arrhythmias but causes passing catecholamine release that can aggravate arrhythmias briefly Nimodipine 23. Txt Acute subarachnoid hemorrhage by preventing post hemorrhagic vasospasm Atropine 24. ↓ excess vagal tone as seen in Sinus Bradycardia Nitrates 25. ↓ preload= venous pooling. ↓ MVO2= reflex tachy. ↑ ventr work= dec O2 demand Propranolol 26. Blocks reflex tachy but causes excess brady= ↑ diastole time= ↑ EDV Verapamil 27. ↑ O2 supply via ↓ in vasospasm Txt Prinzmetal’s variant angina Aspirin 28. Prevents arterial platelet adhesion (not DVThrombi). Inactivates COX= ↓ platelet production of TxA2, a potent vasoconstictor Warfarin 29. (-)Vit. K dependent gamma carboxylation of clotting factors= anticoagulation state Heparin 30. Dependent on Antithrombin III activation TPA 31. Binds to fibrin clots & activates plasminogen on the spot. Short t1/2, given IV. 32. Does not discriminate b/t fibrin-based clots= bleeding & stroke complications arise Streptokinase 33. From bacteria= allergies arise. Can see excess bleeding in post-op pts. Urokinase 34. Human source. ↑ plasmin. Can see excess bleeding in post-op pts. Colestipol 35. Bile acid sequestrants. Interrupt bile acid reabsorption= ↑↑ LDL uptake. Cholestyramine same MOA. Lovastatin 36. HMGCoA reductase(-)= ↑ LDL-r synthesis. Pravastatin/ Mevastatin same MOA. Losartan 37. ↓ Aldosterone. ↑ Renin 2-3x’s Diazoxide 38. Txt insulinomas. Not balanced vasodilator= onlt dilates arterial smooth muscle Clonidine 39. Central α2(+). ↓ TPR via ↓ symapthetic effect Methyldopa 40. Central α2(+). (++) Coombs= Hemolytic anemia Phenytoin 41. ClassIb. Reverses mild AV block due to digitoxin toxicity Procainamide 42. ClassIa. SLE like syndrome. Indopamide 43. Only Thiazide that will have no effect on cholesterol levels www.brain101.info 16 Thiazides 44. Older black men w/ HTN due to ↑ Renin. β(-) 45. Young white men w/o asthma (cause bronchospasm) ACEIs 46. (-) change AI  AII. (-) Bradykinin inactivation. Captopril/ Enalapril 47. Cause renal failure = use w/ caution in the elderly Epinephrine 48. ↑ contraction rate & force via β1. 49. ↑ systolic but ↓ diastolic BP. 50. ↓ peripheral resistance via β2 vasodilaiton Norepi. 51. ↑ heart rate and ↑ systolic and diastolic BP 52. ↑ peripheral blood vessel resistance Methyldopa 53. DOC for pregnancy induced HTN Quinidine pre-txt 54. Atrial arrhythmia pretxt w/ a drug that will ↓ ventricular response: Dig.;β(-); Ca Ch.(-) ClassII 55. β(-) ↓risk fo reinfarction & sudden death following MI “Gray man” 56. Amiodarone: ClassIII antiarrhythmia Beperidil 57. Ca Ch(-). Limited clinical use due to Torsades de Pointes ACEIs 58. Vasodilate renal efferents > than afferent arterioles: ↓GFR & Filtration pressure 59. ↓ Diabetic renal failure progression Adenosine 60. Its receptor is blocked by Methylxanthines (ie… Theophyline) 61. Favored for the Txt of Reentrant Supra Ventricular Tachycardia Enoxaparin 62. Low molecular weight heparin = Oral anticoagulant Isoproterenol 63. ↑HR & ↓MAP Variant angina 64. Use Ca Ch. (-)r ie… Nifedipine Contraindicated in CHF 65. β (-)r = you don’t want to ↓ the heart’s pumping strength CNS “TOM” 1. Short –acting BDZs: 2. Triazolam 3. Onazelam 4. Midazolam Butyrophenone 5. Haloperidol & Droperidol Atypical D4 6. Clozapine – Thioridazine – Olanzepine – Risperidone = Do not cause EPS Flumazenil 7. BDZ antidote for OD Methylphenidate 8. Txt attention deficit disorder Phenytoin 9. Causes aplastic anemia/ gingival hyperplasia/ cleft lip & palate Thiopental 10. Short acting Barb Carbamazepine 11. DOC trigeminal neuralgia. Txt lennox gestaut seizures in kids Atypical D4-r 12. Thioridazine; Olamzapine; Clozapine Pimozide 13. Txt Tourette’s Risperidone 14. Good for negative symptoms Thioridazine 15. Most anti cholinergic neuroleptic Haloperidol 16. Neuroleptic malignant hyperthermia due to chronic D2 block. give Dantrolene and Bromocriptine Imirpamine 17. Enurisis Clomirpramine 18. Txt OCD See aggressive behavior w/ use Trazadone 19. Priapism Bupropion 20. Helps to quit smoking SSRIs 21. Primarily used for OCD Fluoxetine 22. Good for negative symptoms Phenelzine 23. Irreversible MAOI Lithium 24. Txt manic phase of Bipolar Disorder 25. Causes goiter by (-) conversion of T4 to T3 26. Nephrogenic diabetes insipidus 27. Low salt diet will lead to Li toxicity Alprazolam 28. DOC stage fright www.brain101.info 17 Propranolol 29. Social phobia κ-r 30. Spinal analgesia. Euphoria. ++euphoria. ++sedation. Constipation. µ-r 31. Supraspinal analgesia. Dysphoria. +respiratory depression. +sedation. Morphine & O2 32. Admin. is contraindicated to pts on morphine sedation= ↓ CO2 sensitivity and O2 admin. can stop breathing. Morphine 33. ↑ ICP = do not give to pt. with head trauma Morphine OD 34. 1.pinpoint pupils 2.↓’d respiraiton 3.coma Meperidine 35. Anesthetic used during labor Hydromorphone 36. µ(+) used in renal failure Tramadol 37. Ambulatory txt for mod. to severe pain Naloxone 38. Txt opioid OD. Reverses respiratory depression Pentazocine 39. Part κ(+) & part µ(-) Butorphenol 40. Part κ(+) & part µ(-) Nalbuphene 41. Part κ(+) & part µ(-) ↓ GABA 42. ↓ seizure focus= Barbs & BDZs ↓ Fast Na Ch. 43. ↓ electrical activity spread = Phenytoin & Carbamazepine Methoxyflurane 44. Can be nephrotoxic. Needs low MAC for anesthetic induction. Enflurane 45. Can cause tonic/clonic muscle spasms Isoflurane 46. Can cause bronchospasm Halothane 47. Can cause ventricular extrasystoles & Malignant hyperthermia & Hepatitis Nitric Oxide 48. No effect on HR. Needs high MAC for anesthetic induction. Thiopental 49. Short acting Barb. Kentamine 50. Dissociative anesthetic Droperidol 51. Can be used in combo w/ Fentanyl for neuroleptoanalgesic effect 52. Neuroleptic tranquilizer. Has mild alpha block Fentanyl 53. Can be used on combo w/ Droperidol for neuroleptoanalgesic effect 54. Used transdermally for chronic pain Midazolam 55. Pre anesthetic. Induces amnesia Primidone 56. Biotransformed to Phenobarb. C & A delta Fibers 57. First fibers to be blocked w/ anesthesia Esters 58. Procaine, Tetracaine, Benzocaine 59. Broken down and make PABA (allergen) Amides 60. Lidocaine, Mepivaciane, Bupivaciane, Etidocaine= “i” before “caine” always an amide 61. Metabolized in the liver Amphetamine 62. DA reuptake (-)’r. MAOI. Parkinson’s txt Bromocriptine 63. D2(+). Used w/ L-Dopa for “on-off” phenomenon of Parkinson’s Benztropine 64. Ant M w/ some DA reuptake (-). Parkinson’s txt Amantidine 65. ↓ DA reuptake. Can cause livido reticularis= skin mottling. Diphenhydramine 66. Txt early Parkinson’s stages Pergolide 67. > Effective & longer acting than Bromocriptine Ethosuximide 68. DOC for Absence seizures Tranylcypromine 69. MAOI = antidepressant SSRI & MAOI 70. Fatal combo, especially seen with the use of Paroxetine or Fluoxetine (SSRIs) and Tranylcypromine (MAOI) Labor opioids 71. Meperidine & Nalbuphine Desipramine causes 72. Sudden cardaic death in children www.brain101.info 18 Anti-Infective Primaquine 4. Malaria profylaxis 5. Used for extraerythrocytic forms Plasmodium vivax or P. ovale Ciporfloxacin 6. Quinolone derivative Sulfonamides 7. PABA structural analogs 8. Inhibit Folic acid synthesis Tertacyclines, anuria & the exception 9. Should not be used in anuric pt due to production of (-) Nitrogen balance & ↑d BUN levels. 10. Doxycycline is the exception Ceftriazone 11. 3 rd generation cephalosporin 12. DOC for bacterial meningitis in kids (ie… HiB) 13. One dose txt of gonorrhea Hepatic coma DOC 14. Neomycin (aminoglycoside) – it supresses the normal flora = ↓g NH4 production = ↓g free nitrogen levels in the bloodstream. Clavulanic acid 15. Irreversible (-)r of β lactamases, but ot of transpeptidase = use w/ a β lactamase sensitive penicillin Piperacillin 16. Txt Pseudomonas aeruginosa & Klebsiella 17. Broad spectrum antibiotic Streptomycin (aminoglycoside) 18. Txt Mycobacterium tuberculosis Isoniazid 19. Most commonly used drug for TB. 20. Usually combined w/ Rifampin and/or Ethambutol 21. Pre Txt w/ Pyridoxine (Vit B6) can prevent peripheral neuritis‘ Pyrantel Pamoate 22. Txt of Hookworm disease 23. Depolarizing NMJ (-)r Buy “AT” 30, “CELL” at 50 24. A = Aminoglycosides 25. T = Tetracyclines 26. C = Chloramphenicol 27. E = Erythromycin (macrolide) 28. L = Clindamycin 29. L = Lincomycin Cefoxitin 30. Txt intraabdominal infections (ie… w/ Bacteroides fragilis) 31. Traditional txt has been Clindamycin & Gentamycin Chloramphenicol 32. Broad spectrum antibiotic 33. Bone marrow depression (common) – Aplastic anemia (rare) 34. Gray baby syndrome (chloramphenicol cannot be conjugated) 35. DOC Typhoid Fever (symptomatic Salmonella infection) 36. DOC HiB meningitis in kids – especially resistant strain to ampicillin Nifurtimox 37. Txt trypanosomiasis Metronidazole 38. Txt Leishmaniasis & Amebiasis 39. Good for anaerobic bacteria = Bacteroides fragilis 40. DOC Trichomoniasis 41. DOC Giardia lamblia Txt P. carinii 42. TMP-SMX & Pentamidine Tetracycline 43. Txt of Brucellosis & Cholera 44. Txt Rocky Mountain Spotted Fever 45. Txt spirochete infections = Lyme disease (Borrelia burgdorferi) TMP-SMX 46. (-) dihydrofolate reductase activity Benzathine Penicillin G 47. Long duration of action = given once every 3-4 weeks for Txt of Syphilis Praziquantel 48. Txt Schistosomiasis (trematode [fluke] infections) Melarsoprol 49. Txt Trypanosomiasis that has neurological symptoms Stibogluconate 50. Txt Leishmaniasis Fluconazole 51. Txt fungal encephalitis Amphotericin B 52. Polyene antifingal Ketoconazole MOA 53. (-) fungal ergosterol synthesis = disrupts membrane Griseofulvin MOA 54. Accumulates in keratinized layers of the skin = used in dermatomycoses infections www.brain101.info 19 Mefloquine 55. Anti malarial 56. Txt Chloroquine resistant strains = P. falciparum Chloroquine 57. Txt for Malaria when inside RBC Nifurtimox 58. DOC Chagas disease due to Trypanosoma cruzi Erythromycin 59. Used in pts allergic to penicillins Nystatin 60. Topical txt of superficial mycotic infections = Candidiasis Acyclovir 61. Guanine analog 62. Txt Herpes infections Imipenem 63. Used w/ Cilastatin 64. Can cause seizures Cefoperazone side effects 65. Bleeding due to vit K level alterations 66. Contraindicated in pts w/ bleeding disorders Vancomycin 67. Used for MRSS (methicillin resistant Staph. Aureus) 68. “Red neck”: due to histamine release causes facial flushing Meropenem 69. used w/ Cilastatin 70. Does not cause seizures (cf w/ Imipenem) Nafcillin 71. Only penicillin that does not need dose adjustment in renal impairment Peripheral neuropathy 72. Seen w/ use of: 73. Metronidazole – Isoniazid – Vincristine – ddI – AZT – Allopurinol Sulfonamides & newborns 74. Kernicterus can occur “O.N.E.” for gonorrhea 75. Fluoroquinolones used in a one dose deal for gonorrhea: 76. O = Ofloxacin 77. N = Norfloxacin 78. E = Enoxacin Ribavirin 79. Txt RSV (Respiratory Syncytial Virus) Anti-Neoplastics Cyclosporine 80. Protects against rejections from organ transplants 81. Does not induce bone marrow depression Cyclophosphamide 82. Alkylating agent of both purine & pyrimidine bases of DNA 83. Txt CLL Cisplatin’s toxicities 84. Nephro- & Ototoxicity Methotrexate 85. Antimetabolite of folic acid: (-)dihydrofolate reductase Leucovorin Rescue 86. Can block/reduce Methotrexate = ↑ folic acid via a reduced folate Bleomycin toxicities 87. Pneumonitis & pulmonary fibrosis Azathiorine 88. Used in organ transplantation = kidney allografts 89. Allopurinol can ↑ its activity by (-) its biotransformation to xanthine oxidase MOPP 90. Chemotherapy used in the txt of Hodgkin’s disease 91. M = Mechlorethamine – nitrogen mustard 92. O = Oncovin (Vincristine) – prevents microtubule assembly 93. P = Procarbazine 94. P = Prednisone – glucocorticoid, inducing apoptosis Tamoxifen 95. (-) estrogen receptor 96. Txt of breast tumors, can see associated endometrial CA Flutamide 97. Antiandrogenic 98. Used w/ Leuprolide (LH-RH analog) 99. Txt prostatic CA Megestrol 100. (-) progesterone receptor 101. Txt endometrial CA Fluoxymesterone 102. Androgenic steroid 103. Txt mammary CA in postmenopausal women Methotrexate 104. Folic acid analog that (-) tetrahydrofolate synthesis by (-) dihydrofolate reductase 105. Txt of ALL 106. Txt of Psoriasis Brain tumor Txt 107. Lomustine 108. Carmustine – Causes pulmonary fibrosis www.brain101.info 20 Streptozocin 109. Attaches to β cells 110. Txt of pancreatic insulinomas Cytarabine (AraC) 111. Pyrimidine analog 112. DOC for AML Dactinomycin 113. Used for Wilms tumor & rhabdomyosarcoma Etoposide 114. Used for oat cell CA Paclitaxel 115. Used for ovarian CA Amifostine 116. Can ↓ nephrotoxicity due to chronic use of Cisplatin Pathology Mobitz I 117. Usually due to inferior MI. Rarely goes into 3 rd degree block. 118. Txt w/ Atropine or Isoproterenol. Mobitz II 119. BBB association. Often goes to 3 rd degree AV block. Usually due to anterior MI. P wave 120. Atrial depol. a wave 121. LA contraction T wave 122. Vetricular repol. Wavy fibers 123. Eosinophilic bands of necrotic myocytes. Early sign of MI. Janeway’s lesions 124. Acute bacterial endocarditis. 125. Nontender, erythematous lesions of palms & soles. Osler’s nodes 126. Subacute bacterial endocarditis. 127. Tender lesions of fingers & toes. Thiamine defcy 128. Wet Beri Beri heart. Dilated (congested) cardiomyopathy due to chronic alcohol consumption 129. Dyr Beri Beri = peripheral neuropathy 130. Wernicke-Korsakoff = ataxia; confusion; confabulation; memory loss Fibrinous Pericarditis 131. Associated w/ MI: Dressler’s Serous Pericarditis 132. Associated w/ nonbacterial; viral (Coxsackie) infection; immunologic reaction. Friction Rub 133. Pericarditis association Hemorrhagic Pericarditis 134. Associated w/ TB or neoplasm Restrictive Cardiomyopathy 135. Aka infiltrative cardiomyopathy that stiffens the heart 136. Due to amyloidosis in the elderly 137. Due to , also see schaumann & asteroid bodies in young (<25 yoa). PML’s infectious agent 138. JC Virus (Papovavirus = dsDNA, naked icosahedral capsid) Edema 139. ↑Pc (more seeps out) 140. ↓πc (less reabsorbed) 141. ↑ permeability 142. Block lymphatic drainage Adult Polycystic Kidney Disease 143. Commonly see liver cysts & Berry aneurysms along w/ kidney cysts. Hematuria & HTN also present. 144. 3 cysts in ea. Kidney w/ + family history confirms diagnosis Malignant HTN & Kidneys 145. Petehial hemorrhages are seen on kidney surfaces = Flea-Bitten surface = young black men Nephritic signs 146. Hematuria; RBC casts; HTN Nephrotic signs 147. Proteinuria; Hypoalbuminemia; Edema Podocyte Effacement seen w/ 148. Minimal Change (Lipoid nephrosis) disease ASO seen in 149. Acute post-streptococcal GN (due to βHGASrtep) 150. Anti streptolysin O Crescentic GN 151. Rapidly progressive GN – nephritic syndrome 152. Associated w/ multi system disease or post-strep/post infectious glomerular nephritis Hereditary Nephritis 153. Alport’s syndrome. X linked 154. Renal disease w/ deafness & ocualr abnormalities Membranoproliferative GN 155. Can be secondary to complement deficiency; chronic infections; CLL 156. See tram tracking TypeI Membrano Proliferative GN deposits 157. C3 & IgG deposits TypeII Membrano Proliferative GN deposits 158. Only C3 deposits www.brain101.info 21 GN deposits 159. Aka Dense deposit disease Focal segmental glomerulosclerosis deposits 160. IgM & C3 deposits Cold agglutinins 161. Seen in atypical pneumonia 162. It is IgM Ab with specificity for I Ag on adult RBCs Scrofula 163. TB in the lymph nodes Aspirin-Asthma Triad 164. Nasal polyps – Rhinitis – bronchoconstriction Ferruginous bodies 165. Hemosiderin (pigment w/ Fe 3- ) covered macrophages that have been pahgocytised Pancoast’s tumor causes 166. Ulnar nerve pain & Horner’s syndrome Fatty degeneration 167. Made up primarily of triglycerides 168. Most commonly due to alcoholism which commonly leads to hepatic cirrhosis 169. Associated w/ CCl4 - Cloudy swelling 170. Failure of cellular Na pump 171. Seen in Fatty degeneration of the liver and in Hydropic (Vacuolar) degeneration of the liver Hydropic degeneration 172. Severe form of cloudy swelling 173. Seen with hypokalemia induced by vomitting/diarrhea Liquefaction necrosis 174. Rapid enzymatic break down of lipids 175. Seen commonly in Brain & Spinal cord (CNS) injuries 176. Seen in suppurative infections = pus formation Coagulation necrosis 177. Result of sudden ischemia 178. Seen in organs w/ end arteries limited collateral circulation) = heart, lung, kidney, spleen Caseation necrosis 179. Combination of both coagulation & liquefaction necrosis 180. Seen w/ M. tuberculosis & Histoplasma capsulatum infection Fibrinoid necrosis 181. Seen in the walls of small arteries 182. Associated w/ malignant hypertension, polyarteritis nodosa, immune mediated vasculitis Fat necrosis 183. Result of lipase actions liberated from pancreatic enzymes 184. Seen w/ Acute pancreatitis = saponification results Hemoptysis 185. Blood in sputum Pulmonary embolism 186. Most commonly thrombus from lower extremity vein Phlebothrombosis 187. From a vein of lower extremities, of a pregnant uterus, in Congestive heart failure, bed ridden pt, 188. As a complicaiton in a pt w/ Pancreatic CA due to ↑d blood coagulability Saddle embolus 189. Embolus lodged in bifurcation of pulmonary trunks 190. ↑↑ RV strain = RV & RA dilate = Acute cor Pulmonale Paradoxical embolism 191. Right to Left shunt allows a venous embolism to enter arterial circulation 192. Patent ovale foramen or Atrial septal defect Tuberculoid granuloma 193. Collection of macrophages w/o caseation 194. Seen w/ Sarcoidosis (non-caseating); Syphilis; Brucellosis and Leprotic infections Cellulitis 195. Spreading infection due to streptococcus PSA 196. Prostate Specific Antigen = elevated in prostatic CA ↑↑5-HT 197. In cases of metastatic carcinoid, txt w/ Methysergide (5HT antagonist) ↑ αFeto Protein 198. Hepatocarcinoma 199. Neural tube defects CEA 200. Carcinoembryonic Antigen = elevated in Colon CA Chromosome 13 201. Retinoblastoma Chromosome 11p 202. Wilms tumor of the kidney Vinyl Chloride 203. Associated w/ Angiosarcoma of the liver Agent Orange 204. Contains dioxin 205. Implicated as a cause of Hodgkin;s disease, non-Hodgkin’s lymphoma & soft tissue sarcomas Parasites & CA 206. Schistosoma haematobium = Urinary bladder CA 207. S. mansoni = Colon CA 208. Aspergillus flavus = potent hepatocarcinogen Ochronosis 209. Alkaptonuria 210. Error in tyrosine metabolism due to Homogentisic acid (oxidizes tyrosine) 211. Involving intervertebral disks = Ankylosing Spondilitis = Poker spine 212. See dark urine; dark coloration of sclera, tendons, cartilage www.brain101.info 22 Lead poisoning 213. Acid fast inclusion bodies 214. ↑ urinary coproprophyrin 215. Anemia: microcytic/ hypochromic 216. Stippling of the basophils 217. Gingival line & lead line in bones: x-ray 218. Mental retardation Heroin OD, clinically 219. Massive pulmonary edema w/ frothy fluid from the nostrils Fetal alcohol syndrome 220. Small head, small eyes, funnel chest, ASD, mental deficiency, and hirsutism Atypical mycobacterium 221. M. kanasasii & M. avium intracellulare Cold abscesses 222. Liquefied TB lesions similar to pyogenic abscesses but lacking acute inflammation Actinomyces isrealli 223. Farmers infection 224. Lumpy jaw (from chewing grain) & PID (IUD), but most common is due to saprophyticus Congenital Syphilis 225. Saddle nose, Saber shin, Hutchinson’s teeth, nerve deafness, interstitial keratitis Warthin-Finkeledy cells 226. Reticuloendothelial giant cells on tonsils, lymph nodes, spleen 227. Seen with Rubeola (measles) due to paramyxovirus Diphyllobothrium latum 228. Tapeworm infection causing megaloblastic anemia by consuming large amount of vit B12 in the host Subacute Bacterial Endocarditis 229. α Hemolytic Streptococci (S. viridans) = usually in pt w/ pre-existing heart problem Acute Bacterial Endocarditis 230. Staph aureus, β Hemolytic Streptococci, E. coli 231. Common among drug addicts & diabetics Mitral Insufficiency 232. Ruptured papillary muscle Left Anterior Descending branch 233. Branch of the Left Coronary artery 234. Highest frequency of thrombotic occlusion 235. MI = anterior wall of the LV, especially in apical part of interventricular septum Left Circumflex branch 236. Branch of the Left Coronary artery 237. Occlusion = MI of posterior/lateral wall of the LV Dissecting Aneurysm 238. False aneurysm: it is splitting of the media of the aorta 239. Usually accompanied w/ long history of severe hypertension, also seen w/ familial hyperlipidemia, atherosclerotic disease, Marfan’s Collagen disease 240. Zones of medial necrosis +/- slitlike cysts = Medial Cystic Necrosis of Erdheim Cor Pulmonale 241. Right ventricular strain, associated w/ right ventricular hypertrophy Acute Cor Pulmonale 242. Sudden right ventricular strain due to a massive pulmonary embolism Bronchopneumonia 243. Lobular (rather than lobar) 244. Due to Staph aureus; Pseudomonas aeruginosa; Klebsiella; E. coli 245. Abscess formation is common Lobar pneumonia 246. Due to Strep. Pneumoniae infection (5% due to Klebsiella) 247. Red Hepatization: days 1-3 of the pneumonia 248. Gray Hepatization: days 3-8 of untreated pneumonia 249. Complicaitons: pleural effusion; atelectasia; fibrinous pleuritis; empyema; fibrinous pericarditis; otitis media Bronchiectasis 250. Permanent dilatation of the bronchi – predisposed by chronic sinusitis and post nasal drip 251. Supparation associated 252. Lower lobe > than upper lobe involvement Cold Agglutinins 253. Found w/ Mycoplasma pneumoniae Panlobular Emphysema 254. α1 – antitrypsin deficiency, causing elastase ↑ = ↑ compliance in the lung Bulla 255. Associated w/ Emphysema = “Bleb” = outpouching - If it ruptures causes Pneumothorax Farmer’s Lung 256. Due to Micropolyspora faeni (thermophilic actinomycetes) Bagassosis 257. Due to M. vulgaris (actinomycetes) 258. Inhalation of sugar cane dust Silo-Filler’s Lung 259. Due to Nitrogen dioxide from nitrates in corn G6PDH Deficiency 260. Sex-linked chronic hemolytic anemia w/o challenge or after eating fava beans 261. Heinz Bodies appear in RBCs HbF ↑↑ 262. Sickle Cell Anemia Multiple Myeloma 263. Lytic lesions of flat bones (“salt & pepper lesions”) = vertebrae, ribs, skull; Hypercalcemia; Bence- Jones protein casts Hodgkin’s Disease 264. Malignant neoplasm of the lymph nodes causing pruritis; fever = looks like an acute infection 265. Reed Sternberg cells www.brain101.info 23 Polyarteritis Nodosa 266. Immune complex disease of Ag-Ab complexes on blood vessel wall 267. Half of the immune complexes have Hepatitis B Ag 268. Can see fever; abd.pain; ↓ wt; HTN; muscle aches Sprue 269. Celiac disease due to a gluten-induced enteropathy = small intestine villi are blunted 270. High titers of anti-gliadin Abs & ↑ IgA levels Regional Enteritis 271. Crohn’s Disease 272. Association w/ Arthritis; Uveitis; Erythema Nodosum Whipple’s Disease 273. Intestinal Lipodystrophy = malabsorption syndrome Kulchitsky cells 274. Neural cest cells from which carcinoids arise = of the Bronchi; GIT; Pancreas Ulcerative Colitis 275. Inflammatory disease of the colon w/ ↑ colon CA incidence 276. Crypt abscess in the crypts of Lieberkuhn 277. Pseudopolyps when ulcers are deep 278. Not transmural involvement Vaginal Adenosis 279. Women exposed to DES (Diethylstilbesterol) in utero before the 18 th week of pregnancy 280. Some develop clear cell adenocarcinoma of the vagina & cervix Scirrhous Carcinoma 281. Infiltrating Duct Carcinoma w/ fibrosis – most common type of breast carcinoma Hofbauer Cells 282. Lipid laden macrophages seen in villi of Erythroblastosis Fetalis Retinopathy of Prematurity 283. Retrolental Fibroplasia = cause of bindness in premies due to high O2 concentrations IgA deficiency 284. Pt has recurrent infections & diarrhea w/ ↑ respiratory tract allergy & autoimmune diseases 285. If given blood w/ IgA = develop severe, fatal anaphylaxis reaction Priamry Sjorgen’s 286. Dry eyes & dry mouth, arthritis. ↑ risk for B cell lymphoma. HLA-DR3 frequent. Autoimmune disease. Secondary Sjorgen’s 287. Rheumatoid arthritis, SLE, or systemic sclerosis association 288. RA association shows HLA-DR4 LDH1 & LDH2 289. Myocardium. LDH1 higher than LDH2 = Myocardial Infarction LDH3 290. Lung tissue LDH4 & LDH5 291. Liver cells Keratomalacia 292. Severe Vit A deficiency. See Bitot’s spots in the eyes = gray plaques = thickened, keratinized ET Metabisfite Test 293. Suspending RBCs in a low O2 content solution 294. Can detect Hemoglobin S, which sickles in low O2 Microangiopathic Hemolytic Anemia 295. Can be due to Hemolyitc Uremic Syndrome & Thrombotic Thrombocytopenic Purpura (TTP) 296. See Helmet cells Wright’s stain 297. Stain for Burkitt’s lymphoma Mononucleosis 298. Due to EBV infeciton 299. If Mono is treated w/ Ampicillin, thinking that it is a strep pharyngitis, a rash will occur. T(8;14) 300. Burkitt’s lymphoma = c-myc oncogene overexpression T(9;22) 301. CML = c-abl/bcr gene formation = Philadelphia translocation Langerhan Cell Histiocytosis 302. Letter Siwe syndrome; Hand Schuller Christian Disease; Eosinophilic Granuloma 303. Birbeck granules are present = tennis racket shape Myeloid Metaplasia 304. Alkaline phosphatase ↑/normal compare to CML = low to absent 305. Anemia; splenomegaly; platelets > 1 million = extensive extra-medullary hematopoiesis Multiple Myeloma 306. Weakness; wt. loss; recurrent infection; proteinuria; anemia; ↑ proliferation of plasma cells in BM = plasma cell dx 307. Serum M protein spike – most often of IgG or IgA 308. Hypercalcemia (↑ bone destruction) T(14;18) 309. NH Lymphoma = bcl2 proto-oncogene overexpression seen w/ Small Cleaved Cell (Follicualr) Lymphoma Focal Segmental GN exs 310. IgA Focal GN = Berger’s disease; SLE; PAN; Schonlein-Henoch purpura (anaphylactoid purpura) Nephrotic Syndrome exs 311. Focal (Segmental) GN; Membranous GN; Lipoid (Minimal Change) GN; Membranoproliferative GN; Hep B; Syphilis; Penicillamine Schistosoma Haematobium 312. Infection is assocaited w/ Squamous cell CA of the Bladder (most common Bladder CA is transitional cell type) 313. Associated w/ portal HTN due to intrahepatic obstruction Penicillin Resistant PID 314. PID is usually due to N. Gonorrhoeae, but if unresponsive to penicillin think of Bacteroides species www.brain101.info 24 Duret Hemorrhages 315. Severe ↑ in ICP w/ downward diplacement of cerebellar tonsils into Foramen Magnum causing a compression on the brainstem w/ hemorrhaging into the pons & midbrain 316. Nearly always associated w/ death due to damage to the vital centers in these areas Hypertensive Hemorrhage 317. Predilection for lenticulostriate arteries = putamen & internal capsule hemorrhages Cerebral Embolism from 318. MI w/ Mural Thrombi; Atrial Fib Thrombi = Marantic thrombi; L-sided Bacterial Endocarditis; Paradoxical Embolism of septal defect Neurosyphilis 319. Tabes Dorsalis = ↓ joint position sensation, ↓ pain sensation, ataxia, Argyl Robertson pupils 320. Syphilitic meningitis 321. Paretic neurosyphilis 5p- 322. Cri di Chat: mental retardation; small head; wide set eyes; low set ears; cat-like cry Trisomy 13 323. Patau’s: small head & eyes; cleft lip & palate; many fingers Acute Cold Agglutinaiton 324. Abs to I blood group Ag. Mediated by IgM Abs 325. Complication of EBV or Mycoplasma pneumoniae infections Chronic Cold Agglutinaiton 326. Associated w/lymphoid neoplasms. See agglutination & hemolysis in tissue exposed to cold. IgM Abs RBC Osmotic Fragility 327. Hereditary Spherocytosis Non-Hodgkin’s Lymphomas 328. Small Lymphocytic: low grade B cell lymphoma of the elderly. Related to CLL. 329. Small Cleaved cell (Follicualr): low grade B cell lymphoma of the elderly. T(14;18) bcl-2 oncogene 330. Large Cell 331. Lymphoblastic: high grade T cell lymphoma of kids progressing to T-ALL 332. Small Non Cleaved = Burkitt’s: high grade B cell lymphoma. EBV infection. Starry sky histo appearance. T(8;14) c-myc proto-oncogene. Related to B-ALL Singer’s Nodules 333. Benign laryngeal polyps associated w/ smoking & overuse of the voice Paraseptal emphysema 334. Associated w/ blebs (large subpleural bullae) that can rupture and cause pneumothorax Superior Vena Cava Syndrome 335. Obstructed due to bronchogenic carcinoma. Causing swollen face & cyanosis. Betel nuts 336. Associated to oral cancer. Fundal (Type A) Gastritis 337. Antibodies to parietal cells; pernicious anemia; autoimmune diseases Antral (Type B) Gastritis 338. Associated w/ Helicobacter (Campylobacter) pylori infection. 90% of duodenal ulcer Primary Biliary Cirrhosis 339. Autoimmune origin; middle aged women; anti-mitochondrial Abs 340. Jaundice; itching; hypercholesterolemia (can see cutaneous xanthomas) Acute Pancreatitis 341. ↑ pancreatic enzymes = fat necrosis; sapponification = hypocalcemia; ↑ serum amylase 342. Severe epigastric ab pain; prostration; radiation to the back Radiating Back Pain 343. Chronic pancreatitis Complete Hydatidiform Mole 344. No embryo. Paternal derivation only. 46XX Partial Hydatidiform Mole 345. Embryo. 2 or more sprems fertilized 1 ovum: triploidy/tetraploidy occurs Cold Nodules 346. Hypoplastic Goiter nodules that do not take up radio active iodine. [Opposite: hot & do take up iodine] Acidophils 347. Mammotrophs = Prolactin 348. Somatotrophs = GH Basophils 349. Thyrotrophs = TSH 350. Gonadotrophs = LH 351. Corticotrophs = ACTH & FSH Lacunar Strokes 352. Small/focal aa occlusions. Purely motor or sensory. 353. Sensory: lesion of thalamus 354. Motor: lesion of internal capsule CSF of Bacterial Meningitis 355. ↓ Glucose; ↑ Protein; ↑ Neutrophils; ↑ Pressure CSF of Viral Meningitis 356. Normal Glucose; +/-↑ Protein; ↑ Lymphocytes Marble Bone Disease 357. Osteoporosis: Albers-Schonberd Disease = inspite of ↑d bone density, many fractures = ↓ osteoclasts C5a 358. Involved in Chemotaxis (for Neutrophils) C3b 359. Involved in Opsonization (& IgG) Anaphylotoxins 360. C3a & C5a (mediate Histamine release from Basophils & Mast cells) Vasoactive Mediators 361. Vasoconstriction: TxA2; LTC4; LTD4; LTE4; PAF 362. Vasodilation: PGI2; PGD2; PGE2; PGF2α ; Bradykinin; PAF 363. ↑d Vascular Permeability: Hist.; 5HT; PGD2; PGE2; PGF2α ; LTC4; LTD4; LTE4; Bradykinin; PAF www.brain101.info 25 Platelet Aggregation 364. ADP; Thrombin; TxA2; collagen; Epinephrine; PAF Platelet Antagonist 365. Prostacyclin (PGI2) Intrinsic Pathway 366. F XII (Hagman): APTT Extrinsic Pathway 367. F VII: PT Lines of Zahn 368. Aterial thrombi = pale red colored (dark red is venous thrombi) Currant Jelly appearance 369. Post mortem clots Emigration: Chemotaxis 370. Margination 371. Pavementing 372. Adhesion 373. Chemotaxis 374. Phagocytosis 375. Intracellular microbial killing Transudate 376. Specific gravity < 1.012 – low protein Exudate 377. Specific gravity > 1.020 – high protein Hurler’s 378. Lysosomal storage disease α L Iduronidase – Heparan/Dermatan Sulfate accumulation Galactosemia 379. Deficiency of Galactose 1 Phosphate Uridyl Transferase. ↑ Galactose 1 Phosphate Phenylketonuria 380. Deficiency: Phenylalanine Hydroxylase. ↑ Phenyalanine & degradation products 381. Mousy body odor Autosomal Dominant Diseases 382. Adult Poly Cystic Kidney Disease 383. Familial Hypercholestrolemia Disease 384. Hereditary Hemorrhagic Telengectasia (Osler-Weber-Rendu) 385. Hereditary Spherocytosis 386. Huntington’s Disease (chromosome 4p) 387. Marfan’s Syndrome 388. Neurofibromatosis (von Recklinghausen’s) 389. Tuberous Sclerosis 390. Von Hippel Lindau Disease Autosomal Recessive Diseases 391. Tay-Sachs 392. Gaucher’s 393. Niemann-Pick 394. Hurler’s 395. Von Gierke’s 396. Pompe’s 397. Cori’s 398. McArdle’s 399. Galactosemia 400. PKU 401. Alcaptonuria X Linked Recessive Diseases 402. Hunter’s Syndrome (L-Iduronosulfate Sulfatase deficincy, ↑ Heparan/Dermatan Sulfate) 403. Fabry’s Disease (α Galactosidase A deficiency, ↑ Ceremide Trihexoside) 404. Classic Hemophilia A (Factor VIII deficiency, F8 Gene on X chromosome is bad, ↑ Ceremide Trihexoside) 405. Lisch-Nyhan Syndrome (HGPRT deficiency, ↑ Uric acid) 406. G6Phosphatase deficiency (G6PDH deficiency, ↑ Ceremide trihexoside) 407. Duchenne’s Muscular Dystrophy (Dystrophin deficinecy, ↑ Ceremide Trihexoside) Hypersensitivity Reactions “ACID” 408. Type I (Anaphylactic): IgE mediated. Exs: Hay Fever; Allergic asthma; Hives 409. Type II (Cytotoxic): Warm Ab autoimmune hemolytic anemia; hemolytic transfusion reactions; Erythroblastosis Fetalis; Grave’s Disease; Goodpastures 410. Type III (Immune Complex): Insoluble complement bound aggregates of Ag-Ab complexes. Exs: Serum sickness; Arthus Reaction; Polyarteritis Nodosa; SLE; Immune Complex Mediated Glomerular Disease 411. Type IV (Delayed = Cell mediated immunity): Delayed hypersensitivity. Involves memory cells. Exs: Tuberculin reaction; Contact dermatitis; Tumor cell killing; Virally infected cell killing Transplant Rejections 412. Hyperacute Rejection = occurs w/in minutes of transplant. Ab mediated. 413. Acute Rejection = occurs w/in days to months of transplant. Lymphocytes & macrophages. Only rejection type that can be treated w/ therapy. 414. Chronic Rejection = occurs months to years of transplant. Ab mediates vascular damage. Blood Metastasis 415. Sarcoma, exception – renal cell CA: early venous invasion Lymph Metastasis 416. Carcinoma, exception – renal cell CA: early venous invasion www.brain101.info 26 Aflatoxin 417. Seen w/ Aspergillus. ↑ risk for Hepatocellular CA Cleft Lip 418. Incomplete fusion of maxillary prominence w/ median nasal prominence Cleft Palate 419. Incomplete fusion of lateral palatine process w/ each other & median nasal prominence & medial palatine prominence Craniopharyngioma 420. Pituitary tumor - usually calcified Lateral Geniculate Nucleus Inolved in Vision relay Medial Geniculate Body Involved in Hearing relay Lung Development Glandular: 5-17 fetal weeks Canalicular 13-25 fetal weeks Terminal Sac 24 weeks to birth Alveolar period birth-8yoa Heart’s 1 st Beat 21-22 days Foregut Mouth  Common Bile Duct - supplied by Celiac Artery Midgut Duodenum, just below Common Bile Duct  Splenic flexure of the Colon supplied by Superior Mesenteric artery Hindgut Splenic Flexure  Butt crack  supplied by Inferior Mesenteric Artery Hypnagogic Hallucinaitons Narcolepsy Type I Error α: “Convicting the innocent” – accepting experimental hypothesis/rejecting null hypothesis Subdural Hematoma Ruptured cerebral bridging veins Epidural Hematoma Ruptured middle meningeal artery “intervals of lucidness”, 2 ry to Temporal bone fracture Type II Error β: “Setting the guilty free” – fail to reject the null hypotesis when it was false Power 1 - β Sensitivity TP/TP + FN Specificity TN/TN + FP Positive Predictive Value TP/TP + FP Negative Predictive Value TN/TN + FN Odds Ratio ad/bc d-Dimers DIC Delusion Disorder of thought content Loose Association Skip from topic to topic 5 Stages of Death Denial – Anger – Bargaining – Depression – Acceptance 1 st Branchial Arch Meckel’s cartillage – gives rise to incus/malleus bones of ear 2 nd Branchial Arch Reichert’s cartillage – gives rise to stapes bone of ear Median nerve lesion No pronation Radial nerve lesion Wrist drop – seen w/ humerus fracture Common peroneal lesion Foot drop. No dorsiflexion or eversion of the foot Diract inguinal hernia Goes through superficial inguinal ring. Medial to inferior epigastric artery Seen in older men Indirect inguinal hernia Goes through deep & superficial inguinal ring Lateral to inferior epigastric artery Seen in young boys – processus vaginalis did not close @ Diaphragm T8, T10, T12 T8 = Inferior vena cava T10 = Esophagus/ Vagus T12 = Aorta/ Thoracic duct/ Azygous vein Hemiballism Wild flailing of 1 arm. Lesion of the sub thalamic nucleus O Linked Oligosaccharide In the Golgi N Linked Oligosaccharide In the RER MLF Syndrome Internuclear Ophthalmoplegia: medial rectus palsy on lateral gaze; Nystagmus on abducting eye. Seen w/ MS ADA Deficiency SCID Raphe Nucleus Initiation of sleep via 5HT predominance www.brain101.info 27 β waves Alert; Awake; Active mind – also seen in REM, therefore we say “paradoxical sleep” Irreversible Glycolysis Enzymes Hexokinase PhosphoFructo Kinase = Rate Limiting Step Pyruvate Kinase Pyruvate Dehydrogenase Irreversible Gluconeogenesis Enzymes PyruvateCarboxy Kinase PEPCarboxyKinase Fructose 1,6 BiPhosphatase Glucose 6 Phosphatase **muscle dose not take part in Gluconeogenesis, only takes place in the liver, kidney & GI epithelium Pellagra Diarrhea, Dermatitis, Dementia Niacin Deficiency (Vit B3 deficiency) Hartnup’s Disease Malignant Carcinoid Syndrome INH use TLCFN Needed as co-factor for Pyruvate DH complex & α Ketoglutarate DH complex LCAT or PCAT Esterification of cholesterol: lecithin cholesterol acetyltransferase Lecithin = Phosphatidylcholine, therefore phosphotidylcholine acetyltransferase HMGCoA Reductase Rate limiting step in cholesterol synthesis Changes HMGCoA  Mevalonate (-) by Lovastatin Ketogenic amino acids Leucine & Lysine Glucogenic amino acids Methionine, Threonine, Valine, Arginine, Histadine Keto & Gluco amino acids Phenylalanine, Trytophan, Isoleucine Carnitine Shuttle Feeds FA into the mitochondria for their consumption Cori Cycle Keeps muscles working anaerobically. Transfers lactate to the liver to make glucose which is sent back into the muscles for energy use (-) Na + Pump (ATPase) Ouabain [(-) K + pump] Vanadate [(-) phosphorylation] Digoxin [↑ heart contractility] TCA Cycle Products “Citric Acid Is Krebs Starting Substrate For Mitochondrial Oxidation” Citrate  Aconitate  Isocitrate  α Ketoglutarate  Succinyl  Succinate  Fumarate  Malate OAA Cones Color vision. Contain Iodopsin = Red-Blue-Green specific pigment. For acuity. Rods Contain Rhodopsin pigment. High sensitivity. Concentrated in the fovea. Night vision. Gastrula Seen @ 3 rd week: Ecto, Meso & Endo Epiblast @ 2 nd week: forms the primitive streak, from which Meso & Endo come from. Directly gives rise to Ecto. Sydenham’s Chorea Post streptococcal infection. Necrotizing arteritis of the caudate, putamen, thalamus (+) Frei Test Chlamydia trachomatis types L1, L2, L3 = Lymphogranuloma venereum Sabouraud’s Agar Culture for all Fungi ie…Culture Cryptococcus neofromans which is found in pigeon droppings FMR1 Gene Defect Fragile X Syndrome: macro-orchidism; long face; large jaw; large everted ears; autism, mental retardation Barr Body Present in Kleinfelters: Male: XXY Not present in Turner’s: Female: XO Aortic Insufficiency Signs Traube Sign = Pistol shot sound over the femoral vessels Corrigan pulse = water hammer pulse over coratid artery = aortic regurgitation Scleroderma :”CREST” Calcinosis; Raynauds; Esophageal; Sclerodactyl; Telangiectasis Cretinism Sporadic: bad T4 phosphorylation or developmental failure of thyroid formation Endemic: no Iodine in diet: protruding belly & belly button Hemochromatosis Triad Micronodular pigment cirrhosis; Bronze Diabetes; Skin pigmentation = due to ↑ Fe 3+ deposition www.brain101.info 28 Signature Drug Toxicities Agranulocytosis 421. Clozapine, Chloramphenical Aplastic Anemia 422. Chloramphenicol 423. NSAIDs 424. Benzene Atropine-like Side Effects 425. Tricyclics Cardiotoxicity 426. Doxorubicin 427. Daunorubicin Cartilage Damage in Children 428. Fluoroquinolones (Ciprofloxacin & Norfloxacin) Cinchonism 429. Quinidine Cough 430. ACE Inhibitors Nephrogenic Diabetes Insipidus 431. Lithium (Txt w/ Amiloride) Disulfiram-like Effect 432. Metronidazole 433. Sulfonylureas (1 st generation) Extrapyramidal Side Effects 434. Antipsychotics (Thioridazine, Haloperidol, Chlorpromazine) Fanconi’s Syndrome 435. Tetracycline Fatal Hepatotoxicity (necrosis) 436. Valproic Acid 437. Halothane 438. Acetaminophen Gingival Hyperplasia 439. Phenytoin Gray Baby Syndrome 440. Chloramphenicol Gynecomastia 441. Cimetidine 442. Azoles 443. Spironolactone 444. Digitalis Hemolytic Anemia in G6PD- deficiency 445. Sulfonamides 446. Isoniazid 447. Aspirin 448. Ibuprofen 449. Primaquine Hepatitis 450. Isoniazid Hot Flashes, Flushing 451. Niacin 452. Tamoxifen 453. Ca ++ Channel Blockers Induce CP450 454. Barbiturates – Phenobarbital 455. Phenytoin 456. Carbamazepine 457. Rifampin Inhibit CP450 458. Cimetidine 459. Ketoconazole Interstitial Nephritis 460. Methicillin 461. NSAIDs (except Aspirin) 462. Furosemide 463. Sulfonamides Monday Disease 464. Nitroglycerin Industrial exposure → tolerance during week → loss of tolerance during weekend → headache, - ach, dizziness upon re-exposure Orange Body Fluids 465. Rifampin Osteoporosis 466. Heparin 467. Corticosteroids Positive Coombs’ Test 468. Methyldopa Pulmonary Fibrosis 469. Bleomycin 470. Amiodarone Red Man Syndrome 471. Vancomycin Severe HTN with Tyramine 472. MAOIs www.brain101.info 29 SLE-like Syndrome 473. Procainamide 474. Hydralazine 475. INH Tardive Dyskinesia 476. Antipsychotics (Thioridazine, Haloperidol, Chlorpromazine) Clozapine: only antipsychotic to not 477. give you tardive dyskinesia Tinnitus 478. Aspirin 479. Quinidine Microbiology Lactose formers 1. “CEEK” 2. Citrobacter 3. Enterobacter 4. E.Coli (K1 capsule most important) 5. Klebsiella Non lactose formers 6. “SHYPS” 7. Shigella 8. Yersinia enterolytica (AKA Pestis) 9. Proteus 10. Salmonella May lack color 11. “These rascals may microscopically lack color”: 12. Treponema 13. Ricksetta 14. Mycobacterium 15. Mycoplasma 16. Legionella 17. Chlamydia ↑ cAMP 18. “CAPE” 19. Cholera 20. Anthracis (Poly D glutamate capsule) 21. Pertusis (via Gi) 22. E.coli (LT enterotoxin) Have Capsules [ie… are Quellung Reaction (+)] 23. “Some killers have pretty nice capsules” 24. Strep. Pneumoniae 25. Klebsiella 26. HiB 27. Pseudamona Aeroginosa 28. Neisseria meningitis 29. Cryptococcus neoformans (only encapsulated fungal pathogen) Dimorphic Fungi 30. “Can Also Have Both Shapes” 31. Cocciodes 32. Aspergillus 33. Histolpasma 34. Blastomyces 35. Sprothrix schenkii Have β Prophage 36. “OBED” 37. O = Salmonella 38. B = Botulinum 39. E = Erythrogenic strep 40. D = Diptheria Spore Forming Bacteria 41. Bacilus & Clostridium (have calcium di-picolinate) IgA Proteases 42. Neisseria, Haemophilus, S. pneumoniae Widal Test 43. Salmonella (Salmonella begins in the ileocecal region) agglutination indicates Abs to O, H, Vi Salmunella Ags Wayson’s Stain 44. Yersinia Pneumonic Plaque Transmission 45. Person to person cf w/ Bubonic plaque that was via infected flea Splenectomy 46. Predisposes to septicemia Invasins 47. Yersinia pseudotuberculosis Fusiform 48. Vincent’s trench mouth Motile: make H2S Non Motile: noH2S www.brain101.info 30 S. viridans 49. Dextran mediated adherence Obligate Aerobes 50. Pseudomonas & Mycobacterium Obligate Anaerobes 51. Clostridium, Actinomyces, Bacteroides Staph aureus 52. A Protein, Catalase +/ Coagulase + Spirochetes 53. Treponema, Borrelia, Leptospira Non Motile Gram (+) Rods 54. Corenybacterium D & Nocardia Acid Fast Organisms 55. Mycobacterium; Cryptosporidium; Nocardia (partially); Legionella micdadei; Isospora Pigment Producing Bacteria 56. Serratia – red (can cause pseudohemoptysis) 57. Pseudomonas A – piocyanin blue/green 58. Staph Aureus – yellow – Protein A 59. Mycobacteria – photo/scoto chromogenic – caritinoid – yellow/orange 60. Corneybacterium D – black/gray – pseudomembrane plaque in throat 61. Bacteroides (Porphyromonas) melaninogenicus – black (heme) 62. E. coli – irredescent green sheen Bacterial Morphology 63. Pneumococci – lancet shaped diplococci 64. Neisseria – kidney bean shaped diplococci 65. Camphylobacter – gulls’ wings/comas 66. Vibrio Cholera – coma shaped 67. Corneybacterium D – club shaped (nonmotile, G+Rod) 68. Yersinia – safety pin seen in Wayson’s stain Inclusion Bodies 69. Rabies – Negri bodies – intracytoplasmic 70. Pox virus – Guarnieri – intracytoplasmic & acidophilic 71. CMV – Owl’s eyes – intracytoplasmic & intranuclear 72. HSV – Cowdry bodies – intranuclear Schistosoma Japonicum Monsoni 73. Intestinal – contact w/ bad water Schistosoma Haematolium 74. Vesicular – contact w/ bad water Non Human Schistosom 75. Swimmer’s itch – contact w/ bad water Clonorchichis 76. Chinese liver fluke – eating raw fish. Txt: Praziquantel Fasciola Hepatica 77. Sheep – eating raw fish. Txt: Praziquantel Fasciola Biski 78. Giant intestinal flukes – eating raw fish. Txt: Praziquantel Paragonimus Westermani 79. Lung fluke – eating raw fish. Txt: Praziquantel Oxidase (+) 80. Neiserria and most Gram (-)s Micro Aerophilic 81. Camphylobacter & Helicobacter Urease (+) 82. All Proteus – can cause Staghorn/Struvite calculi (NH4 - Mg 2- stones): alkaline urine 83. Ureaplasma 84. Campylobacter pylori (Helicobacter) 85. Cryptococcus 86. Nocardia Coagulase (+) 87. Staph A & Yersenia pestis Obligate Intracellular Bacteria 88. Chlamydia Pistacci (Chlamydia do not make own ATP); Mycobacterium Leprae; all Rickettsia except Roachalimea (make suficient ATP to survive) Protozoa 89. Plasmodium; Toxoplasma ghondi; Babesin; Leishmania; Trypanosoma Cruzi Obligate Non Intracellular Parasites 90. Treponema palidum & Pneumocystis Carinii (cannot be cultured on inert media but can be found extra cellularly in the body) Haemophilus Factors 91. X = Protoporphyrin & V = NAD All cocci are 92. Gram (+) except for Neisseria & Moraxella “Eaton Fried Eggs” 93. Mycoplasma pneumoniae has fried egg colonies on Eaton agar (needs cholesterol) Mycoplasma 94. No cell wall. Membrane has cholesterol. Smallest living bacteria. 95. P1 protein inhs ciliary action 96. Fried egg colonies 97. Atypical pneumonia – young adults Sabrands 98. Fungal media Malassazia furfur 99. Spaghetti & meat ball Measles’ 3C’s 100. Cough – Coryza – Conjunctivitis. Can also have photophobia 101. May lead to subacute Sclerosing Panencephalitis www.brain101.info 31 Non Motile Bacilli & Clostridium 102. B. Anthracis & C. Perfringens Bloody diarrhea agents 103. EIEC – EHEC – Shigella - Yersenia enterocolitica – Entaemeba histolytica – Salmonella – Campylobacter jejuni YW-135CA 104. N. meningitidis vaccine capsualr polysaccharide strains Indian Ink 105. Cryptococcus neoformans Naegleria causes 106. Colonization in the nasal passages after swimming Need Cysyeine for growth 107. “Ella likes cysteine”: 108. Francisella 109. Brucella 110. Legionella 111. Pasturella Endotoxins, G(+) or G(-) 112. Gram (-): N. meningitidis Ecthyma Gangrenosum, seen w/ 113. Pseudomonas aeroginosa. Target shaped skin lesions w/ a black center and red ring surrounding the lesion Endospores G(+) 114. Gram (+): Bacillus & Clostridium – made up of dipicolinate & Keratin Multi Brain Abscess 115. Nocardia Single Brain Abscess 116. Actinomyces israelli ↑ risk for Strep pneum Infection 117. Asplenic; Sickle cell anemia; immunocompromising illness α Hemolysis/Optochin Sensitive 118. Strep. Pneumoniae α Hemolysis/Optochin Resistant 119. Strep. Viridans (Subacute Endocarditis) Staph. Saprophyticus 120. Novobiocin Resistant (UTIs) Staph. Epidermidis 121. Novobiocin sensitive (Endocarditis in IVDUs) β Hemolysis/Bacitracin Sensitive 122. Strep. Pyogenes (pharyngitis; Scarlet fever; cellulitis; impetigo; Rheumatic fever)) 123. Hyaluronic capsule; non-motile; M proteins; Endotoxin A β Hemolysis/Bacitracin Resistant 124. Strep. Agalactiae (Diabetes predisposes to infection) EFII Ribosylation 125. Diphtheria toxin & Pseudomonas exotoxon A Bacillus Anthracis: 3 toxins (work via adenylate cyclase) 126. Protective Antigen (PA) 127. Lethal Factor = toxic to macrophages 128. Edema Factor = ↑ cAMP Woolsorter’s Disease 129. Bacillus anthracis. DOC: Penicillin Grows in Rice 130. Bacillus Cereus Clostridium Perfringens 131. Double Zone β Hemolysis (test) 132. Lecithinase: α toxin = lyses RBCs 133. 80% of gas gangrene (myonecrosis) cases Clostridium Difficile 134. 2 Toxins: Enterotoxin (Exotoxin A) & Cytotoxin (Exotoxin B) 135. Pseudomembranous colitis (can be precipitated by clindamycin/ampicillin) Spastic Paralysis toxin 136. Clostridium Tetani toxin Clostridium Botulinum 137. Bad canned foods have neurotoxin = flaccid paralysis (block Ach release) Infant Botulinum 138. Floppy Baby Syndrome. Pre formed toxin in honey Thayer Martin Agar 139. Neisseria ID DOC for N. gonorrhoeae 140. Ceftriazone K1 E. Coli Capsular Ag 141. Related w/ neonateal meningitis The A’s of Klebsiella 142. Alcoholics 143. Aspiration pneumonia 144. Abscesses in the lungs Rice H2O Diarrhea 145. Vibrio Cholera: metabolic acidosis Raw seafood intoxicaiton 146. Vibrio parahemolyticus Helicobacter Txt 147. Bismuth salts; Metronidazole; Tetracycline (or amoxicillin) ↑ risk of P. aeroginosa infection 148. Burn patients & Cystic fibrosis Contact lens’ infection 149. Pseudomonas aeroginosa Cat Bites 150. Pasteurella multocida Undulant Fever 151. Brucella Bordet Gengou Agar 152. Bordetella pertusis ID www.brain101.info 32 Lowenstein-Jensen medium 153. M. tuberculosis ID Cat Scratch Disease 154. Bartonella henselae. Leion can resemble Kaposi’s sarcoma. 155. Toxoplasmosis Pink Eye 156. Adenovirus (type 8) True Hemaphrodite 157. Testes & Ovaries are present Pseudo Hemaphrodite 158. External genitalia does not coincide w/ gonads Male Pseudo Hemaphrodite 159. Testicular Feminization HLA Genes Location 160. 6p Parvovirus B19 161. Fifth Disease: Erythema Infectiosum (ssDNA). Linked w/ sicle cell anemia Interferon MOA 162. Inhibits viral replication (translation or transcription) Acute Hemorrhagic Conjunctivitis 163. Seen w/ infections from Enterovirus & Coxsackie A Parainfluenza Causes… 164. Croup (Laryngotracheobronchitis) Swimming Pool Conjunctivitis 165. Adenovirus (types 3 & 4) RSV 166. Bronchiolitis in infants Removed tonsils, find what virus 167. In 80%, Adenovirus. In the immunosuppressed, activation can occur Bone Fever 168. Dengue: Group B Togavirus, from the Arbovirus, transmitted by mosquitos HbsAg 169. Appears in blood soon after infection, before onset of acute illness 170. Disappears w/in 4-6 months after the start of clinical illness HbeAg 171. Appears early acute phase, indicates higher risk of transmitting the disease 172. Disappears before HbsAg is gone Anti-Hbc 173. Present in beginning of clinical illness 174. Seen in the “window phase” Filamentous Bacteria 175. Actinomycetes = Nocardia; Actinomyces; Streptomyces Listeria contaminates 176. Milk, cheese, vegetables (coleslaw) in recent infections Shiga like Toxin 177. E. Coli 0157/H7: Hemorrhagic colitis & Hemorrhagic uremic syndrome Necrotizing Fasciitis 178. Group A Streptococci Relapsing Fever 179. Borrelia recurrentis Loffler’s Medium 180. Corneybacterium diphtheriae Chlamydiae Developmental Cycle 181. Elementary Body: infeccious particle that Enters the cell 182. Reticulate Body: made from elementary body. Replicates, differentiates and releases elementary bodies to infect other cells 183. W/ infection you will see Glycogen containing inclusions 184. Cell wall lacks muramic acid Trench Fever 185. Rochalimaea quintana “Spotted Fever” Members 186. Rickettssia rickettsii (RMSF) & R. akari (rickettsial pox) in the U.S. 187. R. sibirica (tick typhus in China) & R. australis (typhus in Australia) Thrush Txt 188. Nystatin txts candidiasis of the mouth Rose Bush Thorns 189. Have Sporothrix schenckii Contact lens solution infection 190. Acanthamoeba Filiariasis Causant 191. Wucheria bancrofti (infection aka elephantitis & wucheriasis Freshwater lake infection 192. Causes amebic meningoencephalitis due to Naegleria fowleri Reduviid bug bite 193. Transmits Trypanoma cruzi (Chagas’ disease): Romana’s Sign Schistosoma Haematobium causes 194. Bladder calcificaiton & cancer Schistosoma Mansoni causes 195. Presinusoidal HTN, splenomagaly, esophageal varices Snail, intermediate host of… 196. Schistosomiasis Ixodes scapularis transmits 197. Babesia (clinically rembles malaria) & Borelia burgdorferi Nantucket Protozoa 198. Babesia microt Infection by Reduviid Bug 199. Trypansoma cruzi: Chagas’ Disease Infection by TseTse Fly 200. Trypansoma brucei gambiense & rhodiense: African Sleeping Sickness Infection by Sandfly 201. Leishmaniasis: Mucocutaneous Diseases by L. braziliensis & Visceral Disease by L. donovani & Dermal Leishman by L. tropica, mexicana, peruviana Infection by Ixodes Tick 202. Babesia microti: Babesiosis & Borrelia burgdorferi: Lyme Disease www.brain101.info 33 Infection by Anopheles Mosquito 203. Malaria Trophozoites w/ “Face-Like” Appearance 204. Giardia lamblia Nonseptate Hyphae 205. Zygomycosis: Rhizopus & Mucor. Only mycosis w/o septate. Infect Ketoacidotic Diabetics. Histoplasmosis Geography 206. Ohio, Mississippi, Misouri River valleys Coocidioidomycosis Geography 207. Southwestern deserts, California Blastomycosis Geography 208. States east of Mississippi River Paracoccidioidomycosis Geography 209. Latin America Roseola Infection, aka 210. Exanthema Subitum: “Sixth Disease” (Human Herpes Virus-6 dsDNA, enveloped) Herpangina 211. “Hand-Foot-and-Mouth” Disease: Coxsackie A (Picornavirus +ssRNA) Orthomyxovirus 212. –ssRNA, enveloped virus. 213. Spike Glycoproteins (peplomeres): HA = Hemagluttinin & NA = Neuraminidase. These peplomeres are what give the virus antigenis variation 214. Influenza A & B Paramyxovirus 215. –RNA, enveloped. Most common cause of respiratory infections in kids 216. Mumps 217. Croup(Parainfluenza virus) 218. Rubeola(Measles virus) 219. RSV Togavirus 220. +ssRNA, enveloped 221. 3 Day Measles: German Measles: Rubella/ Rubivirus 222. Encephalitis viruses: Alphaviruses: Eastern (more severe) and Western Equine Encephalitis Flaviviris 223. Dengue Fever – icterus & hemorrhage w/ blac vomit 224. Yellow fever 225. St. Louis Encephalitis – no hepatitis or hemorrhage Bunyavirus 226. –ssRNA, enveloped 227. California Encephalitis – severe bifrontal headaches 228. Hantavirus – hemorrhagic fever w/ acute resp. distress syndrome IgA Protease Activity 229. H. Influenzae (needs factors V & X for growth) 230. Strep. Pneumoniae 231. N. meningitidis 232. N. gonnorhoae 233. W/ this activity these bugs are able to colonize the oral mucosa. Diphtheria: ABCDEFG 234. Adenopathy 235. β Prophage encodes the exotoxin 236. Corneybacteria is Club shaped 237. Diphtheria 238. Elongation Factor II 239. Granules (metachromatic) Only ssDNA 240. Parvovirus: “Part of a virus” Only dsRNA 241. Reovirus, “RepeatOvirus” Naked RNA 242. “Naked for CPR”: Calcivirus; Picornovirus; Reovirus 2 circular DNAs 243. Papovavirus & Hepadnavirus BK 244. Papovavirus. Seen in kidney transplant patients (causes renal disease) Hepadna, Retrovirus? 245. No, but has reverse transcriptase Picornovirus: “PERCH” 246. Poiliovirus; Echo; Rhino; Coxsackie; Hep A Hemorrhagic Fevers Filovirus & Bunyavirus (Hantavirus) Segmented viruses All are RNA: Orthomyxo; Arena; Bunya; Reo Eclipse Phase No internal virus. 1 total virus per cell Latent Phase No external virus. Extracellular virus found Naked Capsid Virus Nucleocapsid. DNA or RNA + Structural proteins Enveloped Virus Membrane. Nucleocapsid + Glycoprotein Interferon Non virus specific. Works by RNA endonuclease = digests viral DNA + inh viral prot synth AIDS structural prots Gag, pol, env www.brain101.info 34 AIDS regulatory prots Tat, rev, nef AIDS gp41 env prot Transmembrane AIDS gp120 env prot Surface AIDS p17 gag prot Matrix AIDS p24 gag prot Capsid AIDS p7p9 gag prot Nucleocapsid DNA Viruses A = Adeno E Brick. Rep H = Herpes In Cyto H= Hepadna AH H PPP --- ico Rep in Nuc P = Pox P = Parvo SS P = Papova Circ (+) RNA Viruses E C = Calici P = Picorno R = Reo C P R F T C ------ ico (+) Linear. F = Flavi No segment. Rep in Cyto T = Toga Helical C = Corona R-Tase & Rep in Nuc (-) RNA Viruses 8 F = Filo 2 3 O = Orthomyxo R = Rhabdo F O R P A B ---- (-) E Helical P = Paramyxo Linear. Non seg. A = Arena B = Bunya Bullet Anti sense Hepatitis Window Period After HbsAg disappears & Before HbsAb appears Hepatitis A B C D E Picorna Hepadna Flavi Delta Calici Downey Type II cells EBV Infection by Aedes Mosquito Yellow Fever: Flavivirus: Black vomit, jaundice, high fever “Hot T-Bone stEAk”: ILs IL1 = ↑ Temp: HOT IL2 = stimulate T cells IL3 = stimulate Bone Marrow stem cells’ growth & differentiation (GM CSF) IL4 = stimulate IgE (& IgG) IL5 = stimulate IgA (& eosinophils) ILs Secreted by CD4s IL2, IL4, IL5, IFN gamma ILs Secreted by Macrophages IL1 & TNF α C5a Neutral chemotaxis. When it is w/ C3a, participates in anaphylaxis C5 Convertase When both Alternative and Classic pathways come together Alternative: C3b, Bb, C3b + C3a  C5 Classic: 2b, 3b, C3a + C4b  C5 Only Richettssia not Intracellular Quintana www.brain101.info 35 Plasmodium Life Cycle Sporozoites: from blood to liver Primary tissue schizont Trophozoites: in RBC Erythrocytic schizont Merozoite: ruptured RBC Gametozyte Zygote: inside the mosquito Acanthamoeba Star shaped cysts Mucor, Rhizopus, Absidia Nonseptate, filamentous, 90 degree branching, indian in, capsular halos Cryptococcus Neoformans Monomorphic Candida Yeast normally, pseudo & true hyphae in tissue infections Aspergillus Fumigatum 45 degree branching point, asoc’d w/ cystic fibrosis & burns pt Cocciodes Hyphae in wild. Artroconidia. Arthocondida & Hyphae. Sherules w/ endospores Histoplasma Cap Hyphae in wild. Microcondida w/ tuberculate macrocondida. Fac intracellular. In the tissue it’s a yeast w/ a small neck. Blastomycosis Hyphae in wild Sporothrix Schenkii Hypahe in wild. Potas iodide in milk. Pneumonia in alcoholics. PCP Obligate parasite. Kills type I pneumo cells. Ground glass Gram (-) Bugs w/ Exotoxins E. Coli; V. Cholera; Bordetella Pertussis Dermatophytes Trichophyton: SHN Microsporium: SH Epidermophyton: SN Tinea tavus: permanent hair loss Transmission Diagnosis E. Histolitica Cysts Trophozoites or cysts in stool Giardia Cysts Trophozoites or cysts in stool Cryptosporidium Cysts Acid fast oocysts Balantium C. Cysts Trophozoites or cysts in stool Trichomonas V. Trophozoites Motile trophozoites Fever Fever Spike Vivax Benign 3 degrees 48h Enlarged Host Cell Ovale Benign 3 degrees 48h Oval/Jagged Malariae 4 degrees of Malarial 72hrregular Crescent Falciparum Malignant 3 degrees Miscellaneous 1. Fastest growing tumor – Burkitt’s 2. PE’s are found in half of all autopsies 3. Courvoisier’s Law: tumors that obstruct the common bile duct cause enlarged gallbladders, but obstructing gallstones do not (too much scarring), so if you can palpate the gallbladder you’e probably looking at cancer. 4. Only DNA virus to replicate in cytoplasm: Pox 5. Only RNA virus to replicate in nucleus: Influenza 6. Bacillus anthracis has the only protein capsule 7. Bordetella pertussis (Whooping Cough) elicits lymphocytosis rather than granulocytosis 8. Bronchioalveolar carcinomas grow without destroying the normal architecture of the lung 9. Cryptococcus neoformans often lacks a capsule and, when stained with GMS, looks just like Pneumycistis carinii, except that Cryptococcus lacks the prominent nucleoli. 10. Weil Felix reaction: (+)R. rickettssi & (+)Proteus vulgaris & P. mirabilis 11. Treponema pallidum (Syphilis) tests: 1)VDRL 2)FTA-Abs: most widely used 3)TPI (immobilization test – most expensive but the Gold Standard) www.brain101.info 36 Cytokine Source Function IL 1 12. Monocytes, macrophages Stimulates T cell proliferation & IL2 produciton IL 2 13. Macrophages, T & NK cells Stim prolif of B, T & NK cell IL 3 14. T cells GF of tissue mast cells & hematopoietic stem cells IL 4 15. T cells ↑ growth of B & T cells/ ↑ HLA II Ags IL 5 16. T cells Maturation of B  plasma cell IL 6 17. T cells, monocytes Maturation of B & T cell/ (-) fibroblasts IFN α 18. B cells, macrophages Antiviral activity IFN β 19. Fibroblasts Antiviral activity IFN gamma 20. T & NK cells Antiviral activity, (+) macrophages, ↑ HLA II Ags TNF α 21. Macrophages, T & NK cells T cell prolif, IL 2 prod, cytotoxicity TNF β 22. T cells T cell prolif, IL 2 prod, cytotoxicity Tumor Suppressor Genes Genes Chrom. Associated Tumors VHL 3p Von Hippel Lindau, Renal Cell CA APC 5p Familial adenomatous polyposis, Colon CA WT-1 11p Wilm’s tumor Rb 13q Retinoblastoma, Osteosarcoma BRCA-2 13q Breast CA p53 17p Most human Cas NF-1 17q Neurofibromatosis type 1 BRCA-1 17q Breast CA, Ovarian CA DCC 18q Colon & Stomach CA DPC 18q Pancreatic CA NF-2 22q Neurofibromatosis type 2 = bilateral acoustic neuroma Physio Equations: Resistance in Series: Add all Resistance in Parallel: Invert the answer RENAL: Filtration Fraction = RPF GFR GFR: Glomerular Filtration Rate RPF: Renal Plasma Flow Filtered Load = GFR x [Conc] Excretion Rate = [Urine] x VelUrine Clearance = ] [ ) ( ] [ Plasma Urine xVel Urine or ] [Plasma Excretion Clearance of PAH = [ERPF] ERPF: Eff renal plasma flow www.brain101.info 37 Renal Blood Flow = Hct ERPF − 1 Free Water Clearance = VelUrine - ) ( ) ( ) ( osm P urine xVel osm Urine CARDIO: CO = HR x SV CO = difference VO PulmonaryA consumed O 2 ) ( 2 − Pulse Pressure = Systolic – Diastolic MAP = Diastolic + 1/3 Pulse Pressure CO = TPR MAP MAP = TPR x CO F = R P P 2 1− LUNGS: PAO2 = (760 – 47) FO2 - R PACO2 Where: FO2 = [O2] PACO2 = Alv. Press. Of CO2 R = Resp. Exchange Ratio consumed O produced CO 2 2 ≅ .8 or 1 Flow = difference AtoVO consumed O 2 2 Velgas Diffusion = Thickness Area x Gas Diffusion Constant x Difference of Partial Press VentTot = VentTidal x #of Respirations VentAlv = (VentTidal – VentDead) x # of Respirations Compliance = ess Vol Pr P = Radius Tension 1.0 = Va/Q Diffusing Capacity = 2 PACO COuptake Resp Doubles: 150mmHg & 40mmHg New PCO2 = 20 New PO2 = 170 1 www.brain101.info Neuro Test 1. A 63-year-old man complains of trouble swallowing and hoarseness. On physical exam, he is noted to have ptosis and a constricted pupil on the left, and a diminished gag reflex. Neurological examination shows decreased pain and temperature sensation on the left side of his face and on the right side of his body. Which of the following vessels is most likely occluded? A. Anterior inferior cerebellar artery (AICA) B. Anterior spinal artery C. Middle cerebral artery (MCA) D. Posterior cerebral artery (PCA) E. Posterior inferior cerebellar artery (PICA) 2. A 3-year-old child is brought to the emergency room by her concerned parents. They state the girl has been complaining of a severe headache and has had two episodes of vomiting. On physical examination, there is bilateral papilledema and an impaired level of consciousness. Emergency contrast CT scan demonstrates displacement of the ventricular system by a multilocular "mass" with well-defined white high-attenuation rings around black low- attenuation centers. The lesion involves the cerebellum. To which of the following conditions is this lesion most likely related? A. Bacteremia following tooth extraction B. Bacterial meningitis C. Lung abscess D. Otitis media E. Sinusitis 3. An 80-year-old woman dies after a long history of progressive memory loss, apraxia, and recurrent episodes of confusion. In the last months of life she was bedridden and unable to recognize familiar faces and objects. The pathologist identifies numerous flame-shaped intracytoplasmic inclusions in neurons of the neocortex and hippocampus. These consist of paired helical filaments (PHFs) on electron microscopy. Which of the following biochemical changes most likely accounts for the development of PHFs in this condition? A. Abnormal degradation of amyloid precursor protein (APP) B. Abnormal phosphorylation of tau C. Accumulation of advanced glycosylation end (AGE) products D. Increased expression of APP E. Precipitation of insoluble a-tubuli 4. A 54-year-old white male presents with gradual onset of mild dementia, ataxic gait, and startle myoclonus. An MRI scan is normal, and an examination of his cerebrospinal fluid reveals no abnormalities, but the patient's EEG is remarkable for recurrent bursts of high-voltage slow waves. Over the next 6 months, the patient's dementia rapidly worsens, accompanied by general hypertonicity and profound dysarthria. The patient dies shortly thereafter. Which of the following is the mostly likely neuropathological finding on autopsy? A. Cerebellar hyperplasia B. Diffuse spongiform change C. Multiple lacunar infarcts D. Negri bodies E. Neurofibrillary tangles 5. In which of the following neurodegenerative conditions would you expect to observe the phenomenon known as anticipation? A. Familial Alzheimer disease (FAD) B. Familial amyotrophic lateral sclerosis (ALS) C. Huntington disease D. Pick disease E. Progressive supranuclear palsy 2 www.brain101.info 6. A 37-year-old woman is in a serious automobile accident and sustains a closed head injury. She does not immediately seek medical attention, but is brought to the emergency room two hours later by her brother. On physical examination, there is mydriasis and loss of the pupillary light reflex. Several hours later, she is unable to follow a flashlight with her eyes. Which of the following types of herniation is most likely occurring in this patient? A. Cerebellar tonsils into the foramen magnum B. Cerebellum upward past the tentorium C. Cingulate gyrus under the falx D. Medulla into the foramen magnum E. Temporal lobe under the tentorium 7. A 10-year-old boy with history of epilepsy and mental retardation is brought to a specialty clinic for evaluation. Physical examination is remarkable for several ovoid hypopigmented areas on the trunk and large numbers of red and yellow papules on the face, particularly near the mouth. Biopsy of the papules demonstrates angiofibromata. This patient is most likely to have which of the following central nervous system pathologies? A. Acoustic neuromas B. Capillary hemangioblastomas C. Herniation of cerebellar tonsils into foramen magnum D. Large cortical hamartomas E. Leptomeningeal angiomatosis 8. A 25-year-old man presents with bilateral hearing loss. MRI reveals bilateral tumors within the cerebellopontine angles. Surgery is performed, and the tumors are removed. Both are found to be neurilemomas ("schwannoma"). Which of the following is the most likely diagnosis? A. Metastatic disease B. Multiple sclerosis C. Neurofibromatosis type 1 D. Neurofibromatosis type 2 E. Tuberous sclerosis 9. Which of the following is the most frequent primary malignant tumor of the CNS? A. Glioblastoma multiforme B. Medulloblastoma C. Meningioma D. Oligodendroglioma E. Pituitary adenoma 10. A child develops a tumor of the cerebellum. Biopsy reveals evidence of both neuronal and glial differentiation. Which of the following is the most likely diagnosis? A. Astrocytoma B. Glioblastoma multiforme C. Medulloblastoma D. Meningioma E. Oligodendroglioma 11. Which of the following nuclei undergo transsynaptic degeneration as a consequence of primary degeneration affecting dorsal root ganglia in Friedreich ataxia? A. Cerebellar dentate nucleus B. Contralateral gracile and cuneate nuclei C. Ipsilateral gracile and cuneate nuclei D. Pontine nuclei E. Vestibular nuclei 12. An adult patient presents with persistent headaches. A CT scan of the head demonstrates a 2-cm spherical mass at the junction of the white and gray matter of the lateral aspect of the cerebral hemisphere. Which of the following would most likely produce this lesion? A. Astrocytoma B. Ependymoma 3 www.brain101.info C. Glioblastoma multiforme E. Metastatic carcinoma D. Meningioma 13. Which of the following regarding dementia is true: A. To make the accurate diagnosis of dementia memory impairment is not enough, impairment in every day activities is also required. B. Patients with Alzheimer's disease often have problems with balance and abnormal gait. C. Neuronal loss is not observed in Alzheimer disease D. Finding Lewy bodies is often diagnostic postmortem for Alzheimer's disease E. Progressive Supranuclear palsy is often associated with symptoms of visual hallucinations 14. A 34-year-old man with AIDS suddenly falls to the floor and has a tonic-clonic seizure. His concerned friends call paramedics, who take him to the hospital. On arrival at the hospital he is conscious, but confused. Physical examination is remarkable for cachexia and oral thrush. Neurological examination reveals isolated weakness of lateral gaze on the right. MRI reveals multicentric mass lesions in the brain and meninges. One of the masses is biopsied and appropriate immunohistochemical stains are performed. From which of the following cell types did the masses most likely derive? A. Astrocyte B. B lymphocyte C. Ependymal cell D. Melanocyte E. Oligodendrocyte 15. Which of the following nuclei play a central role in auditory reflexes and descending efferent pathways? A. Superior Olivary Nucleus B. Inferior Olivary Nucleus C. Cochlear Nucleus D. Inferior Colliculus 16. A 63-year-old man has taken an antidepressant for the past 3 months. He is on no other medications and is generally in good health. After attending a graduation party for his son at which he consumes wine, bread, and cheese, he is rushed to the emergency room with tachycardia, headache, and a blood pressure of 200/100 mm Hg. Which antidepressant is he most likely taking? A. Fluoxetine B. Imipramine C. Phenelzine D. Trazodone E. Venlafaxine 17. Which of the following statements regarding the nasal anatomy is false? A. Obstruction of the ostiomeatal complex results in congestion of the Anterior ethmoid, frontal and maxillary sinuses. B. The most common site of bleeding is from Kesselbach's plexus C. The primary form of treating allergic rhinitis is Avoidance D. Triad asthma (which includes the symptoms of asprin sensitivity, asthma, and polyps) should lead to a suspicion of cystic fribrosis. E. All of the above statements are true. 18. Wegener's disease is associated with which of the following nasal conditions? A. Saddle Nose Deformity B. Septal perforation C. Allergic Rhynitis D. Cystic Fibrosis E. A and B 4 www.brain101.info F. A and D G. B and D H. B and C 19. A 52-year-old female complains of sudden visual abnormalities. Her history reveals a 30 pack-year history of smoking, hypertension, and hypercholesterolemia. A head CT shows a lesion in the right occipital lobe and an angiogram reveals an embolic stroke of the right posterior cerebral artery. What type of visual deficit is she most likely experiencing? A. Bitemporal hemianopia B. Central scotoma C. Left homonymous hemianopia D. Left superior quadrantanopia E. Right homonymous hemianopia F. Right superior quadrantanopia G. Total left eye blindness H. Total right eye blindness 20. A 38-year-old woman with multiple sclerosis (MS) has stable neurologic deficits resulting from old demyelinated plaques. Which of the following histopathologic features would be prominent in this patient's old plaques? A. Complete loss of axons B. Gliosis C. Histiocytic infiltration D. Lymphocytic infiltration E. Myelin breakdown 21. A patient with long term severe hypertension develops progressive dementia. CT scan of the head demonstrates a diffuse loss of deep hemispheric white matter. Which of the following terms best describes the pathological process that is occurring? A. Anemic infarcts B. Hemorrhagic infarcts C. Hypertensive encephalopathy D. Lacunae E. Subcortical leukoencephalopathy 22. A 40-year-old man with adult polycystic kidney disease is brought to the emergency room in a coma. CT scan of the head demonstrates a subarachnoid hemorrhage without parenchymal hemorrhage. Which of the following is the most likely source of the bleeding? A. AV malformation B. Bridging veins C. Charcot-Bouchard aneurysm D. Circle of Willis E. Middle meningeal artery 23. During a boxing match, a contestant is "knocked out" by a blow to the lateral skull. He recovers after a few minutes, and is asymptomatic for the next 12 hours. He then develops a severe headache, changes in mental status, nausea, and vomiting. Which of the following is the most likely diagnosis? A. Basilar skull fracture B. Epidural hemorrhage C. Intracerebral hemorrhage D. Subarachnoid hemorrhage E. Subdural hematoma 24. A 60-year-old man presents to a physician because of difficulty in reading and coming down stairs, which he attributes to an inability to "look down." Physical examination reveals that the patient looks around by moving his head rather than his eyes and also shows a distinctive axial rigidity of neck, trunk, and proximal limb muscles. He shows poverty of movement and dysarthric speech. Mentally, the patient responds very slowly but has better memory and intellect than are initially apparent. Which of the following pathologic findings of the brain would most likely be present? A. Depigmentation of the substantia nigra and locus ceruleus B. Diffuse cortical atrophy with relative sparing of primary motor and sensory areas C. Selective frontal and temporal lobe atrophy D. Striking degeneration of the caudate nucleus E. Widespread neuronal loss and gliosis in subcortical sites 5 www.brain101.info 25. A neurological examination of a 47-year-old woman reveals a normal corneal reflex in her right eye, but no consensual corneal reflex in her left eye. Which of the following additional findings might be expected? A. Absence of pupillary light reflex of the left eye B. Hyperacusis of the left ear C. Inability to abduct the right eye D. Loss of pain and temperature of the left face E. Loss of taste from the anterior two-thirds of the right tongue F. Ptosis of the left eye 26. A 64-year-old man begins to show behavioral changes and irritability, and is found wandering in the park near his home. On neurological examination, there is evidence of mild aphasia and cognitive dysfunction, but motor function is preserved. CT scan of the head demonstrates selective atrophy of the cortex of the frontal lobes. Which of the following is the most likely diagnosis? A. Alzheimer's disease B. Friedreich's ataxia C. Huntington's disease D. Parkinson's disease E. Pick's disease 27. Careful testing of the visual fields in a patient complaining of difficulty reading demonstrates a central scotoma involving one visual field. This defect is most likely due to a lesion involving which of the following structures? A. Macula B. Optic chiasm C. Optic radiations in the parietal lobe D. Optic radiations in the temporal lobe E. Optic tract 28. In the evaluation of a 73 year old man who complains of frequent confusion and disorientation. His neighbor has noticed that he no longer is keep up his garden and is concerned about his frequent wandering in the neighborhood. She also noticed that he often looks disheveled, which is particularly abnormal for him. All of the following lab exams would be relevant in the assessment for dementia except: A. Blood sample for CBC, glucose, electrolytes and LFT's. B. Imaging studies such as CT and MRI C. Urinanalysis D. Presenilin (sequencing) E. All are appropriate 29. A 76-year-old female presents with complaints of difficulty reading. Ophthalmologic examination is remarkable for bilateral, white opacifications in her eyes, consistent with cataract formation. In which of the following structures are the opacifications located? A. Aqueous humor B. Cornea C. Lens D. Optic nerve E. Retina 30. A middle-aged woman comes to her physician's office with complaints of visual difficulties. A review of systems and physical examination are unremarkable except for her eye exam. When a light is shined in her right eye, there is no pupillary response in either eye. However, upon shining a light in her left eye, both ipsilateral and contralateral pupillary responses are apparent. Her extraocular movements are intact. What is the most likely location of her lesion? A. Oculomotor nerve, left side B. Oculomotor nerve, right side C. Optic nerve, left side D. Optic nerve, right side E. Trochlear nerve, left side F. Trochlear nerve, right side 6 www.brain101.info 31. In attempting to introduce a catheter into the right internal jugular vein, a resident inadvertently damages the cervical sympathetic trunk in a patient. Which of the following findings is most likely to be seen in this patient as a result of the injury? A. Constriction of the right pupil B. Dilation of the right pupil C. Inability to abduct the right eye D. Inability to close the right eye E. Paralysis of the platysma muscle on the right side 32. During a follow-up visit to her psychiatrist, a 17-year-old female admits that she is contemplating suicide. She has been morbidly depressed for the past 2 months; she denies drug abuse or excessive drinking. After a complete evaluation, her physician prescribes imipramine. This drug acts by A. increasing dopamine release B. inhibiting monoamine oxidase C. inhibiting the reuptake of serotonin and norepinephrine D. potentiating GABA E. selectively inhibiting the reuptake of serotonin 33. If an Acoustic Reflex Test was being performed on the right ear of a normal man with a loud sound we would expect a tympanogram from the right ear to have a: A. Higher than normal compliance peak B. Lower than normal compliance peak C. A shift in the compliance peak to the left D. A shift in the compliance peak to the right E. no change 34. Which of the following statements are true regarding hearing aids: A. Amplified sound does not have the same fidelity B. completely in the canal hearing aids are reserved only for patients with severe hearing problems C. Bond conduction aids are useful for people with conductive hearing loss D. All of the above are true E. None of the above are true 35. You have a patient who was diagnosed with a large pituitary tumor. Which of the following visual fields most likely would be affected? A. Right nasal field and left temporal field B. Left nasal field and right temporal field C. Right nasal field and left nasal field D. Right temporal field and left temporal field E. Right nasal and right temporal field 36. An elderly patient dies with chronic dementia. At autopsy, the brain shows diffuse cortical atrophy with relative sparing of primary motor and sensory areas. Which of the following would most likely be a prominent feature on microscopic examination of her brain tissue? A. Central chromatolysis B. Lewy bodies C. Gliosis of the caudate nucleus D. Loss of pigmented neurons E. Neurofibrillary tangles 37. In the patient above (Q. 36) which types of cells are most likely affected? A. GABAnergic neurons B. Cholinergic neurons C. Dopaminergic neurons D. Glutamatergic neurons E. Glial cells 7 www.brain101.info 38. A 6-year-old girl manifests acute vomiting and nuchal rigidity. MRI reveals a tumor in the posterior fossa consisting of a large cyst with a nodular mass attached to its wall (cyst with "mural nodule"). Histologic examination shows elongated astrocytes with long bipolar processes and numerous Rosenthal fibers. Which of the following is the most likely diagnosis? A. Astrocytoma, WHO grade II B. Ependymoma C. Glioblastoma multiforme D. Medulloblastoma E. Pilocytic astrocytoma 39. A 35-year-old man is referred to a psychiatrist because of erratic behavior. The man had been adopted in infancy, so his family history is not known. Over the next year, he develops uncontrollable erratic movements, such that attempts to pick up a cup or use a pencil produce sudden uncontrolled lurches. When he tries to walk, he staggers, thrusts, and abruptly changes direction. Eventually, with disease progression, he develops increasing rigidity and is unable to move, and finally dies ten years after the onset of symptoms. Which of the following changes would most likely be seen on examination of his brain at autopsy? A. Depigmentation of the substantia nigra and locus ceruleus B. Diffuse cortical atrophy with relative sparing of primary motor and sensory areas C. Selective frontal and temporal lobe atrophy D. Striking degeneration of the caudate nucleus E. Widespread neuronal loss and gliosis in subcortical sites 40. All of the following appear to play a role in depression except: A. Puberty B. Oral Contraception C. Pre mentrual D. Time Zone changes E. Menopause 41. Which of the following drug is most appropriate for treating a unipolar patient? A. Fluoxetine B. Lithium C. Oxazepam D. Clozapine E. Midazolam 42. Which of the following statements are true regarding the tensor tympani and stapedius muscles? A. They are similar in that they are both innervated by the facial nerve B. They are similar in that they are both innervated by the trigeminal nerve C. They are different in that the stapedius muscle is innervated by the CN. V and tensor tympani is innervated by CN. VII D. They are similar in that they both act to dampen sound. They both act in opposition of each other; the stapedius dampens sound and tensor tympani amplifies sound 43. A 48-year-old male is brought to the psychiatric emergency room after an attempted suicide. He claims to hear voices telling him to kill himself. The patient's family notes that he has been on several different kinds of antipsychotic medications, with no improvement of his symptoms. The attending psychiatrist places the patient on a new medication, and admits him. One week after therapy has begun, a routine blood test reveals profound depletion of polymorphonuclear leukocytes. Which of the following drugs is most likely responsible for these symptoms? A. Chlorpromazine B. Clozapine C. Fluoxetine D. Haloperidol E. Imipramine F. Phenelzine 8 www.brain101.info Answers 1 2 3 4 5 6 7 8 9 10 11 12 13 14 15 16 17 18 19 20 21 22 23 24 25 26 27 28 29 30 31 32 33 34 35 36 37 38 39 40 41 42 43 e d b b c e d d a c c e a b a c e e c b e d e e b e a d c d a c b a d e b e d e a d b 7. The correct answer is D. The disease is tuberous sclerosis. The facial angiofibromata are also called adenoma sebaceum, and the hypopigmented patches on the trunk are called ash-leaf spots. This disease is one of the neurocutaneous disorders called phacomatoses. Tuberous sclerosis is inherited as an autosomal recessive trait, and epilepsy and mental retardation are commonly seen in this disorder. Large, firm, white hamartomatous nodules (tubers) are seen in the cortex and in subependymal sites. The tubers consist of aberrantly arranged neurons and/or glia. Patients may also have pancreatic cysts, renal angiomyolipomas, and cardiac rhabdomyomas. Rarely, an astrocytoma will arise in a tuber. 9 www.brain101.info 24. The correct answer is E. The disease is progressive supranuclear palsy, a degenerative disorder characterized by ophthalmoplegia, pseudobulbar palsy, axial dystonia, and bradykinesia. The presentation described in the question is typical. The pathologic changes consist of widespread neuronal loss and gliosis in subcortical sites with sparing of the cerebral and cerebellar cortices. 26. The correct answer is E. Pick's disease is a condition that is clinically similar to Alzheimer's disease. It differs from Alzheimer's disease in that the pronounced brain atrophy characteristically involves the frontal and temporal lobes, with sparing of the posterior aspects of the cortex. Microscopically, the affected cortex contains characteristic ballooned neurons (Pick cells) or cytoplasmic inclusions (Pick bodies). Clinically, there is a slowly progressive dementia with language disturbances and behavioral changes that may eventually lead to mutism. The progressive nature of the dementing process has been termed a "descent into a sea of mindlessness." CHOLINOMIMETIC: Direct StimuIating Name ADMIN INFO USE/CONTRA/RXN's Choline Ester DYNAMICS: eye - miosis, facilitates the outflow of aqueous humor; CV - Hypotension and brady, reflex tachy {4 effects 1) vasodilation 2) dec in cardiac rate 3) dec conduction of SA and AV 4) dec in force of cardiac contraction}; Resp - bronchospasm, inc tracheobronchial secretions; GI - inc peristalitic act and relaxation of sphincter, inc salivary and gastic secretions; GU - stim of detrusor muscle and relax the trigone and sphincter; Secretetory glands - stim of secretory activity of the lacrimal, tracheobroncial, salivary, digestive, and exocrine; CNS - nicotine has mind altering action, tremors, emesis, and stim of resp centers, cunvulsions, and fatal coma; NM junct - nicotinic effect - fasiculation to strong contraction of entire muscle - lead to depolarization blockade CONTRA: Asthma, hyperthyroidism (inc receptors), Coronary insufficiency, acid- peptic disease. TOXICITY: pilocarpine and choline esters: Nausea, vomiting, diarrhea; salivation, sweating; cutaneous vasodilation, tremors, bronchial constriction, AV block, bradycardia, and PVC's. TOXICITY: nicotine: acute - CNS stimulation, Hypertension, Cardiac arrhythmias, convulsions, coma, neuromuscular blockade resulting in resp paralysis: Chronic exposure - recurrance of ulcer in pts with peptic ulcers Acetylcholine* - no therapeutic value; forms from glucose and choline ÌV, Oral Rapid hydrolysis by Ach-esterase and plasma esterase - butylcholinesterase; rapid hydrolysis in GÌ tract; CNS penetration limited USE: diagnostic tool, no therapeutic value Methacholine - clinical apps twice as resistant to hydrolysis; > muscurinic activity then nicotinic; See effects when administered subcutaneously USE: diagnostic tool for broncial hyperactivity Bethanechol - clinical apps; carbomic esters = long acting - 6 hrs activates bowel and bladder smooth muscle; mainly muscurinic action USE: postoperative abdominal distention and gastric atony; postoperative and postpartum urinary retention; promotes salivation Carbachol - clinical apps; carbomic esters = long acting - 6 hrs substantial nicotinic activity (especially on autonomic ganglia, therefore can activate the adreanal medulla USE: glaucoma Natural Alkaloids Pilocarpine Well absorbed; from leaf of SA shrub pilocarpus; muscurinic exceeds nicotinic; Hypertension after a brief hypotensive responce USE: glaucoma Muscurine from poisonous mushroom (amanita muscuria); muscurinic receptor site Arecoline from areca or betel nuts (areca catechu); nicotinic receptors Notes by T. Chengot - 2002 Page 1 CHOLINOMIMETIC: Indirect StimuIating (AntichoIinesterases) - accumulates ACH DYNAMICS: Muscurinic Effects - miosis, bronchoconstriction, inc secretions, inc peristaltic activity, brady, hypotension. Nicotinic Effects - dose dep effect - higher dose results in fibrillation of muscle fibers, fasciculation, and non- depolarizing blockade. CNS Effect - dose dep effect - low conc. results in diffuse activation of the ECG and subjective alerting response. High conc. results in generalized convulsions, coma, and resp arrest. Name ADMIN INFO USE/CONTRA/RXN's Reversible - Carbamoyl Esters Physostigmine carbamate ester that covalently bonds with Achesterase. KINETICS: well absorbed (lipid soluble); duration 1/2 - 60 hrs USE: in glaucoma, Alzheimer's disease, atropine overdose Neostigmine - also has some direct nicotinic agonist activity carbamate ester that covalently bonds with Achesterase. KINETICS: poorly absorbed; post-op; duration 1/2 - 6 hrs USE: adynamic ileus, non-obstructive urinary retention, diagnostic tool and treatment of myasthenia gravis, Nondepolarizing neuromuscular blockade reversal Edrophonium - quaternary alcohol forms reversible electrostatic and hydrogen bonds to active site of the enzyme ~ short duration; diag-X-short; 2-10 min duration USE: diagnostic tool for myasthenia gravis Pyridostigmine [Mestinon] KINETICS: poorly absorbed Donepezil [Aricept] - 1st piperidine- type reversible cholinesterase inhibitor; > affinity for CNS, therefore less peripheral effects oral: t1/2 = 70 hrs KINETICS: 1st pass 100%; metabolized CP450 2D6 and 3A4 and glucoronidation; excreted in urine (17% unchanged), no food interaction, peak levels 3-4 hrs, once a day dosing. TOXIC: GÌ - nausea, vomiting, diarrhea, anorexia. Also reported - headache, pain (various locations), syncope, ecchymosis, weight loss, arthralgia, insomnia, dizziness, dyspepsia, and increase urine frequency USE: alzheimer's disease. CONTRA: GÌ bleeds and hepatic disease (for cautionary measures - not hepatotoxic but could be) Tacrine [Cognex] - has direct agonist activity (both direct cholinomimetic and reversible inhibition of ACH-esterase oral: t1/2 =1.5- 4 hr KINETIC: 1st pass 5-30%; metabolized CP4501A2; excreted in urine, food dec absorption by 30-40%, peak levels 1-2 hrs. TOXIC: hepatic* - elevated liver enzymes; GÌ* - nausea, vomiting, diarrhea, anorexia, constipation, dyspepsia, flatulence; CNS* - myalgia, syncope, dizziness, tremors, insomnia and mental status change; also seizures, asthma, exacerbations [*-expected] USE: alzheimer's disease. CONTRA: GÌ bleeds and hepatic disease Irreversible - Organophosphates Bind with phosphates and form covalent phosphorus-enzyme complex that may undergo 'aging' where bond is strengthened Parathion Malathion Ìsoflurophate [Floropryl] USE: in glaucoma Echothiophate [Phospholine] KINETICS: not well distributed in body USE: in glaucoma Treatment for Poisoning: Antidotes 80% exposure to pesticides - occupational exposure, dermal absorption 1) CV support 2)Decontamination 3) TX of seizures (anti siezure meds) Atropine antagonist to Ach Pralidoximine (2-PAM) [Protopam] effect in 24 hr (best), no later then 48 hrs; Ach-esterase reactivator pesticides - KINETICS: highly toxic b/c not detoxed in vertebrates; highly toxic pesticide - absorbed dermally decrease effect of too much Ach Notes by T. Chengot - 2002 Page 2 AntichoIinergics Name ADMIN INFO USE/CONTRA/RXN's Belladonna Alkaloids Atropine* - From atropa belladonna {nightshade]; Reversible Competitive antagonism of Ach @ muscurinic receptor sites (however non specific for subtypes); low potency at nicotinic; @ high conc. Able to produce partial blockade -PROTOTYPE Oral, Conjunct: t 1/2 = 2-4 hr KINETICS: Well distributed 30 min - 1 hr; well absorbed GÌ and conjunctival; metabolism liver, excretion - 60% unchanged in urine. DYNAMICS: CNS - dose dependent - @ low dose mild vagal excitation as a result of stim of medulla and higher cerebral centers. Ocular - mydriasis, cycloplegia, reduction of lacrimal secretion; CV - tachy; Blood Vessels - moderate to toxic doses cause atropine flush; Resp - bronchodilation and decreased secretory gland secretions, dec mucillary clearance; GÌ - dec motility, reduction in gastric secretions, dec salivation; GU - relax sm of ureter and bladder wall (reduce voiding); sweat glands (suppressed)- thermoreg, inc body temp. TOXIC: CNS - restlessness, irritability, disorientation, hallucinations, and delirium; @ inc conc. - circ collapse, paralysis of parasym organs, coma, death CONTRA: glaucoma, prostate hyperplasia, gastric ulcer. SIDE EFFECTS: dry as a bone, blind as a bat, hotter then a hen red as a beet, mad as a hatter, plus tachy Scopolamine [Transderm- Scop] - greatest CNS penetration dermal patch ≈ to Atropine except the CNS toxicity; @ thereputic doseage = depression; TOXIC: CNS - drowsiness, amnesia, fatigue USE: treatment of motion sickness Semisynthetic/Synthetic Derivatives of the Belladonna Alkaloids *QUATERNARY: mech is same as atropine w/ greater nicotinic ganglionic blocking resulting in adverse rxn that are attributable to ganglionic blockade. KINETICS: Absorption - poorly and unreliable after oral (10-30%) absorption; distribution - more ionized ∴ less cross @ BBB and conjunctival barrier; duration of action longer. PHARM: less CNS penetration. TOXIC: = to belledonna, but less CNS effects; significant ganglionic blockage produce sig nicotinic side effects; OD leads to curariform NM block - respiratory paralysis. *TERTIARY: better lipid soluability, useful in ophthalmic and centrally mediated diseases. TOXIC: equal to that of natural products, systemic effects possible, but less likely. Ìpratropine [Atrovent] - Quaternary inhalation (<1% absorbed, 90% swallowed): max response 30-90 mins, 4 hr duration, eliminated in feces; PHARM: bronchodilations with no effect on mucillary clearance USE: COPD Methscopolamine [Pamine] - Quaternary less potent then atropine, poorly absorbed, duration 6-8 hrs; derivative of scopolamine w/o central effects (different then atropine - MECH unknown) USE: Gastrointestinal diseases (adjunct to treating ulcers - decrease secretion) Benzatropine [Cogentin] - tertiary similar to trihexyphenidyl but less CNS effects; synthetic compound similar to atropine and diphenhydramine; [has anti-muscurinic, anti-histaminic, and local anesthetic effects. ] USE: parkinsonism and drug induced extrapyramidal side effects Tropicamide [Mydriacyl] - tertiaryξ Cyclopentolate [Cyclogy] - tertiaryξ Dicyclomine [Bentyl] - tertiary less potent the atropine and minimal effect on salivary or sweat glands or CV system USE: irritable bowel disease [antispasmodic properties] Homatropine hydrobromide [Ìsopto Homatropine] - tertiary topical, ophthalmic semisynthetic derivative of atropine; USE: topical administration [ophthalmic formula]; Mydriasis, induction of cycloplegia topical, ophthalmic shorter duration then atropine, scopolomine or homatropine USE: topical administration [ophthalmic formula]; Mydriasis, induction of cycloplegia Notes by T. Chengot - 2002 Page 3 Oxybutynin [Ditropan] - tertiary synthetic antimuscurinic compound - has nonspecific relaxation effect USE: adult urinary incontinence and neurogenic bladder (relaxation of sphincter) Tolterodine [Detrol] - tertiary M3 receptor selective; effects similar to oxybutynin, but lipophilicity is 30x less - limited CNS penetration - less CNS side effects USE: Adult urinary incontinence Trihexyphenidyl [Artane] - tertiary CNS permeability exploited; synthetic compound similar to atropine USE: parkinsonism and drug induced extrapyramidal side effects Notes by T. Chengot - 2002 Page 4 Drugs effecting the GangIia Name ADMIN INFO USE/CONTRA/RXN's Indirect Ganglionic Blocker Nicotine [Nicoderm, Nicotrol, Hibitrol, Nicorette] absorbed from resp tract, buccal mem, and skin; ionized in stomach: t1/2 = 2 hr DYNAMICS: Ganglionic Blockade - initial stim, followed by depression; NM junction initial stim followed by paralysis, NM bloackade by receptor desensitization; CNS - stim followed by depression, dose dep tremors, convulsions, resp stim followed by depression = death - central paralysis and peripheral blockade; CV - vasoconstriction, tachy, and elevated BP; GÌ - parasym stim, nausea, vomiting, and occasional diarrhea; Exocrine glands - initial stim of salivary and bronchial secretions then inhibition. KINETIC: majority of absorption in sm intestines; 80-90% altered in body by liver (also in kidney, lung); Renal elimination - dependent on pH b/c it is basic; found in breast milk. TOXIC: acute exposure - accidental ingestion/sprays, nausea, salivation, abdominal pain, vomiting, diarrhea; cold sweat, headache, dizziness; disturbing hearing and vision, mental confusion, and marked weakness, hypotension, resp diff, CV collapse, terminal convulsions, death due to resp failure. *inhaled nicotine - metabolized mainly by the lungs Ganglionic Blockers Mecamylamine [Ìnversine] - non- competitive antagonist at several nicotinic acetylcholine receptor subtypes oral KINETICS: absorption decreased by effect of ganglionic blockade on the GÌ tract; readily enters CNS; accumulates in liver and kidney, excreted slowly by kidney in unchanged form - long duration of action. PHARM: CNS - sedation, tremors, choreiform movements, mental aberrations reported; Eye - cycloplegia, moderate mydriasis; CV - hypotension, ortho hypo, tachy; GÌ - reduction in secretions, inhibition of motility, GU - hesitancy in urination, impaired erection; loss of thermoregu control. TOXIC: exacerbation of side effects, visual disturbances, dry mouth, urinary hesitancy, moderate constipation, abdominal discomfort, anorexia, heartburn, nausea, postural hypotension; SEVERE RXN: - marked hypotension, constipation, syncope, paralytic ileus, urinary retention, cycloplegia, CNS - tremors, mental confusion, seizures, mania, or depression USE: hypertensive emergencies CONTRA: history of cardiac problems, uremia, glaucoma (b/c moderate mydriasis), organic pyloric stenosis. Notes by T. Chengot - 2002 Page 5 Drugs effecting the NeuromuscuIar Junction BIockers Depolarization results in the Na+ and K+ influx; Ach is metabolized by ACH- esterase; Transmission inhibited by competative agaonitst of Ach receptors (nondepolariziing blockade; Excessive agonist result in initial depolarization and desensativation Name ADMIN INFO Indication/CONTA Depolarizing (Non competitive Antagonist) Succinylcholine* - 2 Ach molocules together - PROTOTYPE parenteral, ÌM, ÌV Mech: PHASE 1: depolarization - binds to post synaptic nicotinic receptors, not metabolized in the synapse - remains depolarized resulting in flaccid paralysis PHASE 2: despite continued exposure, endplate becomes repolarized, AND desensatived to stimulation. KINETIC: duration 5-10 mins, ionized - no X @ BBB, rapid hydrolysis by plasma cholinesterases @ liver and plasma (effects prolonged in pts w/i abnormal plasma choilesterase. DYNAMICS: skeletal muscle paralysis (1st fasiculations over chest and abdomen, 2nd arms legs and neck, 3rd respiratory muscle weakness); CV - arrhythmias and bradycardia; hyperkalemia - leads to cardiac arrest; inc intraocular pressure, inc intragastric pressure - emisis, muscle pain. TOXIC: muscle fasiculations, tremors, jaw rigidity, postoperative myalgia, muscular weakness/pain, renal tubular obstructuon, muscle paralysis, prolonged apnea & CV collapse, hpersensativity, malignant hyperthermia (ine Ca release - muscle rigidity, Tx Dantrolene [Dantrium] ) DRUG INTERACTIONS: Antibiotics - exacerbate or potentiate; local anesthetics - enhance NM transmissions; non- depolarizing NM junct blocker - 50% more needed for effect. CONTRA: malignant hyperthermia, burns (extra inc in K+), cardiac disease. [Malignant hyperthermia - autosomal dominant; release of inordinaent Ca++ from sarcoplasmic reticulum; Tx - w/ Dantrolene {Dantrium} - controls levels of Ca++] Non-Depolarizing (Competitive Antagonist) Tubocurarine* - PROTOTYPE ÌV long acting non depolarizing NM junction blocker; causes release of greatest amount of histamine. MECH: competative antagonism of Ach @ nicotinic receptor sites, higher dose - block the ion channels, theory - block prejunctional Na channels interfering w/ Ach release, BLock reversed by lg amts Ach or choinesterase inhibitors. KINETICS: onset 2-3 mins, distribution <1.5 hr, duration 30-60 min, limited VD of 80 - 140 ml/kg, high ionized - not X BBB, excreted unchanged in bile/urine. DYNAMICS: Skeletal Muscle - initial weakness - then paralysis, sm then lg effetced, last is diaphram - recovery is in reverse order; CV - hypotension, liberation of histamine - WHEALS when ÌntraQ or ÌA, bronchospasm, hypotension, excessive bronchial and salivary secretion, histamine relase is direct mast cell effect. TOXIC: histamine release + ganglionic blockade - rash and puritis, (flushing, edema, urticaria, puritus, erythema - rare), hypersalivation & bronchospasm, wheezing and arrhythmias, hypotension; sinus tachy; muscle paralysis, apnea, dyspnea, and resp depression, malignant hyperthermia USE: for surgical anesthesia (intubation), diagnostic tool in myasthenia gravis. DRUG INTERACTIONS: inhaled anesthetics, antibiotics - inc Ach release; local anesthetics; non-depolarizing NM junct blocker - more needed for effect. ANTIDOTE: cholinesterase inhibitors (neostigmine - b/c inc Ach competatively). Atracurine [Tracrium] - intermediate acting ÌV produces a mild/moderate histamine release; readily crossed BBB, laudanosine is a toxic metabolite (responsible for seizures); less potent then Pancuronium or Vecuronium; has faster recovery time compares to tubocurarine or Pancuronium; less hypotension then Tubocurarine. USE: adjunct to general anesthesia, tracheal intubation Mivacurium [Mivacron] - Shorter acting ÌV short acting; rapidly metabolized by plasma cholinesterase (inc duration in pts w/ renal failure due to dec plasma levels); duration less then Atracurium, less hypotention then Tubocurarine USE: endotracheal intubation, neuromuscular blockade Notes by T. Chengot - 2002 Page 6 Pancuronium [Pavulon] - long acting ÌV long acting agent, minimal histamine release, metabolized in liver to active metabolite, excreted in urine; potency is 5x Tubocurarine USE: endotracheal intubation, neuromuscular blockade Vecuronium [Nocuron] - Ìntermediate acting intermediate acting agent, minimal histamine release and is less likely to cause side effects (as opposed to Atrcurium, Mivacurium, Succinylcholine, Tubocurarine) , meabolized by liver; excreted in bile (85%) and urine (15%); more potent the Pancuronium or Atracurium USE: endotracheal intubation, neuromuscular blockade Notes by T. Chengot - 2002 Page 7 Sympathomimetic Drugs Thereputic uses: Shock, Hypotension, Hypertension, Cardiac Arrhythmias, Congestive Heart Failure, Limiting Hemorrhage (topical vasoconstictor effects), inc duration of action of local anesthetics, nasal decongestion, asthma, allergic rxn, opthalmic indications: mydiriasis, narcolepsy, weight reduction, ADHD Name ADMIN INFO USE/CONTRA/RXN's Direct acting Epinephrine* - endogenous catacholamine parenteral, inhalation MECH: potent stimulant of both β and α adernergic receptors; KINETICS: metabolized by hepatic COMT and MOA; excretion urine; slow absorption following subQ - local vasoconstricion. DYNAMICS: CV - powerful stimulant w/ direct activity of β1 receptors, hypotension (potent vassopressor), raise systolic > then diastolic, inc CO, inc O2, arrhythmias(stim different pacemakers of the heart); Vasoconstriction; Smooth Muscle - relaxation of GÌ smmoth muscle, inhibits uterine tone and contraction, uriniary hesitancy and retension;Resp - relaxation of bronchial muscles, bronchodilation, negligable CNS due to polarity. TOXIC: fear, anxiety, tenseness, restlessness,; headaches, weakness, dizziness, pallor; resp difficulty; palp; hypertension (esp w/ hyperthyroid & hypertensive; cerebral hemorrhage and cardiac arrhythmias - depends on administration METABOLIC EFFECTS: elevation of glucose and lactate blood levels, inc O2; (other effects stim lacrimation, mydrisis). CONTRA: pts who receive nonselective b- andrenergic blocking drugs - see only alpha response. norepinephrine [Levophed] - endogenous catacholamine (ineffective oral) poor subQ 95% β and 5% α; blood flow reduced to kidney, liver, and skeletal muscles. KINETICS: rapid met by COMT and MAO; small amount excreted in urine unchanged, ineffective orally; DYNAMICS: equipotent to epi @ β1 receptor, lil action @ β2; potent agonist at α (<epi) however, it is less potent then epi on the α receptors of most organs; CV - hypertension w/ inc systolic and diastolic pressures and pulse preassure are inc, marked vascoconstriction, inc total peripheral resistance, dec HR due vagal reflex, peripheral vascular resistance inc (blood flow reduced to kidney liver and skeletal muscle); hyperglycemia (<epi), inc. freq of contraction in pregnancy. TOXIC: ~ to epi, less pronounced and less frequent, anxiety, resp diff.; awaeness of slow, forceful beating heart, transient headaches; O.D. - severe hypotension, photophobia, retrosternal pain, pallor, intense sweating, and vomiting. CONTRA: anesthesia with agents that sensatize the automatic tissue of the heart (Halothane); pregnancy Ìsoproternol [Ìsuprel] - β adernergic agonist (low affinity to for α - adernergic receptors) - potent synthetic parenteral or inhalation KINETICS: metabolized by COMT in liver & others, poor substrate for MAO; longer duration then Epi and norE. DYNAMICS: CV - lowers peripheral vascular resistance, hypotension, inc CO (+ inotropic and chronotropic effect); relaxation bronchial and GÌ smoother muscle. TOXIC: palpatations, tachy, headache, fluched skin (common); lipolysis = epi METABOLIC EFFECTS: less hyperglycemia then epi'; lipolysis equal to epi; Dopamine [Ìntropin] ÌV metabolic precursor of norE and Epi; KINETICS: substrate for COMT and MAO - ineffective orally; DYNAMICS: CV - vsodilation (low conc. - inc GFR, renal blood flow, and Na+ excretion); positive inotropic effects on the myocardium acting on β1 receptors and stimulates release of norE; Less tachy then Ìsoperternol; Ìnc systolic blood pressure and pulse pressue (min effect on diastolic); stim of vascular α1 (high dose) - adernergic receptors leading to vasoconstriction and dec renal blood flow and urinary output; no CNS effects - no X BBB. TOXIC: Nausea, vomiting; tachy, anginal pain, arrhythmias, hypertension, vasoconstriction; headache USE: Shock; Oliguria and with low or normal pheripheral vascular resistance Notes by T. Chengot - 2002 Page 8 Doubutamine [Dobutrex] - β adernergic agonist (stuct sim to Dopamine) DYNAMICS: inc CO(>isoperternol); TOXIC: hypertension and tachy, facillitates AV conductance, tolerance, myocardial infacts Clonidine [Catapres] - α2 adernergic agonist oral: t1/2 = ~12 hrs KINETIC: peak plasma concentration 1-3 hr;renal elimination (50% unchanged); ~100% bioavailability. PHARM: CV - transient hypertensive response followed by prolonged hypotension; Bradycardia. TOXIC: Dry mouth and sedation (50%); Sexual dysfunction; Marked Brady; Contact dermatitis (15- 20%); withdrawl rxn w/ abrupt discontinuation of long term therapy USE: Hypertension, adjunct in narcotics, alcohol, and tobacco withdrawl Indirect acting Amphetamine♠ − non specific and Promotion of the release of Norepinephrine CNS stimulant; MECH: releases biogenic amines from their storage sites in nerve terminals. DYNAMICS: CV effect - hypertention, reflex brady, arrhythmias (large doses); Effect on smoother muscle is similar to other sympathomimetic drugs; CNS stim, dec fatique, elevation of mood, euphoria, inc motor activty and speech; Resp stim; appetite suppressent (hypothalimic activity on feeding centers). TOXIC: CNS STIM - restlessness, dizziness, tremors, hyperactive reflexes, irritability, weakness, insomnia, fever, and euphoria; confusion, aggressiveness, anxiety, delirium, paranoid hallucinations; CNS DEPRESSION - fatique and depression; CV effects - headaches, pallor, flushing, palpatations, cardiac arrhythmias, anginal pain, blood pressure changes, and circ collapse; GI - dry mouth, metallic taste, anorexia, nausea, vomitting, diarrhea, and abdominal cramps; Convusions, coma, and cerebral hemorrhages; psychological dependence adn tolerance Methamphetamine [Desonxyn] - β2 adernergic agonist Dextroamphetamine [Dexedrine] 3x greater potency then Amphetamine; Greater CNS effects Dual Acting Ephedrine - direct and indirect acting sympathomimetic and Promotion of the release of Norepinephrine in excess oral: t1/2 = 3-6 hrs α and β andrenergic agonist; promotes release of NE from nearve terminal; KINETIC: majority excreted in urine unchanged; DYNAMICS: potent CNS stimulant; cardiac stimulan, tachy, inc CO, inc pheripheral resistance, hypertension; dec renal flow; bronchodilation; TOXIC: hypertension and arrhythmias, insomnia, tachyphylaxis(a decreased response to a medicine given over a period of time so that larger doses are required to produce the same response) Methoxamine [Vasoxyl] - α1 adernergic agonist (less specific @ high conc.) Direct acting vasoconstrictiors; lacks CNS stim; dose related inc in peripheral vascular resistance; hypertension, reflex brady USE: hypotensive states, paroxysmal atrial tachy Pseudoephedrine [Sudafed] Phenylpropanolamine Phenylephrine [NeoSynephrine] - a1 adernergic agonist ÌV Direct acting vasoconstrictors; ~ Methoxamine, marked atrial vasoconstriction during ÌV infusion USE: nasal decongestant, mydrriatic agent Anti-Asthmatic ψ ADVERSE EFFECTS OF β 2-Selective Agonist: Skeletal muscle tremor(most common); Feeling restless, apprehhension, and axiety; tachy; arrhythmias or MI - inc risk in pts w/ heart disease, talking MAO inhibitors, tricyclic antidepressents. ψ METABOLIC EFFECTS for β 2- selective agaonist: elevation in glucose, lactate, and free fatty acids Notes by T. Chengot - 2002 Page 9 Terbutaline [Brethine]ψ - β2 adernergic agonist oral, subQ, inhalation not metabolized by COMT USE: obstructive airway diseases, acute bronchospasm (COPD and asthma) Metaproterenol [Alupent, Metaprel]ψ - β2 adernergic agonist oral, inhalation less β2 selective then albuterol or terbutaline; resistanct to methylation by COMT USE: obstructive airway diseases, acute bronchospasm (COPD and asthma) Uterine Release Ritodrine [Yutopar]ψ - β2 adernergic agonist oral and ÌV renal elimination; 2 fold administrations: 1st IV, 2nd oral dose USE: tocolytic - inhibits uterine contractions Inhibitor of Norepineprine release from the nerve terminal Bretylium [Bretylol] prevent release of NE from nerve terminal (Cardiac Drug) Interference with Norepineprine release from the nearve terminal Metyrosine [Demser] Ìnhibition of tyrosine hydroxylase (inhibits tyrosine change to Dopa) Methyldopa [Aldomet] inhibition of L-amino acid Decarboxylase (False Dopa competes w/ Dopamine) Depletion of Vesicle Stores Reserpine block the vesicular uptake of amines, promoting depletion of the transmitter from the vesicles and consequently the metabolism by MOA USE: old time hypertensive Interference with the reuptake of the transmitter Cocaine Ìmipramine [Tofranil] Interference with the Destruction of the transmitter Tolcapone [Tasmar] inhibitors of COMT Selegiline [Carbex] inhibitiors of MOA Tricyclic Antidepressensts Notes by T. Chengot - 2002 Page 10 α - Adernergic Receptor Antagonist DYNAMICS: CV - pheripheral vascular resistance and blood pressure; tachy (β agonist - inc HR b/c α inhibited); compensatory inc in blood volume (chronic use); postural hypotension (dizy when standing) and reflex tachy; Ocular - miosis (dilation is antagonized) ; resp - nasal stuffiness; block constriction of trigone and sphincter muscle in the base of the urinary bladder in the prostrate (promotes urinations - promotes voiding) Name ADMIN INFO Indication/CONTA Nonspecific reversible Phentolamine [Regitine] - α antagonist parentral only MECH: An imiadazoline derivative that is a competative inhibitor of α1 and α2 receptors; produces an antagonist response @ muscurinic and H1 and H2 receptors. KINETICS: unknown, but is extensively metabolized. TOXIC: CV - hypotension, severe tachy(stim of unapposed β), arrhythmias, and MÌ (exacerbation by inc CO, O2 demand - exacerbation); GÌ diarrhea and inc gastric acid production USE: Pheochromocytoma (catacholamine secreting tumor - causes hypotension), erectile dysfunction - non FDA approved method; used to prevent locally to prevent dermal necrosis after the inadvertent extravastation of an a adrenergic agonist(needle stick w/ epi). CONTRA: Acute MÌ, Angina, CAD, Peptic ulcer disease, Pregnancy (efficacy not established) Ergotoamine [ergomar] - α antagonist MECH: α adrenergic antagonist/partial agonist, seratonin receptor antagonist/partial agonist and dopamine receptor antagonist/partial agonist. DYNAMIC: potent peripheral and coranary vasoconstriction, brady, inc smooth muscle activity (uterus). TOXIC: Nausea and vomiting (essentially mediated side effects), vascular insufficiency and myocardial inschemia and gangrene (extreme vasoconstriction) USE: post partum stim of uterus (dec bleeding); migraines - seratonin effect on vasculature CONTRA: pregnancy Tolazoline [Priscoline] - dicontinued Nonspecific Irreversible new receptors MUST be created Phenoxybenzamine [Dibenzyline] - α antagonist - only one MECH: covalent bond to a receptors (duration 14-48 hrs), some selective for α1(but less so then Prazosin); inhibits reuptake of released norE by presympathetic; blocks H1, Ach, and seratonin receptors. KINETICS: absorption well after oral administration, low bioavailability, renal elimination. DYNAMICS: hypotension, Tachy, arrhythmia and MÌ, diarrhea, inc gastric secretion. TOXIC: CNS - fatigue, sedation, and nausea; Carcinogenic in animals USE: Pheochromocytoma - catacholamine secreting tumor α 1 receptor specific antagonist also inclused in this group is Labethalol [Normodyne, Transdate] Prazosin [minipress] oral: t1/2 - 2-3 hr MECH: highly potent and selective α1 receptor antagonist (low affinity for α2). KINETIC: 50-70% bioavailability on first pass (loose 50-30%); renal excretion; 95% protien bound. DYNAMICS: less tachy then Phentolamine; dec peripheral vacsular resistance. USE: hypotension; [congestive heart failure = off lable use - build up tolerance] Terazosin [hythrin] - struc analog of prazosin t1/2 - 9-12 hrs less potent then prazosin; highly specific for α1 receptors KINETICS: 90% bioavailability, duration 18 hrs, renal excretion, 10% unchanged USE: hypertenssion, benign prostatic hypertrophy Doxazosin [cardura] - struc analog of prazosin; once a day dosing t1/2 - 10-22 hr duration of action 36 hr. TOXIC: first dose phenomenon ( marked postural hypotension and syncope; headache, dizziness, drowsiness, and nausea. O.D.: effects indistinguishable from α - non selective (@high enough conc.) USE: hypertenssion, benign prostatic hypertrophy CONTRA: Acute MÌ, angine, CAD, peptic ulcer,pregnancy Notes by T. Chengot - 2002 Page 11 β - Adernergic Receptor Antagonist DYNAMICS: CV - inc peripheral resistance (acutely); lowers peripheral resistance in pts with hypertension (chronically - reflex becomes desensatived and resistance lowers back to normal); negative chronotrope (dec HR) and inotrope (dec CO) - lowers blood preassure; PULMONARY - Bronchospams (β1 specifics not completely void of β2); METABOLÌC effect - inhibition of lipolysis and glycogenolysis; OCULAR - reduced intraocular preassure - dec aqueous humor production. TOXIC: CV - congestive heart failure (in pts w/ CAD; Bradycardia (only ones with ÌSA); cold extremities intermittent claudication (a effect is vascoconstriction); rebound effect - exacerbation of angina, sudden death; Bronchospasm; CNS - fatigue, sleep distrubances( insomnia, nightmares in elderly); metabolic - delayed recovery from insulin induced hypoglycemia, inc triglycerides; other - constipation, diarrhea, or indigestion, sexual dysfunct. O.D.: hypotension, brady, prolonged AV cunduction times, widened QRS; siezures; hypoglycemia; bronchospasm. DRUG interactions: aluminum salts (kelation); Cholestyramine (clolesterol reducing agent - binds reducing absorption); phenytion, rifampin, phenobarbital ( induces P450); Cimetidine (H2 blocker, inhibit P450); CA++ channel blockers (orthostatic hypotension). CONTRA: abrupt discontinuation, asthma, bradycardia, AV block Name ADMIN INFO USE/CONTRA/RXN's Nonselective ISA - Intrinsic Sympathomimetic activity; MSA - Membrane-stabalizing activity propanolol [inderal]* - prototype oral Large Vd (4 l/kg); high lipid soluability - X's BBB; MetaboIism - extensive first pass (25% bioavailability); renal elimination USE: Hypertension (dec CO); Angina (b/c dec O2 demand); arrhythmias; CV effects seen with hyperthyroidism (seem to have inc # of receptors); migraine (prophalaxic, effect on cerebral vasculature); dec lipolysis - dec prostoglandins timolol [ timoptic] oral and topical: t1/2 = 4 hrs Non selective agent with no ÌSA and no MSA. KINETICS: hepatic metabolism and renal elimination, 50% bioavailability USE: glaucoma (dec intraocular preassure), migraine nadolol [corgard] t1/2 - 20 hr Non selective agent with no ÌSA and no MSA. Longer duration, less lipid solubility then Propanolol (less penetration into CNS); limited metabolism - majority excrete unchanged in urine (Accumulatetion in pts w. renal failure) USE: Glaucoma pindolol [ visken] oral: t1/2 - 4 hr Non selective agent with no ÌSA and low MSA. Mepatic metabolism; 20% metablized, 80% bioavailable; renal elimination of metabolites and unchanged drug USE: Hypertention Labetolol [Normodyne, Trandate]§ - specific a1 and nonspecific b adrenergic antagonis oral, ÌV: t1/2 - 8 hr competative antagonist at α1 and adrenergic receptors; affinity for α1 less then phentolamine; b inhibition is less then propranolol; less tachy (reflex) then Propanolol. KINETICS: extensive first pass metabolism with 20-40% bioavailability, renal elimination of metabolites USE: hypertension β 1 selective Metoprolol [lopressor] oral: t1/2 - 3- 4 hr no ÌSA, weak MSA; extensive metabolism with bioavailability of 40%; renal elimination of metabolite and unchanged (10%) USE: Hypertension and phrophylaxtic for migraine Atenolol [terormin] oral: t1/2 - 5-8 hr no ÌSA; highly hydrophillic (limited BBB Xing). KINETICS: excreted unchanged in the urine USE: Hypertension Esmolol [brevibloc] ÌV: t1/2 = 8 mins minimal ÌSA; causes hypotension in norm individuals. KINETICS: metabolism w/ plasma esterases; excreted in urine USE: critically ill pts in which there is a need for rapid withdrawal of drug = hypertensive crisis Acebutolol [sectral] - NOT a prodrug - Diacetolol - active metabolite oral: t1/2 = 3 hr (A), 8- 12 hr (D) has ÌSA and MSA. KÌNETÌCS: metabolized in liver to active metabolite Diacetolol; excreted in urine USE: hypotension, ventricular arrhythmias Notes by T. Chengot - 2002 Page 12 Chemotherapy of Parasitic Infections Name ADMIN INFO USE/CONTRA/RXN's Anti-Malarials *sporazoites invade liver cells; Schizonts mature in liver; Merozoites from liver invade RBC Choroquine [Aralen]ℵ - used also and an antiaemebial - not a prodrug, but has active metabolite oral, parentral: t1/2 1-2 mo MECH: uncertain concentrates in food vaculoes of parasite; Ìntercalation with parasite DNA/RNA; Resistance - alteration of drug transport resulting in a dec ability to concentrate in cells; KINETICS: Vd - 100-1000 l/kg; plasma protien binging - 50%; accumulates w/in erythrocytes. METABOLISM: active metaolite is monodesethylchloroquine - contributes to avtivity. TOXIC: GÌ - Nausea, vomiting, adbominal pain; CNS - headache, anorexia, malaise; Ocular and dermal - blurring of visions, opacification, retinopathy, urticaria, puritics; discoloration in nail beds and mucous membrane (conc in melanin tissue); prolonged neuropathy(prolonged tx); Hypotension, vasodilation, EKG changes - eventual cardiac arrest; Hemolysis and blood dyscrasias (G-6-P difficiency) USE: non-falciform, and sensative falciform; blood schizonticide; gametocytes of vivax, ovale, malariae; effective only in erythrocytic phase; Hepatic Amebiasis CONTA: pregnancy, ocular disease, hepatic disease or severe GÌ, Neurological, blood disorders, G-6-P Dehydrogenase dificiency; Porphyria psoriasis Hydrocychloroquine [Plaquenii] Mefloquine [ lariam] oral only; t1/2 - 20 days MECH: theory ~ choroquine; moy form toxic complexes with free heme; resistance (like chloroquine). KINETIC: absorption enhanced by food; high distribution, 98% protien bound; excretion mainly feces, min urine, 5% excreted unchanged. TOXIC: GÌ (most common) - Nausea, vomiting, abdominal pain, diarrhea, dysphoria, diziness; ataxia, headache, EKG changes(AV block), severe neuropsychiatric rxn - disorientation, siezures, encepholopathy, psycosis USE: prevention and tx of chloroquine resistance and multidrug resistant faliciform; effective blood schizonticide, mature trozoite and schizont forms; no activity against hepatic stages and mature gametocytes of faliciform or latent vivax. CONTRA: pregnancy; epilepsy, psychiatric disorders (perputuate siezures); cardiac disease (EKG changes); Quinine or quinidine hypersensativity (X allergic rxn) Primaquine oral only: t1/2 - 6 hr Drug of choice for the eradication of dormant liver forms of vivax and ovalis; MECH: uknown - theory - generates reactive O2 (superoxides) species or interfereing with electron transport in parasite KINETICS: three active metabolites (1 antimalarial, 2 toxic - hemolytic anemia); renal elimination. TOXIC: Nausea, epigastric pain, abdominal cramps, headache, cardiac arrhythmias; hematological effects - mild anemia, methemoglobinemia, leucopeonia, agranulocytosis and leukocytosis (rare) USE: terminal prophylaxis and radical cure of vivax and ovale (relapsing); ineffective agains falciform erythrocytic stage; marked gametocideal effect against all 4 species (esp falciform). CONTRA: Pregnancy, history of Granulocytopenia or methemoglobinemia, parenteral administration (b/c of severe inc K); G-6-P dehydrogenace difficiency; iodine hypersensativity (X react w/ drugs); SLE and Rheumatoid Arthritis Notes by T. Chengot - 2002 Page 13 Quinine [ QM-260; Quinamm] oral or parentral (ÌV no longer used): t1/2 - 11 hr from bark of cinchona tree. MECH: Unknown ~ chloroquine (inhibit heme polymerase. KINETIC: distribution - 1.5 l/kg; metabolism in liver, excretion is renal. TOXIC: VERY TOXÌC! Cinchoism - tinnitis, headache, nausea, and blurred vision(take pts off @ this pt.); vomiting, diarrhea, abdominal pain, dec auditory acuity, photophobia, night blindness; hypersensativity RXN - blackwater fever (hemolysis, hemoglobinemia, and hemogloinuria); Hematologic Effects - hemolysis, leucopenia, agranulocytosis, and thrombocytopenia; severe hypoglycemia(b/c quinine inc. insulin release); stim of uterine contractions (esp 3rd trimester); severe hypotension DRUG INTERACTIONS: Digoxin (dec clearance - toxicity); oral anticoagulants - warfarin - inc toxicity b/c Quinine inhibits clottong factors formed by K); NM blocking Drugs - potentiates NM blocking effects; Cimetidine - inhibits metabolism - increases Quinine toxicity USE: Falciform malaria; blood schizonticide, little effect on sporozoite or preerythocytic forms; gametocidal for vivax, malariae, but not falciparum. CONTRA: pregnancy(congenital malformation - catagory X), previous hypersensativity; hypoglycemia(b/c of insulin stim effect); tinnitus, optic neuritis; cardiac arrhythmias, G-6P dehydrogenase) Pyrimethamine [Daraprim] oral: t1/2 - 3.5 days MECH: inhibits plasmodial dihydrofolate reductase (needed to form folic acid); resistance (mutation of enzymes). KINETICS: 87% protien bound, widely distributed with conc occuring in the liver and kidney; extensive metabolism. TOXIC: GÌ effects, skin rash, pruritis; thrombocytopenia USE: erythrocytic forms of all species, not effective agains gametocytes, not effective agains liver stage of vivax and ovale CONTRA: anemia, pregnancy (crosses placenta), Breat feeding (folate dif in infants); ÌM injections - localized side effects Pyrimethamine + Sulfadoxine [Fansidar] Oral: t1/2 - 170 hr (sulf) & 90 hrs (Pyr) MECH: inhibits plasmodial dihydrofolate reductase (needed to form folic acid- Pyrimethamine) and onhibit dihydropteroate synthetase (Sulfadoxine) KINETICS: Renal elimination. TOXIC: GÌ effects; skin rash, pruritis; dermal rxn - erythema multiforme (steven johnson syndrome - is a severe and sometimes fatal form of erythema multiforme, a disorder that occurs in response to medications, illness, or infections) and toxic epidermal necrolysis USE: Suppression of malaria esp chloroquinine strains of faciparum; toxoplasmosis; pneumocytosis. CONTRA: Infants <2mo; hepatic/renal diseae; hypersensativity; megoblastic anemia; *pregancy*; breast feeding; ÌM; andy pre-existing anemia - folic acid dif. Atovaquine; Proguanil [Malarone] oral: t1/2 (A)=2 3 days and (P)=12-21 hrs combo to treat falcipram. MECH: Atovaquone - interferes w/ mitochondrial electron transport and ATP and pyrimidine buisynthesis; Proguanil (prodrug) - active metabilte Cycloguanil inhibits dihydrofolate reductase-thymidylate synthase of plasmodia (inhib DNA synthesis and folate depletion); KINETICS: A - 23% bioavailabilty; 99% protien binding; hepatic met; 94% excreted unchanged in feces; P - 75% protien bound; hep met by P4502c19 to cytoguanil; elimination mainly feces, minor urine Tetracycline - tertracycline antibiotic therapy No effect in liver stage adjunct to therepy in treament of erythrocytic Doxycycline - tetracycline antibiotic therapy schozonts of all malarial parasites Anti-Protozoals Chloroquine [Aralen]ℵ - see malaria Notes by T. Chengot - 2002 Page 14 Metronidazole [Flagyl] - antiamebial oral: t1/2 = 7.5 hrs MECH: unkown - toxic rxn from formation of reative products and interactions of these products with the DNA in anearobic bact and protozoans (nitro grp. Reduced inside bact or prot - forms reactive product; Resistance - dec reduction in these cells. KINETICS: well distributed (CNS and breast milk - high lipophilicity); metabolism - liver; excretion - renal. TOXIC: CNS -nausea, headache, dry mouth or metallic taste; vomitting, diarrhea, insomnia, weakness, dizziness, thrush, rash, dysuria, dark urine, vertigom paresthesia, neutropenia; pancreatitis, siezures, encephalopathy USE: Amebiasis; Giardiasis; Trichomoniasis DRUG INTERACTIONS: alcohol (disulfram like rxn) - build up of acetoaldehyde - adverse rxn; Warfarin - inhibit metabolism of warfarin (toxicity); phenytoin, phenobarbital - inc hepatic enzymes - dec effect of drug; Cimetitidine - enzyme inhibitor - drug toxicity Ìodoquinol [Yodoxin] - antiamebial oral: t1/2 - 11- 14 hr MECH: unknown - activity is limited to GÌ. KINETICS: 10/% absorbption, excreted renally as glucuronide conjugate. TOXIC: diarrhea, anorexia, nausea, vomitting, abdominal pain; headache, rash, pruritus; neurotoxicity - optic neuritis, optic atrophy, and peripheral neuropathy; alopecia and agranulocytosis (rare) USE: Amebiasis CONTRA: do not use in the renally and hepatically impaired; pre- existing optic neuropathy, or iodine; Hypersensativity or hypersenstivity to 8- hydroxyquinolines Pentamidine [Pentacarinat] - anti amebial; damdage to pancrease with prolonged use - as pancreas get more destoryed there is inc in secretions resulting in hyperglycemis) Parenteral or inhalation: t1/2 2-4 wk MECH: Unknown - theories: interfearance w/ incorpotation of nucloetides into DNA/RNA; inhibition of oxidative phophorylation and biosynthesis of DNA, RNA, protiens, and phospholipids; inhibition w/ formation of folates. KINETICS: widely distributed; 69% protien bound; metabolism unknown; renal emilination. TOXIC: (VERY) nephrotoxicity (25%) - renal tubule necrosis; hyperkalemia (prevents reabsorption of NA in distal tubule, K can't exchange with NA); hypotension, tachy; pancreatitis; hypoglycemia/hyperglycemia (reflection of pancreatic damage: hypogly→pancrease becomes more destroyed→inc secretions (hyperglycemia); dizziness , dyspnea; rash, metallic taste, fever, nausea, vomiting USE: Pnuemosytosis, African Trypanosomiasis; leishmaniasis CONTRA: ÌM (bruising/bleeding) Diloxanide Furoate [Furamide] - CDC only oral MECH: Unknown. KINETIC: hydrolyzed to diloxanide and furoic acid in the intestinal lumen or mucosa; unabsorbed diloxanide has local amebicide activity; excreted renally as conjugate metabolite. TOXIC: Flatulance, vomiting, pruritis, and urticaria USE: asympotomatic Luminal Amebiasis. CONTRA: pregnancy; children <2 years of age Dehydroemetine [Mebadin] - CDC only Synthetic derived from Emetine (not available in US), natural product derived from Ìpecac w/ amebicidal prps. MECH: unknown - theory - inhibits protien synthesis TOXIC: diarrhea, nausea, vomiting; polyneuritis, CV effects, Hypotension, congestive heart failure, arrhythmias USE: extraintestinal amebiasis in pts who do not respond to or cannot receive metronidazole Paromomycin [Humatin] - antibiotic for treatment oral MECH: direct amebicidal activity and indirect activity by interfering with the enteric flora essential for pathogen prolif. KINETICS: 100% fecal elimination; no extraintestinal efficacy. TOXIC: abdominal cramping and diarrhea USE: Luminal Amebiasis. CONTRA: Aminoglycoside hypersensativity; GÌ obstruction and Ìleus (retain in GÌ tract longer - liklihood of absorption) Erythromycin - antibiotic for treatment MECH: interferes w/ enteric flora essential for pathologic prolif of amoebae Notes by T. Chengot - 2002 Page 15 Anthelmintics local or systemic - eradicate adult helminthes or developmental forms that have invadedd organs and tissues GENERAL CONTRAINDICATIONS: pregnancy; ulcers of GÌ tract (b/c inc systemic absorption) •BENZÌMÌDAZOLE oral t/1/2: - A (8-12hr), M (2- 6hr), T (1.2 hr) MECH: impairment of glocose uptake by nematodes (inhibit microtubule synthesis) KINETICS: limited absorption; inc w/ fatty meal; Metabolized in liver; elimination fecal (A & M) and urine (M & T) Albendazole [Albenza] - compassionate use; Benzimidazole • ELIMINATION FECAL. TOXIC: abdominal pain, diarrhea, nausea, dizziness, and headache; ÌNC in serum aminotransferase activity, jaundice or chemical cholestasis (rare) USE: Ascariasis, Trichuriasis, hookworm, pinworm, Stronglyoidiasis, Hydatid disease, Neurocysticercosis Mebendazole [Vermox] - Benzimidazole• dec t1/2 inc toxicity ELIMINATION FECAL AND URINE. TOXIC: abdominal pain, diarrhea; Allergic rxn, alopecia, reversile neutropenia, agranulocytosis (rare) USE: Ascariasis, Trichuriasis, hookworm, pinworm, cutaneous larvea migrans Thiabendazole [Mintezol] - Benzimidazole• ELIMINATION URINE. TOXIC: Dizziness, anorexia, nausea, votiming; epigastic pain, abdominalcramps, diarrhea, pruritus, headache, and giddiness; fever, rashes, erythema multiforme, hallucinations, sensory disturbances, steven johnson syndrome; angioneurotic edema, shock, tinntus, convulsions and intrahepatic cholestasis (rare) USE: Stongliodiasis Ìvermectin [Mectizam] - retained in GÌ tract oral: t1/2 16-27 hr MECH: unknown, may paralyze the nematode and arthropods by intensifying GABA mediated trasmission signals in peripheral nerves (paralyzes N and A). KINETICS: excretion in feces unchanged (retained in GÌ tract). TOXIC: fatigue, nausea, vomiting, abdominal pain and rashes; Mazzotti-like RXN - due to dying microfilariae (milding itching and swollen, tender lymph nodes) USE: Onchocerciasis; Stronglyoidiasis. CONTRA: pregnancy and breat feeding. DRUG INTERACTIONS: barbituated and benzodiazephines Niclosamide [Niclocide] oral MECH: inhibits mitocondrial oxidative phosphorylation and inhibits glucose uptake in parasites. KINETICS: minimal absorption; fecal elimination (kept in GÌ for max effect). TOXIC: Nausea, vomiting, diarrhea, abdominal discomfort, anorexia, drowsiness, headache, rash USE: Taeniasis (tape worm); Hymenoleps anan (dwarf tapeworm); intestinal flike infections Piperazine oral MECH: Non depolarizing drugs that blocks Ach at the myoneural junction of the Ascaris Muscle, causing flacid paralysis of the worm. KINETICS: excreted renally unchanged. TOXIC: Nausea/vomiting, abdominal cramps; headache, dizziness, vertigo, ataxia, seizures (CNS); cateracts, blurred vision USE: Ascariasis. CONTRA: epilepsy, renal dysfunction Prazquantel [Bitricide] - mild CNS effect oral: t1/2 - 2 hr MECH: Ìnc. cell membrane permeability to calcium - inc muscular activity followed by contraction and spastic paralysis, dislodgement and death. KINETIC: Extensive first pass; 80% plasma protien bound; elimination feces and urine; TOXIC: malaise, abdominal discomfort, anorexia, dizziness, headache, fever; transient elevation of liver enzymes USE: Schistosomiasis; Clonorchiasis; Opisthorchiasis. DRUG INTERACTION: carbamazepine; phenobarbital Pyrantel Pamoate [Antiminth] oral MECH: depolarizing nm bloackade of the nicotinic receptors - stimulates Ach release and inhibits ACHesterase @ ganglionic neurons w/Ì worm - resulting in paralysis, detachment, and expulsion. KINETICS poor absorption, activity limited to luminal oranism; fecal and renal elimination of unchanged drug and metabolites. TOXIC: GÌ - Nausea, vomiting, diarrhea, abdominal cramps; CNS -dizziness, drowsiness, headache, insomnia; rash, fever, weakness USE: enterobius vermicuraris; Ascariasis; Hookworm and Trichostrongylus orientalis; trichinosis Notes by T. Chengot - 2002 Page 16 SuIfonimindes derivatives of sulfonilamide; MECH: sulfonamides are struc analogs and competative antagonist of PABA, inhibiting dihydropteroate synthare and dec synthesis of folic acid; effective against bact that produce own folic acid; bact that use preformed folic acid not affected; inc levels of PABA counteract bacteriostasis produced by sulfonimides (competative); Must get into cell to take effect ANTI-BACTERIAL SPECTRUM: (broad spectrum) E-coli, Strep. Pneumoniae, Strep Pyogenes, H. Ìnfluenza, H. Ducreyi, Yersinia pestis, Chlamydia trachomatis, Actinomyces, Nocardia asteroids, Plasmodium Falciparum, Toxoplasmosis Gondii. TOXIC: Renal - crystaluria (greater incidence w/ older compounds); Hemetological effects - G-6-P - acute hemolytic anemia, Agranulocytosis, aplstic anemia; Hypersensativity rxn - rashes (urticarial, erythema nodosum, erythema multiforme(steven-johnson type), exfoliative dermatitis, and photosensativity; Fever malaise, and pruritis; Hepatic - necrosis, hepatomegaly, jaundice, kernicterus (displaces bilirubin in newborns - causes encephalopathy); GI - anorexia, nausea, and vomiting Resistance: alteraction in dihydropteroate synthase; inc metabolism of the sulfonamide by the bact, alternative mech for folate production; inc production of PABA (competative antagonism) USE: UTÌ (Primary Tx), Trachoma and inclusion conjunctivitis DRUG INTERACTIONS: Oral anticoagulant; oral sulfonyurea (hypoglycemic); Phynytoin; Tetracycline (side effect is photosensativity - potentiated); Local anesthetics - metabolite PABA formed, dec effectiveness; Acsorbic Acid - acidifies urine - inc crystalluria; Salicylates, NSAÌDs - displaces sulfonimides; Methotrexate in some populations causes bone marrow suppression Name ADMIN INFO USE/CONTRA/RXN's rapidly absorbed/eliminated Sulfisoxazole [Gantrisin] - rapidly absorbed and eliminated; oral - 70-100% absorption; topical - limited absoption oral and topical: t1/2 - 5 8 hrs Highly protien bound, inc likelihood of displacement rxn (wide distribution); readily X's BBB; metabolized in liver - Acetylation (non-microbial metabolite) and Glucuronidation (anti-microbial metabolite); renal elimination - 70% unchanged; high soluability of drug decreases liklihood of crystalluria (insoluable in urine - obstruction) USE: Trachoma and Ìnclusion Conjunctivitis; Nocardiosis Sulfamethoxazole [Gantanol] - rapidly absorbed and eliminated oral metabolism - hepatic with a higher formation of crystalluria USE: UTÌ; Trachoma and Ìnclusion Conjunctivitis; Nocardiosis Sulfadiazine - rapidly absorbed and eliminated oral hepatic metabolism w/ renal elimination of uncharged drug and aceylated form (15-45%); Poor soluability of drug increases likelihood of crystalluria(slower rxn); less water soluable, higher precipatiation USE: Combination w/ Pyrimethamine for treatment of toxoplasmosis and Malaria; Nocardiosis Poorly abosrbed Sulfasalazine [Azaline, Azulfidine]* - poorly absorded; PRODRUG Metabolized by intestinal bact to Sulfapyridine (responcible for toxic effect) and 5-aminosalicylic acid (mesalamine - anti-inflamitory; responsible for drug efficacy); Phase 2 drug. TOXIC: Nausea, fever, arthragias (severe joint pains), and rashes; Heinz-body anemia, acute hemolysis in pts w/ G-6-P dehydrogenase deficiency, and agranulocytosis USE: ulcerative colitis and rheumatoid arthritis (disease modifying agent - NOT 1st choice). CONTRA: same as sulfonimide plus salicylate hypersensativity (asprin/NSAÌD allergy) Sulfonimides for topical use only Sulfacetamide [isoptocetamide, Sulamyd sodium] - for topical use opthalmic or topical derivative of Sulfanilamide; inc soluability and stabe in solution at pH 7.4 allows it to be used in an opthalmic formulation (most sulfonimides have alkaline pH); 90% more soluable USE: opthalmic infections CONTRA: persons w/ sulfer allergy Silver Sulfadiazine [Silvadene] - for topical use topical MECH: does NOT inhibit folic acid synthesis; toxicity is due to disruption of the bacterial cell membrane and cell wall (silver contributes to toxicity) TOXIC: sulfonamide profile + buring, rash, itching, and skin discoloration USE: Prophylaxically; topically to reduce microbial colonization and the incidence of infection of wounds from burns. SENSATIVITY: bacteria and yeast Notes by T. Chengot - 2002 Page 17 Mafenide [Sulfamylon] - for topical use topical has Gram -/+ coverage. TOXIC: intense pains @ site of application, allergic rxn, and loss of fluids by evaporization from burn surface (not used w/ dressing - open site); Metabolic acidosis, hyperventilation (ability to inhibit carbonic anhydrase - elimination of bicarbonate) USE: adjunctive therepy to second and third degree burns Long Acting Sulfadoxine - long acting TOXIC: severe, fatal, steven-johnson syndrome USE: Ìn combination with pyrimethamine for malaria and toxoplasmosis SYNERGIST Trimethoprim- Sulamethoxazole [bactrim] - synergist - SuIfonimide: competative antagonist of PABA inhibiting dihydropteroate synthase and dec synthesis of folic acid; Trimethoprim: competative inhibitor of microbial dihydrofolate reductase, dec metabolism of folic acid to terahydrofolate and the precurso required in purine synthesis (does not form DNA). Oral: t1/2 (T)=11 hr and S=10 hr KINETICS: (T) is rapidly distributed on concentrated in tissue, @ 40% is bound to plasma protien in presence of (S); Vd of (T) is 9x (S); 65% of (S) is bound to plasma protien; renal elimination of uncharged drug and metabolite TOXIC: Megoloblastosis, leukopenia, or thrombocytopenia in pts w/ dec folic acid levels; derm - exfoliative dermatitis, steven johnsome syndrome and toxic epidermal necrolysis; GI nausea, vomiting, glossitis. and stomatitis (common*); mild and transient Jaundice (allergic cholestatic hepatitis); *hematological* - enemia (aplastic, hemolytic, macrolytic), coagulation disorders, granulocytopenia, agranulocytosis, purpura, Henoch-Schonlein purpura, sulfhemoglobinemia(dec in folic acid synthesis); nephrotoxicity ANTI - Bact sprectrum - most gram -/+; E. colr, Proteus mirabilis, P. Morganii, P. rettgeri, Salmonella, Shigella, Brucella abortus, Chlamydia diphtheriae, N. Meningitis, N. Asteroides, Serratia, Klebsiella, Enterobacter species. USE: UTÌ, Resp. tract infect., GÌ infect, Ìnfec by P. Carinii, Propylaxis in Neutropenic pts., nocardia infect., brucellosis. RESISTANCE: mutation of the enzymes. QuinoIones oral: t1/2 = 3- 10 hr MECH: inhibition of toperisomerase ÌÌ (DNA gyrase) and Toperisomerase ÌV - dec bacterial synthesis; resistance - alterination in enzyme binding sites, or dec premeability of drug into bact. KINETICS: bioavailability 80-90%, wide distribution, renal elimination (some feces) DRUG RXN: Antacids - dec absorption (Al, Ca, Mg - kelate); Caffine and Theophylline - dec met - caffine/theopylline toxicity; Warfarin - drug displacement rxn - inc bleeding time; drugs which inc QT - interval (some Gen ÌÌÌ); Digoxin - inc drug levels; retinoids - photosensativity potentiation; probenecids - sequesters release of quinolones - stops renal tublue secretion of quinolones Name ADMIN INFO USE/CONTRA/RXN's First Generation: Quinolones Cinoxacine [Cinobac] - 1st oral antibiotic Oral suseptable org. : enterobacter sp., E. coli, Klebsiella sp., Proteus Miribilis, Proteus vulgaris USE: UTÌ caused by gram - org. Nalidixic acid [NegGram] oral suseptable org. : enterobacter sp., E. coli, Klebsiella sp., Morganella morganii, Proteus Miribilis, Proteus vulgaris, providencia rettgeri, Salmonella sp., Shigella sp. USE: UTÌ caused by gram - org. Notes by T. Chengot - 2002 Page 18 Second Generation: Fluoroquinolones Norfloxacin [Noroxin] - mainly gram - , some gram + oral and opthalmic SHORTEST t1/2 Poor oral bioavailability SUSEPTÌBLE ORGS.: Citrobacter freundii; Enterobacter cloacae; E. coli; Klebsiella pneumoniae; Proteus mirabilis; Pseudomonas aeruginosa; Staph. Epidermidis; Ìndole + Proteus species (grm +) Groupd D strep; Neisseria gonorrhoeae. **TOXIC:** GI - nausea, vomiting, abdominal pain, anorexia; Hepatic - elevated hepatic enzymes, hepatitis, jaundice, pancreatitis (rare); CNS - headaches (2.9%), Dizziness (1.8), and Fatigue; Maculopapular rash; hypersensativity rxn - erythema multiforme and stenvens johnsons syndrome, and exfoliative dermatitis; Hematologic - hemolytic anemia, leukopenia, neutropenia, thrombocytopenia; Crstalluria (dose dependent) USE: UTÌ; uncomplicated ponorrhea; bacterial conjunctivitis CONTRA: Previous hypersenstivity to any quinolone; pregnancy, breast feeing children <18 (tranfer to fetus - chill - joint pain/changes); tendon pain (rupture)* Ciprofloxacin [Cipro] Oral, parentral and topical BEST COVERAGE FOR PSUDOMONAS. Suseptable org.: broad spectrum antiinfective agent, not effective against anaerobic organisms; most potent floroquinolone agains P. aeruginosa (no effect on anaerobic organisms). TOXIC: GI - nausea, vomiting, abdominal pain, anorexia; Hepatotoxicity; CNS - headaches (2.9%), dizziness (1.8%), and fatigue; RARE - siezures, inc intracranial pressure and toxic psychosis (has occured @ high concentrations); Hypersensativity rxn - maculopapular rash, fever, eosinophillia; can lead to jaundice, hepatic necrosis - DÌNCONTÌNUE USE USE: bact. Conjucntivitis, bone and joint infection, lower respiratory infection, gonorrhea, typoid fever, sexually transmitted diseases, skin and soft tisse infections, UTÌ. CONTRA: ~Norafloxacin Ofloxacin [Floxin] oral, ÌV, opthalmic, otic most complete oral bioavailability in class, highest renal elimination of uncharged drug; less potent then Ciprofloxacin; Most effective against gram - org.; TOXIC: GI - nausea, vomiting, diarrhea; CNS: Ìnsomnia, headache, dizziness, drowsiness and dysarthria (slurred speech) USE: UTÌ, Prostatitis; lower resp tract infect.; skin infect.; uncomplicated gonorrhea; non-gonococcal urethritis and cervicitis; otis media CONTRA: Same as Norfloxacin Enoxacin [ Penetrex] more limited bacterial spectrum then other floroquinolones. Greatest effect on hepatic metabolism of other drugs; TOXIC: Same as Norfloxacin USE: UTÌ; gonorrhea. CONTRA: Same as Norfloxacin Lomefloxacin [Maxaquin] Oral long t1/2 allows once a day dosing, no food interactions (lesser incidence of interactions). TOXIC: greater incidence of phototoxicity; same as Norfloxacin CONTRA: Same as Norfloxacin Third Generation Moxifloxacin [Avelox] Oral and ÌV long t1/2 allows once a day dosing, no effect on P450. TOXIC: Q-T prolongation; same as Norfloxacin USE: Bronchitis, Sinusitis, Pneumonia CONTRA: pts w/ QT prolongation; same as Norfloxacin Levofloxacin [Levaquin] - levo isomer of Olfoxacin oral, ÌV, opthalmic once a day dosing. TOXIC: Same as Moxifloxzcin *BUT no QT prolongation USE: Bact. Conjucntivitis, Sinusitis, Bronchitis, community-aquired pneumonia, skin and skin structure infect., UTÌ, Pyelonepritis. CONTRA: same as Moxifloxacin Gatifloxacin [Tequin] oral and ÌV an 8-methoxyfluroquinolone; broad spectrum once a day dosing; no interaction w/ hepatic P450 enzymes. TOXIC: Same as Moxifloxzcin USE: UTÌ; Community-acquired pneumonia; acute sinusitis; gonorrhea. CONTRA: same as Moxifloxacin Sparfloxacin [Azgam] Oral TOXIC: same as moxifloxacin and QT prolongation USE: community aqired pneumoia; bronchitis. CONTRA: same as Moxifloxacin Notes by T. Chengot - 2002 Page 19 Fourth Generation Trovafloxacin/Alatrofloxacin [Trovan] - alatrofloxacin is a prodrug oral (T) and iv (A) T is the active form of the drug; A is an iv formulation and a PRODRUG that is metabolized into active trovofloxacin TOXIC: elevation og hepatic enzymes and hepatic failure; same as moxifloxacin USE: life of limb threatening infection for which the need for the antibiotic out weight the potential risk of hepatic disease CONTRA: hepatic disease ; same as moxifloxacin Notes by T. Chengot - 2002 Page 20 Antiseptic and AnaIgesic Against for UTI Name ADMIN INFO USE/CONTRA/RXN's Methenamine [Hiprex] - degraded spontaneouly to formaldehyde oral admin of enteric coated compound MECH: @ low pH, spontaneous decomposistion to active form, formaldehyde, and ammonia occurs (broad spectrum use); renal elimination. TOXIC: GÌ (severe) effects include nausea/vomiting, diarrhea, abdominal cramps, stomatitis, and anorexia; dysuria, albuminuria, hematuria, and rashes; crystalluria(b/c of low water output) USE: UTÌ CONTRA: renal insufficiency and hepatic Ìnsufficiency(b/c inc ammonia accumulation); Dehydration - likelihood of crystaluria Nitrofurantoin [Furadantin] oral: t1/2 = 20- 60 mins MECH: reduction of parent drug to form highly reactive intermidiates that interact with bacterial DNA (alter DNA resulting in cell death). KINETICS: wide distribution, X's placenta, breast milk; Renal elimination, 40% unchanged. TOXIC: Pulmonary - cough, dyspnea, pneumonitis, pulmonary fibrosis , respiratory failure (occus quickly and severely); GÌ - nausea, vomtning, anorexia, diarrhea, and abdominal pain; rashes; CNS (widely distribtuted) - headaches asthenia, dizziness, drowsiness, nystagmus, and vertigo, peripheral neauopathy; hemolytic anemia; urine discoloration - brown; hepatotoxicity (rare) USE: UTÌ. ANTIMICROBIAL ACT.: E. coli, enterococci. CONTRA: (pulmonary probs not a contra); renal impairment, oliguria, anuria (due to 40% unchanged); G-6-P dehydrogenase dificiency - hemolytic anemia; Pregnancy and caution breat feeding (to neonates). Phenazopyrdine [Pyridium] - azo dye provides analgesic action on the urinary tract, NOT a urinary antiseptic; TOXIC: Discoloration of urine - orange or red; Methemoglobinemia and hemolytic anemia; acute renal failure; hepatotoxicity; GÌ - nausea, vomiting USE: decrease symptoms of dysuria, frequency, buring, and urgency. CONTRA: G-6-P dehydrogenase dificiency; hepatic disease; renal disease; uremia Notes by T. Chengot - 2002 Page 21 Beta-Lactam Antibiotics MECH: inhibition of cell wasll synthesis; Peptidoglycans is required comp of cell wall that maintains struct and rigidity of cell wall; cell wall prevents cell lysis; the LAST STEP is synthesis of peptidoglycan is the removal of terminal alanine from D-ala-Dala subtrate w/ transpeptidase; transpeptidase is catalyzed by penacillin;Beta-lactam form covalent bond w/ penicillin binding proteins inhibiting transpeptidase rxn; ONLY WORK WHEN CELL IS GROWING. RESISTANCE: inactivation by beta- lactamases; modification of target protein; impaired penetration of drug to target penicillin binding protien(alter transport mech); presence of efflux pump (drug moved out of cell) Name ADMIN INFO USE/CONTRA/RXN's Penicillins: OraI, parentraI (IM, IV) KINETICS: oral absorption is dependent upon the stability of the compound in acid (most have rapid absorption; food delays reabsorption); Parentral (ÌM or ÌV) - absorption is rapid and complete; Distribution (liver, bile, kidney, semen, joint fluid, lymph, and intestine), w/ min penetration of BBB (except in inflamation, inc permeability) and wide range of plasma protien binding; Renal elimination. TOXIC: hypersensativity rxn - rash, urticaria, stevens-johnson syndrome, anaphylaxis; interstitial nephritis; phlebitis and thrombophlebitis (ÌV - irritating drug); GI - Nausea, vomiting Diarrhea, Psuedomembranous colitus (rare); Neurotoxicity; seizures (high enough dose); Hema - leukopenia, thrombocytopenia; Electrolyte disturbances - hyperkalemia, hypokalemia (pen G) DRUG INTERACTIONS: probenecid - dec renal elimination; aminoglycosides antibiotics - chemical antagonism (kelate); methotrexate - inhibit renal tubule secretion of methotrexate ( toxicity of bone marrow suppression); Potassium sparing diuretics ( Hyperkalemia - PEN G potentiates); Warfarin - dec W effectiveness; oral contraceptives SuceptibIe to: inactivation by beta- lactamases; Susceptible - strep, meningococci, enterococci, pneumococci, Non-beta lactamase producing staphylocooi, tremponema pallidum (syphillis); bacillus anthracis; clostridium species,actinomyces CONTRA: previous penicillin hypersensativity; ÌV administration (Pen G procaine and Bezathine); procaine sensativity - pts w/ cross hypersensativity w/ cephalosporins (10%) Penicillin G ÌM, ÌV TOXÌC: Electrolyte disturbances - hyperkalemia, hypokalemia (pen G) Penicillin G Benzathine [Bicillin] ÌM extended release formulation CONTA: ÌV Penicillin G procaine ÌM extended release formulation Penicillin V oral Antistaphylococcal Penicillins Resistance to stalyococcal peniciilins; Suseptibille Orgs - Beta lactamase producing staph, strep, pneumococci USE: bacteremia, soft tissue injections, bone and joint infections, UTÌ, upper respiratory infections Nafcillin [Unipen] oral and parentral Hepatic Metabolism ~ 60%; excretion - bile and urine(minimal); less risk of nephritis TOXIC: Neutropenia Oxacillin [Bactocil] oral and parentral Better oral absorbtion compared to Nafcillin; less incidence of phlebitis from ÌV compared to Nafcillin; hepatic Metabolism ~50%; elimination by bile and urine; better oral absorption ten dicloxacillin - 2x; TOXIC: hepatotoxicity Cloxaciilin [Cloxapen] oral and parentral Hepatic Metabolism ~ 10%; excretion - bile and urine Dicloxacillin [Dynapen] oral only Hepatic Metabolism ~ 10%; excretion - bile and urine Extended Spectrum Penicillins: - Natural, greater coverage Greater efficacy against gram - org, inactivated by beta lactamases; Susceptible orgs. - strep, menigococci, enterococci, pneumococci, non-beta- lactamase producing staph, treponoma palladium, bacillus anthracis, clostridium species, actinomyces, listeria mocytogenes, salmonella species, gram negative cocci and bacilli USE: otis media, sinusitis, bacterial cystitis, skin infection, upper resp tract infect, UTÌ Notes by T. Chengot - 2002 Page 22 Carbenicillin [Geocillin] Oral 1st extended release penicillin; renal elimination CONTA: coagulopathy - show to inhibit platelet coagulability so don't give to hemophilliacs Amoxicillin [Amoxil] - aminopenicillin Oral Better oral absorbtion compared to Ampicillin; Min hepatic Metabolism; Renal emimination; lesser incidence of GÌ side effect compared to Ampicillin; TOXIC: skin rash - non allergy - direct derm responce Ampicillin [Omnipen] - aminopenicillin oral and parentral renal elimination; TOXIC: skin rash - non allergy - direct derm responce Bacampicillin [Spectrobid] Mezlocillin [Mezlin] parentral only hepatic metabolism; renal and bile elimination;acid labile - destroyed by gastric secretions Piperacillin [Pipracil] parentral only: t1/2 - 1 hr hepatic metabolism; renal and bile elimination; renal dysfunct - t 1/2 3-6 hr and hepatic dysfunct t 1/2 - 11 hrs Ticarcillin [Ticar] parentral only more potent the Carbenicillin; hepatic metabolism; renal and bile emilination Cephalosporins - Struc differences create a more stable drug which is more resistacnt to beta- lactamase and which have a broader spectrum of activity. TOXIC: hypersensativity rxn (same as penicillin) - anyphylaxis; fever, skin rashes, nephritis, granulocytopenia, hemolytic anemia, X allergy w. penicillins of 5-10%; GÌ - nausea, vomiting, diarrhea, pseudomembranous colitis (clostridium difficile); Local irrataiton @ site of injectiong (thrombophlebitis); renal toxicity - inerstitial nepritis and tubular necrosis; hypoprothombinemia (Cefmadole, Moxalactam, Cefmetazole, Cefotetan, Cefoperazone) - dec vit K absorption in GÌ tract - dec clotting factors; Disulfiram - like rxn (Cefmadole, Moxalactam, Cefmetazole, Cefotetan, Cefoperazone) - build up of acid aldehyde - nausea , vomiting, death Super infection: eradication of normal bact - prolif of resistanct org and fungi. DRUG interactions - aminoglycoside - not a kelating effect - potentiate nephrotoxicity; probenacid - inhibit renal tubule secretion of cephalosporines; loop diuretics - potentiate nephrotoxicity. CONTRA: penacillin hypersensativity, cephalosporin hypersensativity [First Generation] Cefadroxil [Duricef] - oral Cefazolin [Ancef] - ÌV, ÌM Cephalexin [Keflex] - oral Cephapirin [Cefadyl] - ÌV, ÌM Cepharadine [Anspor] - oral SuseptibIe Orgs: (mostly gram + cocci) - Pneumococci, streptococci, stapylococci, (limited gram -) - E. coli, Klebsiella, H. Ìnfluenza; not effective against methicillin resistanct strains of staph; effect against anaerobic cocci. KINETICS: Wide distribution, does not X BBB at thereputic levels; renal elimination; dosage reduction required in pts w/ renal impairment USE: UTÌ, Minor Staphyococcal lesions, cellulitis, surgical prophylaxis (cefazolin ONLY ONE) [Second Generation] Cefaclor [ Ceclor] - oral Cefamandole [Manole] - ÌV, ÌM Cefmetazole [Zefazone] - ÌV, ÌM Cefonicid [Monocid] - ÌV, ÌM Cefotetan [Cefotan] - ÌV, ÌM Cefoxitin [Mefoxin] - ÌV, ÌM Cefprozil [Cefzil] - oral Cefuroxime [Ceftin] - oral Loracarbef [Lorabid] - oral Susceptible org: braod spectrum - same as first gen, extended gram - coverage KINETICS: widely distributed, does not penetrate the BBB, renal elimination, dosage reduction required in pts with renal impairment (renal insufficiency); Greates incidence of side effects (Cefamandole, Cefmetazole) USE: UTÌ, Lower resp tract infect, peritonitis, septicemia, minor otapholocial lesions, cellulitis Notes by T. Chengot - 2002 Page 23 [Third Generation] Cefdinir [Omnicef] - oral Cefixime [Suprax] - oral *Cefoperazone [Cefobid] - parentral Cefotaxime [Clatoran] - parentral Cefpodoxime proxetil [Banan]*- oral Ceftazidime [Ceptaz, Fortaz] -parentral Ceftibuten [Cedax] - oral Ceftizoxime [Cefizox] - parentral Cefditoren [Spectracef] - oral *Ceftriaxone [Rocephin] - parentral more gram + coverafe; less gram - coverage; anearobic coverage; resistance to Beta-lactamase; SuseptibIe oragaism same as first and second gen; extended gram - coverage except for Cefoperazone; less potent then first gen for gram + coverage; NOT effective against hydrostable enterobacter chromosomal beta-lactamase; can see resistacne due to production of cephalosporinase from serratia providencia and citrobacter KINETICS: most penetrate the meneges @ thereputic levels; Renal elimination; dosage reduction in those with renal impairments Cefoperazone [Cefobid] - extended gram - coverage except for Cefoperazone; ELIMINATION - bile and urine; Ceftizoxime [Cefizox] - Longest t 1/2 = 11 hrs; Ceftriaxone [Rocephin] - ELIMINATION - bile and urine. USE: gonorrhea, meningitis, sepsis, skine infections, upper resp infect. [Fourth Generation] Cefepime [Maxipine] Parentral Susceptible org: (braod spectrum - more resistant to some extended spectrum beta lactamases(MAJOR DIFFERENCE) that inactive 3rd gen); P. arruginosa, Enterobacteriaceae, S. Auereus, S. Pneumoniae, Heamophilus, Neisseria, good act. w/ penicillin resistant strep KINETICS: good BBB penetration, renal elimination USE: same at 3rd gen Monobactams Aztreonam [Azactam] ÌV: t1/2 = 1- 2hr MECH: same as penicillin. KINETICS: Plasma protien binding ~60%; wide distribution X's BBB and placenta; renal elimination TOXIC: GI - nausea and vomiting, Skin rashes, Elevation of serum aminotransferase; phlebitis (ÌV); Pseudmembranous Colitis Spectrum for: gram - only USE: UTÌ, lower respiratory tract infections; Septicemia Carbapenems Struct related to Beta-lactam antibiotics (resistanct to hydrolysis); less potential for hydrolysis by beta-lactamases; NOT resistant to metallo beta lactamases ass w/ - enterococcus faecium*, meth resistant staph*, Clostridium difficile*, Burkholderia cepacia*, stenotrophomonas maltrophilia resistanct. MECH - inhibit cell wall synthesis KINETICS: Wide distribution, X's BBB and placenta, low protien binding, Renal elimination TOXIC: nausea, vomiting, diarrhea, skin rashes, infusion site rxn(thromboplebitis, phlebitis), seizures (BBB Xing), esp real compromised pts USE: infection resistant to other drugs, UTÌ, Lower respiratory tract infec, pen resistant strain pneumococci, enterobacter infections, septicemia, bone and joint infections, endocarditis (stap aureus), childhood bact meningitis [Meropenem only] CONTRA: pen & cephalosporin hypersensativity, X senstivity Ìmipenem/Cilastatin [Primaxin] metabolized in the renal tubules by dehydropeptidase to toxic metbolites - renal damage; co-admin w/ Cilastin (dyhydropeptidase inhibitor); good penetration into bact, resistant to beta lactamases Spectrum for activity: gram negative rods, gram positive org, anaerobes Meropenem [Merrem ÌV] ÌV only Does not require renal dehydropeptidase inhibitor, less gram + activity compared to imipenem. Erapenem [Ìnvanz] Notes by T. Chengot - 2002 Page 24 Beta-Lactamase Inhibitors: Clavulanic Acid & Sulfactam & Tazobactam MECH: resembles beta lactam struct, but have weak antibact action (activity depends on antibact its paired with); potent inhibitor of many bact beta lactamases; most active against plasmid encoded beta lactamases - stph, H. influenza, N. gonorrheoeae, Slomonella, Shigella, E. coli, Kleb pneumoniae; Effective in chromosomal beta lactamases for legionella*, bacteroides, branhamella; NOT effective in chomosomal beta lactamases for Enterobacter*, Citrobacter*, Serratia*, Pseudomonas*; available in fixed combination products; 1) Ampicillin/subbactam sodium [Unasyn]; 2) Amoxicillin potassium clavulanate [Augmentin]; 3) Piperacillin and tozobactam sodium [Zosyn]; 4) Ticarcillin and clavulanate potassium [Timentin] Misc. Inhibitors of ceII waII synthesis Name ADMIN INFO USE/CONTRA/RXN's Vancomycin [Vancocin] - Renal elimination is really long Parentral glycopeptide antibiotic: t1/2 - 6 - 10 days MECH: same as Pen; NOT A BETA-LACTAM; Resistance - modification of D- ala-D-ala binding site; KINETICS: Oral -poor, ÌV - used most often; Wide distribution, X's BBB; Renal elimination TOXIC: Phelbitis; Chills, fever; Ototoxicity - MAJOR SÌDE EFFECT; Red - neck syndrome - flushed neck due to histamine flush; DRUG INTERACTIONS: Cholestyramine - kelate vanco and dec absorption; Aminoglycosides - potentiate ototoxicity and renal toxicity Spectrum: gram + only, particular staphyococci; USE: Sepsis, endocarditis caused by methicillin resistant staph, antibiotic associated entercolitis causeby clostridium difficile (Per os) Notes by T. Chengot - 2002 Page 25 ANTI-VIRAL AGENTS: Name ADMIN INFO USE/CONTRA/RXN's Antiherpes and Anti-CMV agents Acyclovir [Ziagen] - Acyclic guanosine derivative ÌV, topical; Oral: t1/2 = 3- 4 hr , MECH: three phosphorylations - 1) in virus - concversion to monophosphate derivative by thymidine kinase; 2) in host - conversion to diphosphates and triphosphates (competative inhibition of deoxy GTP for viral DNA polymerase and Binding to DNA template - resulting in chain termination). KINETICS: Bioavailability: 15 - 20% (oral); widely distributed X's BBB; elimination - renal. TOXIC: Nausea, diarrhea, heaches (GI -most common); Renal insufficiency and CNS effects - tremors and delirium (doses are higher, fast infusion rate = ÌV) RESISTANCE: alteration of thymidine kinase or DNA polymerase. USE: HSV-1, hsv-2, Vericella Valacyclovir [Valtrex] - Prodrug - ester of acyclovir - remember the herpes comercial with the hot girls. oh baby MECH: converts to acyclovir for activity; KINETICS: >absorption with serum levels 3-5 x greater then Oral Acyclovir, and is almost = to ÌV Acyclovir. TOXIC: = Acyclovir USE: HSV, Vericella, CMV Famciclovir [Famvir] - diacetyl ester derivative of penciclovir Penciclovir [Denavir] Ganciclovir [Cytovene] - Acyclic guanosine analog that oral and ÌV t1/2 - 2-4 hr Req. triphosphorylation for activation. MECH: ~ Acyclovir ( EXCEPTÌON - enzyme used for initial step is dependent on virus targeted); CMV - protein kinase phosphotransferase UL97; HSV - viral thymidine kinase. KINETIC: poor oral bioavailability (6-9%); wide distribution - X's BBB; Renal clearance; TOXIC: NEUTROPENIA (20-40%); CNS - headaches, mental status changes, siezures; Carcinogenic and embryotoxic (Catagory 'C') in animals; GÌ side effects USE: CMV Cidofovir [Vistide] - Cytosine nucleotide analog ÌV: t1/2=2-3 hr MECH: the phosphorylation step does not req the viral enzyme; competative inhibiton of the DNA synthesis via inhibiton of DNA polymerase and false incorporationin the viral DNA chain (~ Acyclovir); KINETICS: Renal elimination. TOXIC: Nephrotoxicity(w/in 1-2 doses), ocular hypotony (12%); RARE - NEUTROPENÌA and metabolic acidosis (directly related to nephrotoxicity) RESISTANCE: pt mutation in the DNA Polymerase; X resistance seen w/ Ganciclovir. USE: administered with Probenicid to dec renal tubular secretions (w/ CMV) CONTRA: creatinine > 1.5 mg/dl; creatinine clearance <55ml/min; urine protien >2+ (b/c of potential of nephrotoxicity) Foscarnet [Foscavir] - Ìnorganic pyrophosphate compound ÌV: t1/2=4-7 hr MECH: does not require phophorylation; Direct inhibition of DNA polymerase, RNA polymerase, and HÌV reverse transcriptase. KINETICS: wide distribution, X's BBB; Renal elimination. TOXIC: Renal insufficiency, CNS: headache(30%), hallucinations and siezures(10%); GÌ: 50% - nausea and vomiting RESISTANCE: Pt mutation in the DNA Polymerase gene (HSV and CMV); mutation in HÌV-1 reverse transcriptase gene. USE: HSV, CMV oral(F), topical(P) . MECH: Famciclovir is a prodrug of Panciclovir, req hepatic metabolism; Penciclovir required the same three phosphorylation steps as Acyclovir EXCEPT - triphosphate form blocks DNA synthesis via competative inhibition of viral DNA polymerase only (NO effect on chain termination); X resistance with Acyclovir, Valacyclovir, and Ganciclovir. KINETICS: Bioabailability = 70%, elimination is renal. TOXIC: Nausea, Diarrhea, Headache USE: HSV Notes by T. Chengot - 2002 Page 26 Fomivirsen [Vitravene] intervitreous MECH: Binds to target mRNA resulting in inhibition of immediate early region 2-protien synthesis. KINETICS: Admininstered intervitreally (w/in vitreous humor). TOXIC: Ìritis; vireitis; increased ocular preassure USE: CMV retinitis in AÌDS pts Trifluridine [Viroptic] - a fluorinated pyramidine nucleoside opthalmic topical (interaction w/ DNA synthesis). MECH: req triphosphorylation; competes with thymidine triphosphate for false incorporation into the DNA by DNA polymerase. KINETICS: opthalmologic topical. TOXIC: Ocular irritation - buring, stinging, sandy feeling; Ìncreased intraocular pressure USE: keratoconjunctivitis and epithelial keratinitis due to HSV-1 and HSV-2 Vidarabine [Vira-A] - an adenine arabinoside and is an adenosine analog. topical ointment MECH: phosphorlyation by host enzyme to ara-ATP (~Acyclovir); the ara-ATP inhibits viral DNA polymerase; ara-ATP is falsely incorporated into the viral and host DNA = cell death). KINETICS: rapid metabolism to hypoxanthine arabinoside by adenosine deaminase. TOXIC: limited side effects because it is just topical USE: HSV-1; HSV-2; Epstien-Barr virus, Varicella Antiretroviral agents: Nucleoside Reverse Transcriptase Inhibitors (NRTIs) MECH: require s the same three phosphorylation steps as Acyclovir (EXCEPT final interaction differs) - competative inhibiton of HÌV1 reverse transcriptase and incorportatuon into viral DNA chain USE: most have activity against HÌV-1 and HÌV-2 Zidovudine [AZT, Retinovir] - synthetic thyamine AZT = Azidothymidine - (a deoxythymidine analog) Oral and Parentral: t1/2 1 hr KINETICS: wide distribution including CNS; high protien binding 35%; hepatic metabolism; renal elimination. TOXIC: Myelo-suppression (anemia, neutropenia, thrombocytopenia); GÌ- intolerance; Headaches, insomnia; Hyperpigmentation of nails; Myopathy - long chronic use; Rare fatal lacticacidosis and severe hepatomegly (every reverse transcription inhibitor) USE: HÌV Didanosine [Videx] - synthetic analog of deoxyadenosine Oral: t1/2 <1.5 hr KINETICS: food decreses AUC; wide distribution including the CNS; plasma protien binding <5%; renal elimination. *Toxic: Pancreatitis (dose dependent) - major side effect; peripheral distal neuropathy (tingling in hands); Diarrhea - 25% of pts get this GI side effect; Hepatitis; esophageal ulcerations; cardiomyopathy; hyperuricemia; retinal changes; [rare reports of lactic acidosis or severe hepatomegaly (indication of removal)] USE: HÌV Lamivudine [Epivir] - is a cytosine analog orally only: t1/2 - 2.5 hrs KINETICS: NO FOOD INTERACTION; Bioavailability >80%; wide distribution, X's BBB; Plasma protien binding <5%; renal elimination. TOXÌC: headache, insomnia, fatigue (most common -30%); GI discomfort USE: HÌV and Hepatitis B Zalcitabine [Hivid] - is a cytosine analog orally only: t1/2 - 2 hrs Less effective against HÌV-1, used in combination with Zidovudine. KINETICS: food dec absorption (dec peak levels up to 39%); antacids dec absorption; wide distribution, X's BBB; plasma protien binding <2%; renal elimination. TOXIC: periperal neuropathy(50% - most common); oral and esophageal ulcerations (stomotitis - mouth); *Pancreatitis (less then didanosine); Headache, nausea, rash, and arralgias (CNS); (RARE - cardiomyopathy, lactic acidosis with severe hemmorage) USE: HÌV Stavudine [Zerit] - a thymidine analog oral only: t1/2 - 3.5 hrs KINETICS: high bioavailability - 80%; no food interactions; negligable plasma protien binding; wide distribution, X's CNS; renal elimination. TOXIC: peripheral sensory neuopathy, rash (potentiation - 52%); pancreatitis; skin rash (very common) ; elevation in serum transferase levels; (RARE - cardiomyopathy, lactic acidosis with severe hematomegaly) USE: HÌV Notes by T. Chengot - 2002 Page 27 Abacavir [Ziagen] - Guanosine analog that is more effective than other agents w/in this class oral only t 1/2 - 1.5 hrs Not metabolized by P450. KINETICS: good absorption - 83%; no food interactions; 50% plasma protien binding; wide distribution, X's BBB; metabolized by alcohol dehydrogenase and glucuronosyltransferase to inactive metabolites. TOXIC: Fatal hypersensativity rxn = 2-5% (multiple organ systems from mild to extreme, fever, malaise, GÌ complaint, skin rash); nausea, vomitning, diarrhea, headache, fatique (most common); infrequent - pancreatitis, hyperglycemia and lactic acidosis USE: HÌV Antiretroviral agents: Non- Nucleoside Reverse Transcriptase Inhibitors (NNRTIs) MECH: they do not require phosphorylation for activation; they do not compete with the endogenous nucleoside triphosphates; bind (different site) to viral reverse transcriptase that is so close to but different from the binding sites used by the nucleoside reverse transcriptase inhibitors; *this results in a blockade of RNA and DNA dependent DNA polymerase activity; little cross risitance seen w/in this class, the nucloeside reverse transcriptase inhibitors class or protease inhibitors (good) USE: HÌV-1 Nevirapine [Viramune] oral only good absorption - 90%; no food interactions; 60% plasma protien binding; wide distribution, X's BBB; Metabolized by CYP3A P450 to inactivate metabolites (inc and dec effectiveneess using hepatic inhibitors/promotors); renal elinimation. TOXIC: Mild to severe life threatening skin rashes(1-2 months 30% require hospitalization); fever, nausea, headache, somnolence, fulminant hepatitis USE: HÌV Delavirdine [Rescriptor] oral only KINETICS: good absorption - 85%; extensive plasma protien binding - 98%; wide distribution with limited penetration of CNS; Extensive Hepatic metabolism by CYP3A P450 and CYP2D P450; Able to inhibit its own metabolism therefore caution in pts with hepatic surgery. TOXIC: skin rashes (35%) first month; 5% of incidences - headaches, fatigue, nause, diarrhea, inc serum aminotransferase levels USE: HÌV Efavirenz [Sustiva] oral only: t1/2 - 40-55 hrs Formally known as DMP 266. KINETICS: good absorption 45% (inc 65% w/ fatty meal); wide distribution w/ limited penetration into the CNS; extensive hepatic metabolism by CYP3A4 and CYP2B6 P450 enzymes to inactivate metabolite; Elimination - feces. TOXIC: CNS(50%) - drowsiness, dizziness, insomnia, headaches, confusions, amneisiaagitation, delusion, depression, nightmares, euphoria; Skin rashes (28%) resolves w/ continuation of medication; nausea, vomiting; diarrhea; elevated liver enzymes; inc serum cholesterol USE: HÌV CONTRA: congenital malfomation in primate test Protease Inhibitors imp role in viral replication in the late stage protien synthesis of HÌV growth cycle. COMMON SIDE EFFECTS: cushing's-like syndrome of abnormal fat distribution, insulin resistance and hyperlipidemia; inc bleeding in hemophilia (spontaneous) Saquinavir [Ìnverase, Fortovase] oral only: t1/2 - 12 hr KINETICS: poor absorption - 4% (Ì), 12% (F); extensive plasma protien binding - 98%; wide distribution with limited penetration of CNS; Extensive first pass CYP3A P450; elimination - feces. TOXIC: GI - nausea, diarrhea, abdominal discomfort, dyspepsia; headache, lethargy, dizziness USE: HÌV Notes by T. Chengot - 2002 Page 28 Ritoavir [Crixivan] oral only: t1/2 - 3-5hr KINETICS: High bioavailability - 75% inc when taken with food; Wide distribution with limited penetration into the CNS; extensive metabolism by P450 enzymes to inactive metabolites; Elimination - feces. TOXIC: nausea, vomiting, and abdominal pain (30%); Asthenia, fever (15%); paresthsia; Elevated serum aminotransferase levels; Hyperlipidemia (45%) - cushingnoid effect USE: HÌV Ìdinavir [Crixivan] oral KINETICS: high bioavailability - 60% dec when taken with food; Wide distribution good, with penetration into the CNS; extensive metabolism by P450 enzymes to inactive metabolites; Elimination - feces. TOXIC: hyperbilirubinemia and nephrolithiasis; thrombocytopenia; elevation in serum transferase levels; nausea, diarrhea, hemolytic anemia. USE: HÌV Nelfinavir [Viracept] oral: t1/2 - 3.5- 5hr KINETICS: food inc absorption; extensive first pass by CYP3A P450; Elimination - feces. TOXIC: principle side effects; flatulance and diarrhea GÌ. USE: HÌV Amprenavir [Agenerase] oral: t1/2 - 3.5- 5hr KINETICS: no effects on absorption is seen with food; P450 metabolism. TOXIC: Rash; Paresthesia; Nausea, vomiting, diarrhea (GÌ) USE: HÌV Anti-Influenza Agents Amantadine [Symmetral] - orthostatic hypotension; peripheral edema (Amantadine - 5% of pts) Rimantadine [Flumadine] Rabavirin [Virazole] MECH: Phosphorylation by the host cells' enzymes is essential - interferes w/ Guanosine triphosphate synthesis; inhbiits capping of viral messenger RNA; inhibits viral RNA-dependent RNA polymerase. TOXIC: Primary side effect is Hemolytic anemia (10% of pts); psychiatric side effects (depression, suicidal behavior); tetrogenic in animals( CATAGORY X - embryocidal ) USE: Ìnfluenza A and B; Respiratory Syncytial Virus (RSV) Zanamivir [Relenza] Osteltamivir [Tamiflu] - PRODRUG Ìnhalation(Z), oral(O) MECH: Ìnhibiton of Neuaminidase, the enzymes essential for viral replication and release from the host cell. KIETICS: (Z) has limited plasma protien binding and rapid renal elimination; (O) is a pro-drug metabolized in the GÌ tract and liver to its active form; Renal elimination. TOXIC: (GÌ for Z and O are ~); (Z) - *GI: diarrhea, nausea, vomiting; Sinusitis, bronchitis, cough, nasal congestion (seen alot); (O) - *GI - diarrhea, nausea, vomiting; bronchitis (seen more often then placebo group); Ìnsomnia, vertigo USE: influenza A and B oral Mantadine is the derivative of Amantadine. MECH: Ìnhibition of the uncoating of viral RNA reventing viral replication; Specifically targets the M2 protien within the viral membrane that is specific for the influenza A virus (Ìnfluenza B does not have M2) TOXIC: GÌ intolerance (b/c is it oral); CNS - dizziness, anxiety, impaired coordination, insomnia, nervousness; USE: prophylaxis and treatment of Ìnfluenza A, in high risk pts - 50-90% efficacy Notes by T. Chengot - 2002 Page 29 AminogIycosides - derived from streptomycete; 2 or more amino sugar w/ glycosidic linkages to a hexose nucleus oral, topical, parentral (ÌM, ÌV) MECH: Often used with beta-lactam antibiotics to enhance activity (better penetration into cell but at high concentration both become chemical antagonist); Ìrriversible inhibition of protien synthesis (passively diffuse through the porin channels of the bact cell wall, then by energy and O2 dependent transport through cell membrane); irriversibly binds to 30S ribosome (except Streptomycin - binds 12S); cell membranes becomes leaky - lyse -cell death. KINETICS: Oral absorption is poor(,1% bioavailability); Distribution limited distribution into cells due to ionic properties; negligable binding to plasma protiens; Vd - 25%; poor CNS penetration; X's placenta and by breat milk; Acc. in reanal cortex and inner ear; Renal elimination. TOXIC: Ototoxicity - tinnitis, vertigo, deafness; nephrotoxicity - protienuria, hyaline and granular cast, reduction in glomerular filtration rate, actue tubular necrosis; NM blockade and apnea; Peripheral neuritis - injection directly into nerve; paresthesia - 30-60 mins after administration and last hrs; hypersensativity rxn - rash, blood dyscrasias; anaphylaxis; psudomembranous colitis (oral only) RESISTANCE: failure of permeation of antibiotic (must have high conc. Ìn cell); low affinity of drug to bact ribosome; inactivation of drug by microbial enzymes(most common). SPECTRUM: gram - baccilli, minimal effect on anaerobic bact CONTRA: Amninoglycoside hypersensativity rxns; intestinal obstructive or ulcerative lesions; pregnancy (deaf child); renal impaiment (nephrotoxicity). DRUG INTERACTIONS: Gen Anesthetics; NM blockers; loop diuretics(ototoxic); Nsaids and Salicylates and Cephalosporins and Vancomycin (nephrotoxic) Name ADMIN INFO USE/CONTRA/RXN's Streptomycin* ÌM least effective antibiotic of gram - rod (resistance); high ototoxicity; used in combination (beta lactams) USE: streptococcal or Enterococcal endocarditis(used w/ penicillin); Tularemia; plague; tuberculosis Gentimicin [Geramycin] ÌM, ÌV, Topical, Ìnterthecal No anerobic effect; gram negative antibiotic derived from Actinomycetes Micromonospora USE: Sepsis, Pneumonia, Endocarditis, Skin Lesions, burns (topical); meningitis, Peritonitis Tobramycin [Nebcin] - Parentral; [Tobrex] - opthalmic ÌM, ÌV, Opthalmic Similar spectrum of antimicrobial activity to Gentamicin (similar Gram - ability BUT less activite againts Serratia, more active against pseudomonas) USE: Same as gentamicin Amikacin [Amikin] ÌM, ÌV Synthetic derivative of Kanamycin; less side effects then Kanamycin (nephro and ototoxicity Ìs less); Broad spectrum agains Gram - Activity, no effect against gram + anaerobic activity; Resistant to Aminoglycoside-inactivating enzymess USE: same as gentimicin; Nosocomial gram - bacillary infection in hospitals where resistance to gentamicin and tobramycin; disseminated atypical mycobacterial infection in AÌDS pts Netilmicin [Netromycin] - newest ÌM, ÌV, Topical, Ìnterthecal gram - antibiotic derived from Actinomycetes Micromonospora; synthetic derivative of sisomicin; no \anaerobic effect; resistant to Aminoglycoside- inactivating enzymes; USE: same as gentimicin Neomycin [Mycifradin] - different then the rest Gram - and gram + (broad spectrum); limited oral and topical use - SEVERE renal and otic toxicity w/ parentral tx; Elimination in feces (unabsorbed), urine (absorbed) USE: Skin infection; preparation for elective bowel surgery; hepatic coma Kanamycin [Kantrex] oral, parentral b/c of toxicity and emergence of resistance - limited use; gram - antibiotic; *elimination via feces (unabsorbed) and urine (absorbed) USE: Tuberculosis; hepatic coma Spectinomycin [Trobicin] ÌM Aminocyclitol antibiotic prodced by streptomyces spectabilis; struc related to aminoglycosides MECH: same as aminoglycosides; Broad spectrum activity. TOXIC: Ìnsomnia, dizziness; Urticaria, puritus; nausea, vomiting; chills, fever; Anaphylaxis (any drug) USE: Acute gonorrheal urethritis in males; acute gonorrheal cervicitis and proctits in females Paromomyin [Humatin] Notes by T. Chengot - 2002 Page 30 TertracycIine Antibiotics MECH: inhibit bact protien synthesis by 1) enters bact cell via passive diffusion through porion channels in the membrane and through an energy dependent active transport pump; 2) the tetracycline concentrates w/in the bacterial; 3) reversibly binds to the 30S bacterial ribosome and prevents the aminoacyl tRNA from binding to the acceptor site on the mRNA-ribosome complex, resulting in inhibiton of protien synthesis. TOXIC: GI - nausea, vomiting, diarrhea (severe enough for discontinuation); esophagitis, esophaeal ulcers, Pseudomembranous Colitis; Dizziness, Vertigo(vestibular effectsDoxycycline); Discoloration of teeth(binds to Ca+); Growth deformation(b/c binds to Ca+ in bones); hepatic necrosis (high ÌV dosages); Renal Tubular Necrosis - Fanconi Syndrome (out dated teracycline); photosensativity (demicocycline); Pseudomembranous cerebri (benign in in intracranial pressure ass w/ children); Hypersensativity rxn. RESISTANCE: 1) dec conc in bact(efflux pump is primary method); 2) alteration of ribosomeal protiens(can't bind, can't inhibit); 3) enzymatic degradation Spectrum: tetracyclines are broad spectrum; bacteriostatic, aerobic and anaerobic gram +/- orgs, rickettsiae, mycoplasmas, chlamydiae. USE: Rickettsial infection (RMSF), mycoplasma, chlamydia, psittacosis, trachoma, STD's, brucellosis, Tularemia, Cholera, Acne DRUG INTERACTIRONS: Food - kelate; Divalent cations; Dairy products (Ca+); Alkaline pH; Antacids - bind to tertacyclines and decrease their absorption; Cimetidine - dec absorption of tetracyclines due to changes in pH; Digoxin - tetracyclines can alter GÌ flora DECREASÌNG amt of digoxin absorbed; Iron Salts - bind to tertacycline and dec GÌ adsoprtion); Litium - can inc/dec litium concentrations; Methoxyflurane - additive nephrotoxic effects; Oral contraceptives - dec effectiveness - results in break through bleeding or pregnancy - enterohepatic recirculation; Bleeding, pregnancy; Sulfonimide - potentiates photosensativity Name ADMIN INFO USE/CONTRA/RXN's Chlortetracycline [Aureomycin] - short acting oral, topical (1° use), opthalmic; t1/2 = 6-8 hr use limited to topical and opthalmic routes of administration b/c of the low oral bioavailability. TOXIC: Burning, stining, pruitus, rsh, nausea, vomiting, headache Tertracycline [Panmycin] - short acting oral, topical; t1/2 = 6-8 hr Produced semisynthetically from chortetracycline; oral bioavaillability is 60- 70%; wide distribution - bones, CNS, placenta, and breast milk; NOT metabolized; excreted in the feces Oxytetracycline [Terramycin] - short acting Oral and ÌV; t1/2 = 6-8 hr Demeclocycline [Declomycin] - intermediate acting t1/2 = 12 hr Product of mutant strain of Strep. Aureofaciens; *Main clinical use if to treat inapporpriate secretion of ADH secondary to its ability to produce nephrogenic Diabetes Incipidus; Photosensativity is more frequent and more severe then with other tetracyclines; not metabolized; renal elimination. TOXIC: as above PLUS polydipsia and polyuria Doxyxycline [Doxy] - long acting oral, ÌV, Subgingival; t1/2 = 16-18 hr Semisynthetic derivatives tetracycline dervived from oxytetracycline; tetracycline of choice in pts with poor renal function due to its limited renal clearance; Oral absorption ~ 95-100%; NO food interactions; more lipid soluable then short and intermidiate acting tetracyclines; less penetration into the CNS; NOT metabolized; elimination - feces; Neutropenia and eosinophillia - chronic useage; drug interaction with CP450 inducers (effect t1/2) Notes by T. Chengot - 2002 Page 31 Minocycline [Minocin] - long acting oral, ÌV; t1/2 = 16-18 hr Semisynthetic derivativves, most lipid soluable and considered the most active of this group; oral absorption ~95-100%; minimal hepatic metabolism (partially metabolized); excretion bile and urine(4-19%); Lupus like symptomes have been seen (autoimmune rxn) MacroIide Antibiotics MECH: Bacteriostatic agents; inhibit protien syntheis by binding reversibly to the 50S ribosomal subunits of sensative microorganisms Spectrum of activity: Bacteriostatic (low dose) and Bactericidal (high dose); broad spectrum (gram +>-); Stapylococcus aureus; Streptococcus Agalactiae; S Pyogenes; S. pnumoniae; S. Viridans; Corynebacterium diptheria, clamydia trachomatis; entamoeba histolytica; listeria monocytogenes; borrelia burgdorferi; mycoplasma pneumoniae; treponoma pallidum; ureaplsma urealyticum - (no effet on virus, yeast, fungi) TOXIC: Hypersensativity rxn - fever, eosinophilia, and skin eruptions; cholestatic hepatitis (erythromycin estolate) - nausea, vomiting, dbominal cramps, jaundice w/ fever, leukocytosis, eosinophilia, and elevated activites of transaminases; Epigastric distress - abdominal cramps, nausea, vomiting, and diarrhea; Thrombophlebitis - ÌV) DRUG INTERACTIONS: + Ergot alkaloids, digoxin, warfarin, theophylline = inhibits P450, therefore inc t1/2 RESISTANCE: 1) decrease in the permeation of the drug (efflux); 2) Dec drug bindingto the ribosomal sub unit via production of methylase enzzyme; 3) hydolysis of macrolides by esterases; 4) mutation of the 50S ribosomal protien. USE: Mycoplasma pneumoniae infections; legionnaires' disease; chlamydia infections; diptheria; pertussis, streptococcal staphylococcal, campylobacter, tetanus, syphilis, gonorrhea, Atypical mycobacterial infections; Rhumatic Fever (prophylaxis); Penicillin-allergic pts - prevention of endocarditis (Clinamycin) Name ADMIN INFO USE/CONTRA/RXN's Erythromycin* - Prototype oral, parentral: t1/2* - 1.5-2 hr (shortest t1/2) poor oral absorption - number's of different oral dosage forms are available to inc oral bioavailability; protien binding is 73-81%; wide distribution, but limited penetration to CNS; X's placenta and is distributed into breast milk; metabolism - liver ; excretion - feces and urine(minor) Clarithromycin [Biaxin] oral: t1/2 = 3-4 hr better penetration of pulmonary tissue and macrophages compared to Erythromycin; hepatic metabolism; excretion is feces and urine (minor) Dirithromycin [Dynabec] - not a prodrug (active metabolite) oral: t1/2* = 8 hr better oral bioavailability compared to Erythromycin; non-enzamatic hydrolysis to active metabolite Erythromycylamine; excretion - feces and urine(minor) Azithromycin [Zithromax] oral, ÌV; t1/2 = 68 hrs better oral bioavailability compared to Erythromycin; better penetration of tissues and reaches higher intracellular concentrations (up to 100x greater then Erythromycin); NOT metabolized**; Excreation - feces and uring(minor) CIindamycin [CIeocin] - derivative of lincomycin(too toxic; off market) oral, parentral, topical: t1/2 ~2.9 hr MECH: SAME as Erythromycin. SPRECTRUM OF ACTIVITY: Gram + (anarobic and aerobic)and anaerobic gram - (not for aerobic gram -); S. Pyogenes; S. Pneumoniae; S. Viridans; Mycoplasma Pneumoniae. KINETICS: good oral absorption, wide distribution, including bones (limited to CNS); X's placenta and distributed into breast milk; hepatic metabolism; Excretion - feces and urine(minor). TOXIC: Psuedomembranous colitis, Nausea, vomiting, hypersensativity rxn (rashes to stevens-johnson syndrome); thrombosis (ÌV); Neutropenia (rare hemotological effect) USE: anaerobic infections; acne; bacterial vaginosis Notes by T. Chengot - 2002 Page 32 Bacitracin - also in aminoglycoside group; bacteria from tracy; Polymixin B - gram - w/ detergetnt like effect on cell membrane topical Polypeptide antibiotic, consisting of a mixture of three components, bacitracin A (chief), B, and C. MECH: inhibits the incorporation of aminoacids and nucleotides into the cell wall, inhibiting bacterial cell wall synthesis. SPECTRUM: gram + cocci and bacilli . No metablism; Renal elimination TOXIC: hypersensativity RXN; Nephrotoxicty (parentral use) - very severe USE: topical treatment of skin and skin structure infections such as minor burns or skin abrasions; external opthalmic infections caused by susceptible orgs. Notes by T. Chengot - 2002 Page 33 www.brain101.info 1 SYNDROMES ENVIRONMENTAL CAUSES • ACUTE RADIATION SYNDROME: Radiation exposure. o 12 hours post-exposure: Vomiting o 24 hours post-exposure: Prostration (extreme exhaustion), fever, diarrhea o Later: Petechial hemorrhage, hypotension, tachycardia, profuse bloody diarrhea, maybe death. • CHINESE RESTAURANT SYNDROME: MSG reaction ------> Chest Pain, burning sensation over parts of body. • BROWN-SEQUARD SYNDROME: Damage (injury) to half of spinal cord ------> symptoms: o Loss of pain and temperature sensation on contralateral side of body. o Loss of proprioception and discriminatory touch on ipsilateral side of body. CARDIOVASCULAR • ADAMS-STOKES SYNDROME: Heart block, with slow or absent pulse, often accompanied by convulsions. • BARLOW SYNDROME: Floppy Mitral Valve Syndrome; Massive Mitral Valve Prolapse ------> Late apical systolic murmur, systolic click, or both. • EISENMENGER'S SYNDROME: Ventricular- Septal Defect ------> Pulmonary hypertension and cyanosis. • FLOPPY-VALVE SYNDROME: Mitral Incompetence due to myxomatous degeneration of the leaflets. • LERICHE'S SYNDROME: Occlusion of distal aorta ------> o Hip, thigh, and calf fatigue. o Impotence • BEHCET'S SYNDROME: Vasculitis ------> secondary symptoms: o Oral and genital ulcers o Uveitis o Optic atrophy • SHOULDER-HAND SYNDROME: Pain in shoulder and swelling in hand, sometimes occurring after Myocardial Infarction. • SICK SINUS SYNDROME: Chaotic atrial activity; continual changes in P-Waves. Bradycardia, alternating with recurrent ectopic beats and runs of tachycardia. • SUPERIOR VENA CAVA SYNDROME: Caused by a tumor. Obstruction of SVC ------> o Edema o Engorgement of the vessels of face, neck, and arms. o Nonproductive cough o Dyspnea • TAKAYASU'S SYNDROME: Arteritis of the Aortic Arch, resulting in no pulse. Seen in young women. • WOLF-PARKINSON WHITE SYNDROME: ECG pattern of Paroxysmal Tachycardia. o Short PR interval o Delta wave = early QRS complex. IATROGENIC (or Secondary to Medical Treatment) • AFFERENT LOOP SYNDROME: Gastrojejunal loop obstruction, proximal to a gastrojejunostomy. o Ingestion of food produces nausea, pain, and duodenal distension. • ASHERMAN'S SYNDROME: Adhesions within the endometrial cavity, causing amenorrhea and infertility. o Adhesions probably were caused by surgery. • ULYSSES SYNDROME: Ill effects from follow-up diagnostic tests following a false- positive screening test. NEOPLASTIC (Malignant or Benign) • CARCINOID SYNDROME: Carcinoid tumor producing Bradykinin + Serotonin ------> secondary symptoms: o Cyanotic flushing o Diarrhea o Bronchial spasm o Edema, ascites. • CRONKHITE-CANADA SYNDROME: GI- Polyps with diffuse alopecia (hair-loss) and nail dystrophy. o May see protein-losing enteropathy and malabsorption. • GARDNER'S SYNDROME: Multiple inherited tumors, hereditary dominant trait. o Skull osteomas, Fibromas, Epidermoid cysts o Colonic polyposis (APC gene) ------> predisposition to colonic adenocarcinoma. • LAMBERT-EATON SYNDROME: Progressive proximal muscle weakness secondary to a carcinoma. • MEIGS' SYNDROME: Fibroma of ovary with ascites and hydrothorax • PANCOAST SYNDROME: Tumor near pulmonary apex ------> www.brain101.info 2 o Neuritic pain of chest and arm o Muscle atrophy of the arm o Horner's Syndrome (impaired cervical sympathetics) • PEUTZ-JEGHERS SYNDROME: Polyposis (hamartomas) of small intestine o Also see melanin pigmentation of buccal mucosa and skin around mouth and lips CONGENITAL • CEREBELLAR SYNDROME: Congenital Cerebellar Ataxia • CERVICAL SYNDROME: Supernumerary C7 rib - -----> Pressure on brachial plexus ------> pain radiating over shoulder, arm, and forearm over C7 distribution. • DANDY-WALKER SYNDROME: Obstruction of Foramina of Magendie and Luschka in infants ------> Hydrocephalus. • DIGEORGE SYNDROME: Congenital absence of 3 rd and 4 th Branchial Arches (Thymus and Parathyroid Glands) ------> secondary symptoms: o No cell-mediated immunity ------> Frequent viral and fungal infections o Characteristic facial deformities • DOWN SYNDROME: Trisomy 21. Mental retardation, characteristic facial features, Simeon crease in hand. • FANCONI'S SYNDROME Type I: Bone-marrow hypoplasia ------> refractory anemia, pancytopenia. • EHLERS-DANLOS SYNDROME: Congenital defect in collagen. o Hyper-elasticity and friability of the skin. o Hyperextensibility of the joints. • FETAL ALCOHOL SYNDROME: Fetal malformations, growth deficiencies, craniofacial anomalies, limb defects. • GOODPASTURE'S SYNDROME: Autoantibodies against basement membranes ------> Glomerulonephritis (kidney) and hemoptysis (lungs). o Often, death by renal failure • KLINEFELTER'S SYNDROME: Trisomy XXY --- ---> testicular atrophy, increase in gonadotropins in urine. • KLIPPEL-FEIL SYNDROME: o Cervical vertebrate fused o Congenital short neck, limited neck rotation o Abnormalities of the brainstem and cerebellum o Low hairline. • LESCH-NYHAN SYNDROME: Deficiency of HGPRT (Hypoxanthine-Guanine Phospho- ribosyltransferase ------> o Hyperuricemia, uric acid kidney stones o Choreoathetosis o Mental retardation, autism, spastic cerebral palsy o X-Linked recessive • MARFAN SYNDROME: Connective Tissue disorder ------> o Arachnodactyly: Abnormally long digits and extremities o Subluxation of lens o Dissecting aortic aneurism • POSTRUBELLA SYNDROME: Infantile defects resulting from maternal Rubella infection during first trimester. o Microphthalmos, cataracts o Deafness o Mental retardation o Patent ductus arteriosis, Pulmonary arterial stenosis • PRADER-WILLI SYNDROME: Short stature, mental retardation, polyphagia with marked obesity, sexual infantilism. • RENDU-OSLER-WEBER SYNDROME: Hereditary hemorrhagic telangiectasia. • SUDDEN INFANT DEATH SYNDROME: Unexplained death in sleeping infants. • TURNER'S SYNDROME: XO monosomy. o Dwarfism o Webbed neck o Valgus of elbow. o Amenorrhea • WILSON SYNDROME: Congenital defect in Ceruloplasmin, leading to buildup of copper ----- -> mental retardation, cirrhosis, hepatolenticular degeneration. ENDOCRINE, REPRODUCTIVE • AMENNORRHEA-GALACTORRHEA SYNDROME: Non-physiologic lactation, resulting from endocrinologic causes or from a pituitary disorder. • CONN'S SYNDROME: Primary Hyperaldosteronism ------> muscular weakness, hypertension, hypokalemia, alkalosis. • CUSHING'S SYNDROME: Hypersecretion of cortisol ------> secondary symptoms and characteristics: o Fatness of face and trunk with wasting of extremities o Buffalo hump o Bone decalacification o Corticoid diabetes o Hypertension • PREMENSTRUAL SYNDROME: Abnormal sensation in breasts, abdominal pain, thirst, headache, pelvic congestion, nervous irritability. o Ocassionally nausea and vomiting. www.brain101.info 3 • SHEEHAN'S SYNDROME: Post-partum pituitary necrosis ------> hypopituitarism. • STEIN-LEVENTHAL SYNDROME: Polycystic ovary ------> infertility, amenorrhea, hirsutism. Seen in obese women. • TESTICULAR FEMINIZATION SYNDROME: Insensitivity to Testosterone. Male Psuedohermaphroditism o Complete female external genatalia, incompletely developed vagina, rudimentary uterus. PULMONARY • KARTAGENER'S SYNDROME: Situs Inversus (lateral transposition of lungs) resulting from chronic sinusitis and bronchiectasis. • HAMMAN-RICH SYNDROME: Interstitial fibrosis of the lung. • MIDDLE-LOBE SYNDROME: Chronic pneumonitis and atalectasis of middle lobe of right lung. • CHURG-STRAUSS SYNDROME: Allergic Granulomatous Angiitis: Asthma, fever, eosinophilia. INFECTIOUS • FITZ-HUGH-CURTIS SYNDROME: Gonococcal Periphepatitis in woman, as a complication of Gonorrhea. • GUILLAN-BARRE SYNDROME: Infectious Polyneuritis of unknown cause. • HUNT'S SYNDROME: Herpe's Zoster infection of Facial Nerve (CN VII) and Geniculate Ganglion ------ > facial palsy. o Zoster of ear • REYE'S SYNDROME: Loss of consciousness and seizures in kids, after a viral infection treated by aspirin. • REITER'S SYNDROME: Symptom cluster. Etiology is thought to be Chlamydial or post- chlamydial. o Urethritis o Iridocyclitis (Conjunctivitis) o Arthritis o Skin lesions like karatoderma blenorrhagicum o Also can see fatty liver or liver necrosis. • SCALDED SKIN SYNDROME: S. Aureus toxic epidermal necrolysis. • STEVENS-JOHNSON SYNDROME: Erythema Multiforme complication. o Large areas of skin slough, including mouth and anogenital membranes. o Mucous membranes: stomatitis, urethritis, conjunctivitis. o Headache, fever, malaise. • TOXIC SHOCK SYNDROME: Caused by superabsorbent tampons. Infection with Staph Aureus and subsequent toxicity of exotoxin TSST ------> systemic anaphylaxis. o Fever, vomiting, diarrhea o Red rash followed by desquamation • WATERHOUSE-FRIEDRICHSON SYNDROME: Meningeococcal Meningitis ------ > DIC, hemorrhagic infarct of adrenal glands ---- --> fulminant adrenal failure. o Vomiting, diarrhea. o Shock o Extensive purpura, cyanosis, circulatory collapse. RENAL • KEMMELSTIEL-WILSON SYNDROME: Diabetic Glomerulosclerosis. • BARTTER'S SYNDROME: Juxtaglomerular Cell Hyperplasia ------> secondary symptoms: o Hyperaldosteronism, Hypokalemic Alkalosis, elevated renin and angiotensin o No hypertension. o Compare to Conn's Syndrome • FANCONI'S SYNDROME Type II: Renal aminoaciduria, glycosuria, hypophosphaturia, cysteine deposition, rickets. • THORN'S SYNDROME: Salt-losing nephritis. NEUROLOGICAL • CARPAL-TUNNEL SYNDROME: Compression of Median Nerve through the Carpal Tunnel ------> pain and parasthesia over distribution of Median N. • FROIN'S SYNDROME: Block in CSF flow ---- --> xanthochromia (yellow discoloration) of CSF. • ACUTE-BRAIN SYNDROME: Delirium, confusion, disorientation, developing suddenly in a person that was previously psychologically normal. • GERSTMANN'S SYNDROME: Lesion between occipital area and angular gyrus ------> symptoms: o Finger agnosia, Agraphia, acalculia o Right-left disorientation • HORNER'S SYNDROME: Loss or lesion of cervical sympathetic ganglion ------> o Ptosis, miosis, anhydrosis o Enophthalmos (caved in eyes) www.brain101.info 4 • KORSAKOFF SYNDROME: Loss of short-term memory in chronic alcoholism, caused by degeneration of mamillary bodies. • RILEY-DAY SYNDROME: Familial dysautonomia. GASTROINTESTINAL • MALLORY-WEISS SYNDROME: Laceration of lower end of esophagus from vomiting ------> hematemesis. Often seen in alcoholics. • MALABSORPTION SYNDROME: Impaired absorption of dietary substance ------> diarrhea, weakness, weight loss, or symptoms from specific deficiencies. • BARRET SYNDROME: Chronic peptic ulcer of the lower esophagus, resulting in metaplasia of esophageal columnar epithelium ------> squamous epithelium. • ZOLLINGER-ELLISOHN SYNDROME: Gastrin- secreting tumor in pancreas ------> Severe peptic ulcers, gastric hyperacidity. • PLUMMER-VINSON SYNDROME: Esophageal Webs, leading to dysphagis and atrophy of papillae of tongue. o Also see hypochromic anemia, splenomegaly. RETICULOENDOTHELIAL, HEMATOLOGIC • BANTI'S SYNDROME: Chronic Congestive Splenomegaly with anemia, caused by either Portal Hypertension or Splenic Vein Thrombosis. • BUD-CHIARI SYNDROME: o ACUTE: Hepatic Vein Thrombosis ------> Massive ascites and dramatic death. o CHRONIC: Gradual hepatomegaly, portal hypertension, nausea, vomiting, edema, ulimately death. • DUBIN-JOHNSON SYNDROME: Defect in excretion of conjugated bilirubin ------> recurrent mild jaundice. Buildup of direct builirubin in blood. • CHIDIAK-HIGASHI SYNDROME: Abnormalities in leukocytes with large inclusions. • CRUVEILHIER-BAUMGARTEN SYNDROME: Symptoms cluster: o Liver cirrhosis o Caput Medussae o Venous hum and thrill • FELTY'S SYNDROME: Rheumatoid Arthritis with splenomegaly, leukopenia, anemia, and thrombocytopenia. • LOFFLER'S SYNDROME: Eosinophilia with transient infiltrates in lungs. UNCATEGORIZED • YELLOW-NAIL SYNDROME: Stop growth of nails ------> increased convexity, thickening, and yellowing of nails. o Found in Lymphedema, bronchitis, chronic bronchiectasis. • COSTOCHONDRAL SYNDROME: Pain in chest with tenderness over one or more costochondral junctions. o Similar to Tietze's Syndrome but no specific inflammation. • TIETZE'S SYNDROME: Costochondritis. Swelling and tenderness of the costal cartilege. • MIKULICZ'S SYNDROME: Salivary and lacrimal enlargement as seen in several diseases: o Sarcoidosis o Tuberculosis o Leukemia • MUNCHAUSEN SYNDROME: Malingering - - fabrication of a clinically convincing disease by an itinerant malingerer. • PICKWICKIAN SYNDROME: Symptom cluster o Obesity o Hypoventilation o Somnolence o Erythrocytosis • RESTLESS LEGS SYNDROME: Need to stretch legs at night before going to sleep; twitch in legs causing insomnia. • STRAIGHT BACK SYNDROME: Loss of normal kyphosis of thoracic spine ------> o Straight spine o Ejection murmur o Widened cardiac silouhette on x-ray • SJÖGREN'S SYNDROME: Autoimmune complex o Keratoconjuctivitis Sicca (dry eyes and mouth) o Dryness of Mucous membranes o Telangiectasias in face o Parotid enlargement 1 www.brain101.info Tutti Frutti - Step 1 Graphs, Radiology, Slides, etc: • Quality of x-rays, great; quality of histo +-; quality of macro satisfactory; quality of diagrams, too small (at least for me, extremely difficult to read letterheads); quality of graphs (curves, drugs) simple and understandable • *: right ans 1. CXR w/pneumothorax, also obvious from q stem 2. CTA, asked about liver perfusion, ?position intraart cath for injection: radiologist inserted catheter through _fem artery, aorta, celiac _ proper hepatic artery 3. Cerebral angiography; clinica of ACA infarct (leg), show artery on cerebral angiography 4. Cervical rib - cervical spine x-ray, obvious symptoms from q stem 5. Calcific tenditinis on shoulder x-ray, ?calcification: ?supraspinatus* ?subscapularis 6. Infant w/Hx of perinatal infection, non communicating hydrocephalus, sagittal MRI, show site of obstruction: _aqueductal stenosis* 7. Large SDH (acc to brain CT image); cannot exclude Epidural; however after “following day/24h”…..LOC, interval pt was “lucid”, asked about what ruptured, both choices: middle mening art (epi) and bridging veins (SDH); tricky… 8. Endometrial ca path micro (even not related to q); MOA estrogen* as carcinogen 9. 35 y o female, CIN slide, multiple partners, asked about assoc _ HPV 10. Diarrhea, slide w/probably Giardia, asked about cause of malabsorption sy 11. History of HIV w/ abn CSF(meningitis like, lymphocytes etc) and multiple ring enhancing lesions of varying sizes on CT (not showed, just described on q stem); showed histo slide (not India ink), ?bug: ?nocardia, ?toxoplasma, ?cryptococcus 12. Brainstem lesion- gross, show area according to clinica 13. Arrow on aqueduct of Sylvius -gross anatomy, ?lesion at this level; non-communicating hydrocephalus* 14. Diagram about protein translation 15. Drawing of alveolar wall (quite bad) which cell of the horrible diagram is a pneumocyte type II 16. EM slide: asked about connexons 17. EM slide: show desmosome, asked about function 18. Multiple myeloma path micro, asked about pathogenesis 2 www.brain101.info 19. ECG with extrasystole, choose the strongest complex (immediate after extrasystole*) 20. Gross spinal lesion in dorsal columns (same as CD) asked about pathogenesis 21. LOTS of physio diagrams 22. Celiac disease; path slide; asked about protein malabsorption (?) 23. Gross kidney + staghorn calculus, asked about bug: Proteus* 24. Gross kidney, young boy + Tu: asked about tu supp gene: WT1 25. Picture of pt w/superficial varices in lower extremity, ?pathogenesis (?stasis, ?insuff valves, ?chronic DVT) 26. Picture w/vesicles within great toe of kid….? Perhaps HSV, asked about virus: ans incl shape, dna/rna, single/double strand but did not mention the virus 27. Pedigree of Duchenne’s in a girl (Mec of inheritance) 28. Picture with child w/blue sclera (OI), asked about pathogenesis 29. Funduscopy of diabetic pt Pathology + Physiology: 1. Pt described with hematuria, flank mass,?disease: Renal cell ca (hypernephroma*) 2. Pt with vertebral compression # + multiple blastic lesions, most likely? Metastatic prostate adenoca 3. Pt w/Hx of working in an asbestos factory + heavy smoker, now SIADH + weight loss ?small cell* ?mesothelioma ?bronchioalveolar 4. Pt w/abn CSF, lymphocytosis, cranial nerve palsies,? most likely: TB meningitis* 5. Pt w/severe osteoporosis: arrows for PTH, ca++, P,etc know this table from BRS path very well..at least 2 or 3 qs 6. Newborn at term, infant of diabetic mother, resp distress,?most likely: Meconium aspiration sy 7. Described pt suffering from Horner’s sy, etc ?underlying disease: Lung ca* 8. Description child w/findings consistent w/ nephrotic sy , urinanalysis? ovoid fat cells*, RBCcasts, WBC casts etc 9. Atrophic pancreas + calcifications on CT, ?underlying disease: ?alcoholism*, ?gallstones 10. Pt w/incr amylase, midepigastric pain, known cholelithiasis; ?etiology: ?cbd stone, stone in ampulla?, stone in cystic duct? 11. Bartholin’s abscess 12. Pt w/hydatiform mola, ?karyotype 13. Pt a month after resection of hydatiform mola, incr hCG, ? chorioca ?residual placenta 3 www.brain101.info 14. Type of diarrhea in AIDS pt 15. Type of diarrhea in Immunocompromised 16. AIDS pt +ring enhancing lesions on CT, abn CSF, +path slide; not India ink!, ?Nocardia, ? Toxoplasma ? Cryptococcus 17. Von Willebrand /ITP/TTP 3 or 4 qs w/different values of incr, decr or N PT, PTT, etc know this well 18. Female 80 yo, on X-rays Dx of OA, pt with articular + muscular pain, given NSAID, got better, ?mostlikely: ?polymyalgia rheumatica ?RA 19. Boy with large polyp in rectum containing different tissue types: ?hamartoma, ?Angiomyolipoma etc 20. What happens to GFR, RPF, etc if u constrict efferent arteriole (arrows) 21. Soccer player injured, ligamentous rupture; didn’t wan to move, died week after,?most likely at autopsy: ?large PE*, ?small PE… 22. Female w/URI, a week after presented w/pain described to me sounding like (relieve when she lies like muslin praying etc) pericarditis* 23. Pancreatic secretion as prohormone/proenzyme (don’t remember): insulin + peptide C 24. Coupling actin/myosin, ryanodine receptor, at least 2 or 3 qs 25. Most precise for diagnosis of DIC? Incr fibrin degradation products* 26. Melanoma, ?worst prognosis: vertical growth* 27. s/p tear muscle, fibrosis at scar, what would be affected? Tension? relaxation? Etc 28. s/p terminal ileum resection, what would be affected? Lipid absorption*? Intrinsic factor secretion?( READ the stems VERY carefully….many of u ans very quickly…and I remember LOTS of qs that were like this one….vit B12 absorption would be impaired but not secretion…..) 29. Pt w/HCCa, single test that would be most diagnostic: HCV* (no option w/HBV) 30. Infant with retinal hemorrhages, fell from sofa while mother boyfriend was taking care of him: Shaken baby sy*(there’s an identical qs in webpath….) 31. Patient described w/bilateral hilar lymphadenopathy, bilateral parenchymal infiltrates, epitheliod cells, no fever, etc (sarcoidosis *, TB not an option…similar q in BRS) 32. Pt w/testicular tu? Assessment spread, what’s better: ?CT abdo, ?CT pelvis, (tricky/bad q)?palpate ingu. LN…etc 33. Pregnant female w/Sheehan’s sy ?pathogenesis: ?ac interruption hypophyseal portal system*, ?slow interruption etc 34. Inhibin regulates?: FSH* 4 www.brain101.info 35. N pt w incr pH (7.6), decr bicarb, what mixture is breathing:?20% O2, 75% CO2, 5%N2?; N2 75%, 5% CO2; 20% O2? 36. Hb shift to the right: Metabolic alkalosis, graph 37. Pt w/ systolic murmur, thrill apex, etc ?VSD* 38. Female from SE Asia, pregnant, murmur described as MVP, ?pathogenesis 39. Calculate SV from Fick’s principle 40. 16 yo girl w/1 o amenorrhea, 1.40m 45kg, ?most likely ovarian appearance: ?atrophy* (Turner’s), ?policystic 41. Endometrial ca 42. Pt w/known gralized complex sz; now described w/partial sz (only hand etc), where is the focus? temporal lobe* 43. Pt goes for routine check up on thread mill, ST elevation, coronary art OK on angiocath, ? which TX, with arrows: to decr afterload, decr CO etc 44. Headaches – types. Migraine…woman vomits, etc at least 3 or 4 qs about headaches 45. Cluster headache *: M.C in men 46. Barbiturates intoxication, ?typical liver findings on biopsy/autopsy: incr SER* (BRS path) 47. To prevent reperfusion injury in mice (during experiment): give antioxidant 1h before, just before injury etc 48. Factor XIIa*: links coagulation, kinin and complement 49. AAA incr risk for damage in/after surgery?, sigmoid colon?, jejunum?, ileum? cecum? 50. Length, tension, force velocity relationship 51. Calculate diffusion or not, given weird numbers of hydrostatic P, oncotic P etc 52. Hyperaldosteronism, different forms, many qs 53. HyperPTH, osteoporosis, ca++, P and PTH relationship w/arrows, at least 3qs 54. Diabetes, lots of qs, receptor for insulin, receptors for sulfonylureas, glucophage/lactic acidosis, Tx etc, HY! 55. Child w/GI hemorrhage, chronic anemia, given H2 blocker, gets better, ?most likely disease: GE reflux* 56. Ca thyroid _ radiation exposure* 57. CF, ?receptor: transmembrane conductance regulator* 58. Constriction efferent arteriole, ?consequences: incr GFR, decr RPF, incr filtration fraction (in arrows) 59. Drug which incr bronchial dilatation and decr heart rate, + other (?x) etc?isoproterenol? other? Not mentioned on q stem name of first drug…..kinda weird 60. which option would incr concentration of 1 25 DHD3? Hypocalcemia*(similar in Gannong) 61. Hypoxic vasoconstriction, ?Site: lungs* 5 www.brain101.info 62. Patient w/multiple skin ca types,? pathogenesis: defective dimers repair* 63. Decr FEV1sec/FEV relationship, in obstructive disease, asthma 64. Alcoholic + thyamine deficiency, ?findings in autopsy: destruction of mamillary bodies*. Anatomy: 1. Infant described w/hydrocele (transillumination etc), ?pathogenesis: patent processus vaginalis* 2. Fx medial humeral epicondyle, ?nerve injury: ulnar nerve* 3. Ant compartment sy, lower extremity, ?damage: tibial artery*, deep peroneal nerve* (2qs) 4. Pudendal block for delivery: landmark, ischial spine* 5. Injury upper trunk of brachial plexus, 2 qs 6. Stillborn, olygohydramnios, lung hypoplasia, ?most likely: kidneys’ agenesis* (Potter) 7. Hydronephrosis + hydroureter in newborn, ?pathogenesis _Post urethral valves BUT they wrote instead: congenital stenosis of membranous urethra* 8. Pt lies supine, with flexed knee, we ask to raise his leg against R about his thigh, ? which muscle are we testing: ?quadriceps femori, ?iliopsoas, etc 9. Dye passes between cells: connexons* (HY!) 10. S/p thyroidectomy, pt described w/tetany symptoms, ?Tx: give ca++ (*) 11. PTH, on surgery, found only 2 sup and 1 inf parathyroid glands, where to look for the 4 th missing one? Thymus* 12. Barbiturates toxicity, pathology in liver? Incr SER* 13. Elderly pt with distended SB on AXR and large bowel just to mid –transverse colon, lactic acidosis, ?most likely: SMA occlusion* 14. How we get retropneumoperitoneum, location of perforation: ascending colon*, jejunum, ileum, cecum, sigmoid 15. Pt described w/huge hiatal hernia, ? associated symptoms: ?lymphatic obstruction*, ?incr vagal activity, ?DVT, varicocele etc(no GER, or GI symptoms mentioned) 16. Embriology middle ear, 1 st arch?, 1 st cleft?, 1 st pouch?, 2 nd arch? 17. Why females get more peritonitis from STD?, fallopian tubes open within the peritoneum* 18. Meiosis I: 1 o oocyte +- mitotic spindle? 2 nd oocyte? Etc 19. Innervation by chorda tympani: 2/3 post tongue + submandibular+submaxill glands* 6 www.brain101.info 20. Macrophages of liver: Kupffer cells* (Kidney - mesangial cells/ Lungs - alveolar macrophages/ Brain - microglial cells/ Lymph - circulating macrophages) 21. PDA closure noted on fetal US,? Drug mother took: indomethacin* 22. In which part of the adrenal gland mineralocorticoids are secreted? Zona glomerulosa* 23. Description of 16 y old w/primary amenorrhea, short slightly overweighted (this mentioned just in measurement), poor secondary sex charact, , if u biopsy ovaries, what would u see? Atrophy?; polycystic ovaries?.etc (there were 2 qs one was clearly Turner….the other might be polycystic ovaries as well…) Biochemistry: 1. Alkaptonuria, ?parents asked u about long term prognosis: ?arthritis*, ?renal calculi 2. Pathogenesis of CGD of childhood 3. Menkes disease, MOA 4. In which part of the cell removal of the introns from mRNA take place?: nuclei*?, RER? Golgi? Etc 5. CF, MOA receptor, CF transmembrane conductance regulator gene _ defect in Cl- channels* (CF is HY, many qs) 6. SCD (but not in q stem), they said disease caused by substitution val for glu, question about which type of bond of bond is affected…. 7. Mother Hx of myelomeningocoele, son with MMC, which vitamin decreases risk for this abnormality? Folate* 8. Hypercholesterolemia, ?pathogenesis: ans description of LDL receptor (Receptors and second messengers r extremely HY!) 9. tRNA wobble diagram, asked to show site of different functions (2 qs..similar q in retired or self test) 10. Fetal alcohol sy: 2 qs 11. Tx child with hyperammonemia, etc 12. ?Reason some pts get lactic acidosis post alcohol ingestion….know everything u can about OH 13. McArdle’s described, ? enzyme: muscle phosphorilase 14. Von Gierke’s*, description disease 15. Drawing IG, where does the Ag bind? 16. Multiple skin ca (xeroderma pigmentosum), ?what’s wrong; DNA repair defect* 7 www.brain101.info 17. Mother has 2 y old son w/ MPS w/ decr iduronidase and w/N chromosomes, now she is pregnant again, amniocentesis is done, demonstrated decr iduronidase, what else is necessary to be done: ?check for MPS in amniotic fluid*, ?chromosomal analysis, ?fetal biopsy of the liver, ?mothers liver biopsy 18. Prader Willi sy, paternal imprinting, microdeletion 15q* Beh. Sciences: 1. Pt described w/delirium* post cholecystectomy 2. Girl with IDDM, not compliant, feel embarrassed about disease, etc 3. Informed consent, 2qs 4. Calculate odds ratio, all values were not given in tables, u have to make ur own table and understand where in the q stem u have to pick the values…very imp!! Lots of qs…this is not difficult and worth knowing this..of course for everything sensitivity, specificity, PPV, NPV etc 5. Asked about a new test, check values glucose many times, given the measurements, what r u checking; ?precision*, ?accuracy 6. Test with high specificity is useful to confirm a diagnosis 7. Clinical trial described u have to choose it from the options 8. Power 9. Boy w/ADHD, last year at school, he did ok, present year not, who is ashamed: ?teacher, ?parents etc (doctor at least not mentioned) 10. Bipolar disorder (student says her project would save the world, doesn’t sleep, etc..1 y before, 1month depressed all time in bed..) 11. Adolescent pt w/symptoms of depression, highest risk for suicide attempt: previous attempt w/ASA 12. Asked about narcissistic personality 13. Schyzoid pt? Schizophrenia?, etc 14. Woman doesn’t want to have sex more than once in 2 months; ?name of disorder 15. 8 y o girl with pain etc in introitum, doesn’t allow physician to check that area, starts yelling not again, Dr asks about abuse, mother gets upset, wants to leave the room, what do u do next: ?inform the mother that she has to stay until u check the girl adequately, ?Call security…. 16. 20 y o pt with ribs fractures, bruises etc while checking her she realized it’s late, and she says she has to go back home, if not her husband gets very upset, how do u approach her? 8 www.brain101.info 17. 30 + y o, mom of 3 kids, says she feel hopeless, cries all days, etc, ?TX 18. Pt referred for primary amenorrhea, eventually she is XY, and how do u explain this to the patient? 19. Pt w/ breast lesion on palpation, what do u tell the patient: feel something there, u don’t know what is it, so u would like her to have a mammo done (most reasonable option) 20. Tx of bulimia 21. Pt w/certain ca type, according to statistics: 1y sv: 90%; 2y SV: 85%; 3y SV:80%; 4y:75%; 5y sv: 70%*; pt has this ca for 2 y, ?survival by 5y ….this is very simple when u talk about survival u don’t have to make calculations….it doesn’t matter how many years that pt has the disease… 22. Q about Chlamydia in students, prevalence decrease /increase of what? (options incl they treat more pts, less awareness (don’t remember exactly…so they described u several situations that might cause these changes…so u need to understand what they r talking about, very HY epidemiology/stats!) 23. Old male with classical podagra symptoms, + incr ESR, u have to TX w/steroids, lots of AE to this TX, what makes u decide to biopsy?: If it’s going to change management*?, to show medical student? etc 24. Narcolepsy 25. Pt that does what she wants goes and chat all time at nurse station, etc, how do u treat this pt? be assertive 26. Elderly pt recovered from CVA, pt speaks slowly, daughter says he is depressed, bad mood, etc, ?what do u do next: ?ask the patient* (ALWAYS whenever possible), ?ask the nurses etc 27. Person who committed brutal suicide, which NT is decr? Serotonin, ?NE 28. Scheme about sensitivity, etc similar to CD 29. Phenotype mentally retarded, fragile X, at least 3 qs of fragile X. 30. Most important preventable cause of cancer: smoking 31. Depression 32. Normal grieving Micro, Immuno: 1. Pt w/CF and bronchiectasis, gets pneumonia, ?bug: P. aeruginosa 2. HZV, why do u give acyclovir? To reduce duration and severity of symptoms of present disease 3. virus: description disease, asked about etiology: virus not mentioned just morphologic description: DNA/ RNA, double or 9 www.brain101.info single stranded, icosahedral, naked etc (3 or 4 qs like this, 1 w/HSV, 1 w/mumps, 1w/exanthema subitum) 4. Female from NW USA, gets cardiomyopathy? Etiology? Viral? Bacterial….. 5. Nasopharyngeal ca a/w: EBV 6. Girl from rural area, no immunizations, doesn’t go to school, sore throat, high fever, exudative membranes-likes (not pseudomemebarnes; ? diphtheria, ?Group A strep (it seemed to b diphtheria..esp since they said no immunization, similar qs in Pretest vignettes) 7. boy 13 y o, swims every morning, on physical exam, pain why u press tragus, pain EAC, etc ?Bug; pseudomona 8. Nurse with sinusitis and air-fluid level on x-rays; bug needed factor V and X, ?source: from hospital?, from patient? N flora from nares?, N flora from nasopharynx? 9. Ig A deficiency vs. Hyper Ig M syndrome 10. Boy and brother get meningococcus, ?deficiency (late Complement) 11. Pt described w/HZV, Gramm and Wright stains done?findings….multinucleated giant cells… 12. Experimental mice w/Bruton’s tirosinase deficiency, which bug r they more prone to get infections from? 13. CIN a/w HPV 14. Picture w/staghorn, ?bug: proteus 15. Mother gets a cold, her child gets a more compl disease a week later w/symptoms like epiglottitis; ?bug: ?parainfluenza, ?RSV 16. Which bugs are used for quality control of a laboratory: ?E.coli + H.influenza, ?E.coli +…. 17. Pt went to Africa, came back w/malaria, given 2 drugs incl primaquine, ?why 18. Pregnant female gets CMV, ?perinatal infection, ?congenital infection etc 19. CGD described, pathogenesis? 20. HIV, why Tx w/3 drugs? For high frequency of mutational R 21. Giardia slide _ malabsorption 22. CD4 _ MHC II Pharmaco: 1. Pt treated for severe trychophytosis w/griseofulvin, what other drug works at the same timing? Paclitaxel 2. MOA acetazolamide 10 www.brain101.info 3. MOA tiazides 4. MOA loop diuretics (diuretics HY) 5. Male w/CHF, gets gynecomastia, ? drug: spironolactone 6. Tx: nephrogenic DI 7. Pt Tx w/captocril, can’t stand cough _ Losartan 8. tricyclics + urinary retention 9. Pt w/agranulocytosis: Tx: G-CSF 10. Pt with documented severe allergic reaction to penicillin, what do u give? Cephalosporins? Aztreonam? Vanco? 11. MOA Cyclosporine (asked which one suppresses IL-2….) Cyclosporin A - direct suppressive effect of B and T helper cells. 12. Tx Hay fever; choose MOA DOC 13. MOA antihistaminic drug 14. MOA Sulfonylureas 15. INH _ hepatitis 16. Procainamide + SLE 17. Flumazenil, diagram, ?competitive antagonist: chlordiazepoxide 18. heroin, now on methadone program, ?adverse effects; ?constipation, SE methadone: constipation 19. Pt treated w/captopril, get cough, change to similar: losartan 20. Prophylaxis contacts w/meningococcus: Rifampin 21. Anions, cations, + acetazolamide 22. Classic: Nortryptiline + MAO inh 23. Nitrates MOA (molecular level) activate guanylate cyclase …NO 24. Amiodarone, AE: pulmonary fibrosis 25. Milrinone 26. Female 30w pregnant, on prenatal US ductus is closed, ?drug given: indomethacin 27. Showed diagram of stomach physiol….asked which drug acts on H2 receptors……cimetidine 28. Spindle poisons _ plant alkaloids 29. Tx addison’s: fludrocortisone + glusocortic 30. tPA + MOA intrinsic plasminogen? Extrinsic? Exogenous….kinda confusing 31. 2 curves with effect of 2 drugs X and Y… why is Y steeper and lower than X? because of quicker reabsorption/ elimination/ bioavailability… volume of distribution.. etc… 32. Antagonist drug response curve 33. Parathion poisoning;atropine + ? = Pralidoxime 34. Tx migraine in asthmatic 35. Description of pt. having Absence seizures, Tx? Ethosuxomide 36. Adverse effects of lovastatin _ muscle pain. www.brain101.info 1 USMLE/COMLEX Most Common Addison’s Autoimmune (2 nd – infection) AMI - children Kawasaki disease Aneurysm Abdominal aorta Aneurysm of Circle of Willis Anterior communicating a.; bitemporal hemianopsia AV fistula Penetrating knife wound. Blindness Diabetic retinopathy Blindness - preventable Chlamydia Breast Mass Fibrocystic Change Carcinoma is the most common is post-menopausal women Bug in Acute Endocarditis Staph aureus Bug in Bacterial Arthritis in young Adults Neisseria ghonorrhea Bug in Bacterial Meningitis – elderly Strep pneumoniae Bug in Bacterial Meningitis – newborns E. coli Bug in Bacterial Meningitis – toddlers Hib Bug in Bacterial Meningitis – young adults Neisseria meningitidis Bug in debilitated, hospitalized pneumonia pt Klebsiella Bug in Epiglottitis Hib Bug in food poisoning Staph aureus Bug in GI Tract Bacteroides 2. E. coli Bug in IV drug user bacteremia / pneumonia Staph aureus Bug in opportunistic infection of AIDS CMV Pneumocystis carinii is most common overall Bug in PID N. Gonnorrhoeae or Chlamydia Bug in Pneumonia - Community - typical Strep pneumoniae 2. H. influenza 3. Klebsiella Bug in Pneumonia - Hospital aquired Klebsiella 2. Psuedomonas 3. E. coli Bug in Pneumonia - Community - atypical Mycoplasma 2. Legionella Bug in Subacute Endocarditis Strep Viridans Cancer - Leukemia - <14 yo ALL Cancer - Leukemia - >60 yo CLL Cancer - Leukemia - 15 -39 yo AML Cancer - Leukemia - 40 -60 yo CML Cancer in Gynecologic - malignancy Endometrial Carcinoma Cancer in Infancy Hemangioma Cancer in Kids Leukemia 2. Medulloblastoma of brain, cerebellum Cancer in Men Prostate CA Cancer in Women Leiomyoma aka fibroids Cancer in Women - malignant Breast CA Cancer of the Adrenal Medulla– adults Pheochromocytoma Cancer of the Adrenal Medulla– kids Neuroblastoma Cancer of the Appendix Carcinoid - rare metastasis Cancer of the Bone Metasteses from Breast & Prostate Cancer of the Bone - primary - adults Multiple Myeloma Cancer of the Brain– Child Medulloblastoma (cerebellum) Cancer of the Brain–Adult Astrocytoma (including Glioblastoma Multiforme) then: mets, meningioma, Schwannoma Cancer of the Breast Infiltrating ductal adenocarcinoma Cancer of the Connective Tissue -benign Lipoma Cancer of the Esophagus Leiomyoma Cancer of the Esophagus SCCA Cancer of the Esophagus - malignant SCC (60%) > adenocarcinoma (40%) Cancer of the Heart - adults Metasteses Cancer of the Heart - Primary– adults Myxoma “Ball Valve” Cancer of the Heart - Primary– kids Rhabdomyoma Cancer of the Liver Metastasis; lung > GI Cancer of the Liver - benign Cavernous hemangioma Cancer of the liver - Primary Hepatocellular CA Cancer of the liver - Primary, benign Cavernous Hemangioma www.brain101.info 2 Cancer of the Mouth SCC or Mucoepidermoid CA Cancer of the Mouth - upper lip Basal Cell CA Cancer of the Nasal cavities SCC Cancer of the Ovary - benign Serous cystadenoma Cancer of the Ovary - malignant Serous cystadenocarcinoma Cancer of the Pancreas Adeno (usually in the head) Cancer of the Pituitary Prolactinoma (2 nd – Somatotropic “Acidophilic” Adenoma) Cancer of the Placenta - benign Cavernous hemangioma Cancer of the Salivery Glands Pleimorphic Adenoma Cancer of the Skin Basal Cell Carcinoma Cancer of the Small Bowel Carcinoid - frequent metastasis from ileum Cancer of the Spleen - benign Cavernous hemangioma Cancer of the Stomach Gastric Adenocarcinoma; intestinal type or diffuse type Cancer of the Testicles Seminoma Cancer of the Thyroid Papillary Carcinoma Cancer that invades the Female GU tract Endometrial Adenocarcinoma Cancer; malignant lymphoma in kids Burkitt's lymphoma Cancer; Site of metastasis Regional Lymph Nodes Cancer; Site of metastasis (2 nd most common) Liver Cancer;Genetic Alteration p53 Cardiomyopathy Dilated (Congestive) Cardiomyopathy Carpal bone dislocation Lunate Carpal bone fx Scaphoid Chromosomal disorder Down’s Cirrhosis Alcohol Congenital Adrenal Hyperplasia 21-Hydroxylase Deficiency then, 11- Congenital cardiac anomaly VSD membranous > muscular Congenital early cyanosis Tetralogy of Fallot Congenital GI anomaly Meckel's diverticulum Coronary Artery thrombosis LAD Cushings Exogenous Steroid Therapy (then, primary ACTH, Adrenal Adenoma, Ectopic ACTH) Death in Alzheimer pts Pneumonia Death in Diabetics MI Death in HTN AMI 2. lenticulostriate stroke 3. renal failure (benign nephrosclerosis) Death in SLE pts. Lupus Nephropathy Type IV (Diffuse Proliferative) Death in USA Ischemic Heart Disease Dementia Alzheimer’s 2. Multi-Infarct Dementia Demyelinating Disease Multiple Sclerosis Diarrhea - children Rotavirus Dietary Deficiency Iron Disk herniation L4-L5 End-stage Renal Disease Diabetes Enzyme deficiency 21 hydroxylase - 95% of CAH Fatal genetic defect in Caucasians Cystic Fibrosis Form of Amyloidosis Immunologic Bence Jones protein in multiple myeloma is also called the Amyloid Light Chain Form of Tularemia Ulceroglandular GI obstruction Adhesions 2. Indirect inguinal hernia Glomerulonephritis IgA nephropathy aka Berger's Heart Murmur Mitral Valve Prolapse Heart Valve in bacterial endocarditis Mitral Heart Valve in bacterial endocarditis in IV drug users Tricuspid Heart Valve involved in Rheumatic Fever Mitral > Aortic Hereditary Bleeding Disorder Von Willebrand’s Disease Hernia Indirect www.brain101.info 3 Hypercalcemia hyperparathyroidism Hyperparathyroidism - Primary Adenomas (followed by: hyperplasia, then carcinoma) Hyperparathyroidism - Secondary Hypocalcemia of Chronic Renal Failure Hypertension Essentail (95%) 2. Renal disease Hypertension - children Renal disease; cystic disease, Wilm's tumor Hypertension - young women Oral contraceptives Hyperthyroidism Graves' Disease Hypopituitarism - adults Nonfunctioning pituitary adenoma Hypopituitarism - kids Craniopharyngioma Hypothyroidism Hashimoto's thyroiditis Intussusception terminal ileum into cecum Liver Disease Alcoholic Liver Disease Liver infection Viral Hepatitis - HVA Location of Adult brain tumors Above Tentorium Location of Childhood brain tumors Below Tentorium Lysosomal Storage Disease Gaucher’s Mental retardation Down’s or Fragile X , Fetal alcohol syndrome Motor Neuron Disease ALS Myocarditis Coxsackie B virus Nephrotic Syndrome - adults Membranous Glomerulonephritis Nephrotic Syndrome - kids Minimal Change Disease Opportunistic infection in AIDS PCP Peripheral Neuropathy Diabetes Portal Hypertension Cirrhosis Protozoal Diarrhea Giardia Pulmonary HTN COPD Renal Failure Acute tubular necrosis Right Heart Failure Left heart failure Secondary Hypertension Renal disease Sexually transmitted disease Chlamydia SIADH Small Cell Carcinoma of the Lung Single Gene Disorder Thalassemia Site of Diverticula Sigmoid Colon Sites of atherosclerosis Abdominal aorta > coronary > popliteal > carotid Surgical Emergency Acute Appendicitis Thyroid disease Goiter Tracheoesophageal Fistula Lower esophagus : trachea / Upper esophagus : blind pouch Type of Hodkin’s Mixed Cellularity versus: lymphocytic predominance, lymphocytic depletion, nodular sclerosis Type of Non-Hodgkin’s Follicular, small cleaved Vasculitis (of medium & small arteries) Temporal Arteritis Viral Encephalitis HSV Worm infection in US 1. Pinworm 2. Ascaris
Copyright © 2024 DOKUMEN.SITE Inc.